Slot System
Featured Buckets
Featured Buckets Admin

Severe hypercalcemia in a 54-year-old woman

Article Type
Changed
Fri, 11/01/2019 - 06:52
Display Headline
Severe hypercalcemia in a 54-year-old woman

A morbidly obese 54-year-old woman presented to the emergency department after experiencing generalized abdominal pain for 3 days. She rated the pain as 5 on a scale of 10 and described it as dull, cramping, waxing and waning, not radiating, and not relieved with changes of position—in fact, not alleviated by anything she had tried. Her pain was associated with nausea and 1 episode of vomiting. She also experienced constipation before the onset of pain.

She denied recent trauma, recent travel, diarrhea, fevers, weakness, shortness of breath, chest pain, other muscle pains, or recent changes in diet. She also denied having this pain in the past. She said she had unintentionally lost some weight but was not certain how much. She denied tobacco, alcohol, or illicit drug use. She had no history of surgery.

Her medical history included hypertension, anemia, and uterine fibroids. Her current medications included losartan, hydrochlorothiazide, and albuterol. She had no family history of significant disease.

INITIAL EVALUATION AND MANAGEMENT

On admission, her temperature was 97.8°F (36.6°C), heart rate 100 beats per minute, blood pressure 136/64 mm Hg, respiratory rate 18 breaths per minute, oxygen saturation 97% on room air, weight 130.6 kg, and body mass index 35 kg/m2.

She was alert and oriented to person, place, and time. She was in mild discomfort but no distress. Her lungs were clear to auscultation, with no wheezing or crackles. Heart rate and rhythm were regular, with no extra heart sounds or murmurs. Bowel sounds were normal in all 4 quadrants, with tenderness to palpation of the epigastric area, but with no guarding or rebound tenderness.

Laboratory test results

Notable results of blood testing at presentation were as follows:

  • Hemoglobin 8.2 g/dL (reference range 12.3–15.3)
  • Hematocrit 26% (41–50)
  • Mean corpuscular volume 107 fL (80–100)
  • Blood urea nitrogen 33 mg/dL (8–21); 6 months earlier it was 16
  • Serum creatinine 3.6 mg/dL (0.58–0.96); 6 months earlier, it was 0.75
  • Albumin 3.3 g/dL (3.5–5)
  • Calcium 18.4 mg/dL (8.4–10.2); 6 months earlier, it was 9.6
  • Corrected calcium 19 mg/dL.

Findings on imaging, electrocardiography

Chest radiography showed no acute cardiopulmonary abnormalities. Abdominal computed tomography without contrast showed no abnormalities within the pancreas and no evidence of inflammation or obstruction. Electrocardiography showed sinus tachycardia.

DIFFERENTIAL DIAGNOSIS

1. Which is the most likely cause of this patient’s symptoms?

  • Primary hyperparathyroidism
  • Malignancy
  • Her drug therapy
  • Familial hypercalcemic hypocalciuria

720tbl1.jpg
The increase in this patient’s uncorrected calcium level from 9.6 to 18.4 mg/dL in 6 months indicates some form of increased calcium resorption or retention. Moreover, her hypercalcemia is very severe (Table 1).1 Patients with severe hypercalcemia can present with life-threatening arrhythmias and seizures, as well as volume depletion.2

In total, her laboratory results were consistent with macrocytic anemia, severe hypercalcemia, and acute kidney injury, and she had generalized symptoms.

Primary hyperparathyroidism

A main cause of hypercalcemia is primary hyperparathyroidism, and this needs to be ruled out. Benign adenomas are the most common cause of primary hyperparathyroidism, and a risk factor for benign adenoma is exposure to therapeutic levels of radiation.3

In hyperparathyroidism, there is an increased secretion of parathyroid hormone (PTH), which has multiple effects including increased reabsorption of calcium from the urine, increased excretion of phosphate, and increased expression of 1,25-hydroxyvitamin D hydroxylase to activate vitamin D. PTH also stimulates osteoclasts to increase their expression of receptor activator of nuclear factor kappa B ligand (RANKL), which has a downstream effect on osteoclast precursors to cause bone reabsorption.3

Inherited primary hyperparathyroidism tends to present at a younger age, with multiple overactive parathyroid glands.3 Given our patient’s age, inherited primary hyparathyroidism is thus less likely.

 

 

Malignancy

The probability that malignancy is causing the hypercalcemia increases with calcium levels greater than 13 mg/dL. Epidemiologically, in hospitalized patients with hypercalcemia, the source tends to be malignancy.4 Typically, patients who develop hypercalcemia from malignancy have a worse prognosis.5

Solid tumors and leukemias can cause hypercalcemia. The mechanisms include humoral factors secreted by the malignancy, local osteolysis due to tumor invasion of bone, and excessive absorption of calcium due to excess vitamin D produced by malignancies.5 The cancers that most frequently cause an increase in calcium resorption are lung cancer, renal cancer, breast cancer, and multiple myeloma.1

Solid tumors with no bone metastasis and non-Hodgkin lymphoma that release PTH-related protein (PTHrP) cause humoral hypercalcemia in malignancy. The patient is typically in an advanced stage of disease. PTHrP increases serum calcium levels by decreasing the kidney’s ability to excrete calcium and by increasing bone turnover. It has no effect on intestinal absorption because of its inability to stimulate activated vitamin D3. Thus, the increase in systemic calcium comes directly from breakdown of bone and inability to excrete the excess.

PTHrP has a unique role in breast cancer: it is released locally in areas where cancer cells have metastasized to bone, but it does not cause a systemic effect. Bone resorption occurs in areas of metastasis and results from an increase in expression of RANKL and RANK in osteoclasts in response to the effects of PTHrP, leading to an increase in the production of osteoclastic cells.1

Tamoxifen, an endocrine therapy often used in breast cancer, also causes a release of bone-reabsorbing factors from tumor cells, which can partially contribute to hypercal­cemia.5

Myeloma cells secrete RANKL, which stimulates osteoclastic activity, and they also  release interleukin 6 (IL-6) and activating macrophage inflammatory protein alpha. Serum testing usually shows low or normal intact PTH, PTHrP, and 1,25-dihydroxyvitamin D.1

Patients with multiple myeloma have a worse prognosis if they have a high red blood cell distribution width, a condition shown to correlate with malnutrition, leading to deficiencies in vitamin B12 and to poor response to treatment.6 Up to 14% of patients with multiple myeloma have vitamin B12 deficiency.7

Our patient’s recent weight loss and severe hypercalcemia raise suspicion of malignancy. Further, her obesity makes proper routine breast examination difficult and thus increases the chance of undiagnosed breast cancer.8 Her decrease in renal function and her anemia complicated by hypercalcemia also raise suspicion of multiple myeloma.

Hypercalcemia due to drug therapy

Thiazide diuretics, lithium, teriparatide, and vitamin A in excessive amounts can raise the serum calcium concentration.5 Our patient was taking a thiazide for hypertension, but her extremely high calcium level places drug-induced hypercalcemia as the sole cause lower on the differential list.

Familial hypercalcemic hypocalciuria

Familial hypercalcemic hypocalciuria is a rare autosomal-dominant cause of hypercalcemia in which the ability of the body (and especially the kidneys) to sense levels of calcium is impaired, leading to a decrease in excretion of calcium in the urine.3 Very high calcium levels are rare in hypercalcemic hypocalciuria.3 In our patient with a corrected calcium concentration of nearly 19 mg/dL, familial hypercalcemic hypocalciuria is very unlikely to be the cause of the hypercalcemia.

WHAT ARE THE NEXT STEPS IN THE WORKUP?

As hypercalcemia has been confirmed, the intact PTH level should be checked to determine whether the patient’s condition is PTH-mediated. If the PTH level is in the upper range of normal or is minimally elevated, primary hyperparathyroidism is likely. Elevated PTH confirms primary hyperparathyroidism. A low-normal or low intact PTH confirms a non-PTH-mediated process, and once this is confirmed, PTHrP levels should be checked. An elevated PTHrP suggests humoral hypercalcemia of malignancy. Serum protein electrophoresis, urine protein electrophoresis, and a serum light chain assay should be performed to rule out multiple myeloma.

Vitamin D toxicity is associated with high concentrations of 1,25-dihydroxyvitamin D and 25-hydroxyvitamin D metabolites. These levels should be checked in this patient.

Other disorders that cause hypercalcemia are vitamin A toxicity and hyperthyroidism, so vitamin A and thyroid-stimulating hormone levels should also be checked.5

CASE CONTINUED

After further questioning, the patient said that she had had lower back pain about 1 to 2 weeks before coming to the emergency room; her primary care doctor had said the pain was likely from muscle strain. The pain had almost resolved but was still present.

The results of further laboratory testing were as follows:

  • Serum PTH 11 pg/mL (15–65)
  • PTHrP 3.4 pmol/L (< 2.0)
  • Protein electrophoresis showed a monoclonal (M) spike of 0.2 g/dL (0)
  • Activated vitamin D < 5 ng/mL (19.9–79.3)
  • Vitamin A 7.2 mg/dL (33.1–100)
  • Vitamin B12 194 pg/mL (239–931)
  • Thyroid-stimulating hormone 1.21 mIU/ L (0.47–4.68
  • Free thyroxine 1.27 ng/dL (0.78–2.19)
  • Iron 103 µg/dL (37–170)
  • Total iron-binding capacity 335 µg/dL (265–497)
  • Transferrin 248 mg/dL (206–381)
  • Ferritin 66 ng/mL (11.1–264)
  • Urine protein (random) 100 mg/dL (0–20)
  • Urine microalbumin (random) 5.9 mg/dL (0–1.6)
  • Urine creatinine clearance 88.5 mL/min (88–128)
  • Urine albumin-creatinine ratio 66.66 mg/g (< 30).

 

 

Imaging reports

A nuclear bone scan showed increased bone uptake in the hip and both shoulders, consistent with arthritis, and increased activity in 2 of the lower left ribs, associated with rib fractures secondary to lytic lesions. A skeletal survey at a later date showed multiple well-circumscribed “punched-out” lytic lesions in both forearms and both femurs.

2. What should be the next step in this patient’s management?

  • Intravenous (IV) fluids
  • Calcitonin
  • Bisphosphonate treatment
  • Denosumab
  • Hemodialysis

Initial treatment of severe hypercalcemia includes the following:

Start IV isotonic fluids at a rate of 150 mL/h (if the patient is making urine) to maintain urine output at more than 100 mL/h. Closely monitor urine output.

Give calcitonin 4 IU/kg in combination with IV fluids to reduce calcium levels within the first 12 to 48 hours of treatment.

Give a bisphosphonate, eg, zoledronic acid 4 mg over 15 minutes, or pamidronate 60 to 90 mg over 2 hours. Zoledronic acid is preferred in malignancy-induced hypercal­cemia because it is more potent. Doses should be adjusted in patients with renal failure.

Give denosumab if hypercalcemia is refractory to bisphosphonates, or when bisphosphonates cannot be used in renal failure.9

Hemodialysis is performed in patients who have significant neurologic symptoms irrespective of acute renal insufficiency.

Our patient was started on 0.9% sodium chloride at a rate of 150 mL/h for severe hypercalcemia. Zoledronic acid 4 mg IV was given once. These measures lowered her calcium level and lessened her acute kidney injury.

ADDITIONAL FINDINGS

Urine testing was positive for Bence Jones protein. Immune electrophoresis, performed because of suspicion of multiple myeloma, showed an elevated level of kappa light chains at 806.7 mg/dL (0.33–1.94) and normal lambda light chains at 0.62 mg/dL (0.57–2.63). The immunoglobulin G level was low at 496 mg/dL (610–1,660). In patients with severe hypercalcemia, these results point to a diagnosis of malignancy. Bone marrow aspiration study showed greater than 10% plasma cells, confirming multiple myeloma.

MULTIPLE MYELOMA

The diagnosis of multiple myeloma is based in part on the presence of 10% or more of clonal bone marrow plasma cells10 and of specific end-organ damage (anemia, hypercalcemia, renal insufficiency, or bone lesions).9

Bone marrow clonality can be shown by the ratio of kappa to lambda light chains as  detected with immunohistochemistry, immunofluorescence, or flow cytometry.11 The normal ratio is 0.26 to 1.65 for a patient with normal kidney function. In this patient, however, the ratio was 1,301.08 (806.67 kappa to 0.62 lambda), which was extremely out of range. The patient’s bone marrow biopsy results revealed the presence of 15% clonal bone marrow plasma cells.

Multiple myeloma causes osteolytic lesions through increased activation of osteoclast activating factor that stimulates the growth of osteoclast precursors. At the same time, it inhibits osteoblast formation via multiple pathways, including the action of sclerostin.11 Our patient had lytic lesions in 2 left lower ribs and in both forearms and femurs.

Hypercalcemia in multiple myeloma is attributed to 2 main factors: bone breakdown and macrophage overactivation. Multiple myeloma cells increase the release of macrophage inflammatory protein 1-alpha and tumor necrosis factor, which are inflammatory proteins that cause an increase in macrophages, which cause an increase in calcitriol.11 As noted, our patient’s calcium level at presentation was 18.4 mg/dL uncorrected and 18.96 mg/dL corrected.

Cast nephropathy can occur in the distal tubules from the increased free light chains circulating and combining with Tamm-Horsfall protein, which in turn causes obstruction and local inflammation,12 leading to a rise in creatinine levels and resulting in acute kidney injury,12 as in our patient.

TREATMENT CONSIDERATIONS IN MULTIPLE MYELOMA

Our patient was referred to an oncologist for management.

In the management of multiple myeloma, the patient’s quality of life needs to be considered. With the development of new agents to combat the damages of the osteolytic effects, there is hope for improving quality of life.13,14 New agents under study include anabolic agents such as antisclerostin and anti-Dickkopf-1, which promote osteoblastogenesis, leading to bone formation, with the possibility of repairing existing damage.15

TAKE-HOME POINTS

  • If hypercalcemia is mild to moderate, consider primary hyperparathyroidism.
  • Identify patients with severe symptoms of hypercalcemia such as volume depletion, acute kidney injury, arrhythmia, or seizures.
  • Confirm severe cases of hypercalcemia and treat severe cases effectively.
  • Severe hypercalcemia may need further investigation into a potential underlying malignancy.
References
  1. Sternlicht H, Glezerman IG. Hypercalcemia of malignancy and new treatment options. Ther Clin Risk Manag 2015; 11:1779–1788. doi:10.2147/TCRM.S83681
  2. Ahmed R, Hashiba K. Reliability of QT intervals as indicators of clinical hypercalcemia. Clin Cardiol 1988; 11(6):395–400. doi:10.1002/clc.4960110607
  3. Bilezikian JP, Cusano NE, Khan AA, Liu JM, Marcocci C, Bandeira F. Primary hyperparathyroidism. Nat Rev Dis Primers 2016; 2:16033. doi:10.1038/nrdp.2016.33
  4. Kuchay MS, Kaur P, Mishra SK, Mithal A. The changing profile of hypercalcemia in a tertiary care setting in North India: an 18-month retrospective study. Clin Cases Miner Bone Metab 2017; 14(2):131–135. doi:10.11138/ccmbm/2017.14.1.131
  5. Rosner MH, Dalkin AC. Onco-nephrology: the pathophysiology and treatment of malignancy-associated hypercalcemia. Clin J Am Soc Nephrol 2012; 7(10):1722–1729. doi:10.2215/CJN.02470312
  6. Ai L, Mu S, Hu Y. Prognostic role of RDW in hematological malignancies: a systematic review and meta-analysis. Cancer Cell Int 2018; 18:61. doi:10.1186/s12935-018-0558-3
  7. Baz R, Alemany C, Green R, Hussein MA. Prevalence of vitamin B12 deficiency in patients with plasma cell dyscrasias: a retrospective review. Cancer 2004; 101(4):790–795. doi:10.1002/cncr.20441
  8. Elmore JG, Carney PA, Abraham LA, et al. The association between obesity and screening mammography accuracy. Arch Intern Med 2004; 164(10):1140–1147. doi:10.1001/archinte.164.10.1140
  9. Gerecke C, Fuhrmann S, Strifler S, Schmidt-Hieber M, Einsele H, Knop S. The diagnosis and treatment of multiple myeloma. Dtsch Arztebl Int 2016; 113(27–28):470–476. doi:10.3238/arztebl.2016.0470
  10. Rajkumar SV. Multiple myeloma: 2016 update on diagnosis, risk-stratification, and management. Am J Hematol 2016; 91(7):719–734. doi:10.1002/ajh.24402
  11. Silbermann R, Roodman GD. Myeloma bone disease: pathophysiology and management. J Bone Oncol 2013; 2(2):59–69. doi:10.1016/j.jbo.2013.04.001
  12. Doshi M, Lahoti A, Danesh FR, Batuman V, Sanders PW; American Society of Nephrology Onco-Nephrology Forum. Paraprotein-related kidney disease: kidney injury from paraproteins—what determines the site of injury? Clin J Am Soc Nephrol 2016; 11(12):2288–2294. doi:10.2215/CJN.02560316
  13. Reece D. Update on the initial therapy of multiple myeloma. Am Soc Clin Oncol Educ Book 2013. doi:10.1200/EdBook_AM.2013.33.e307
  14. Nishida H. Bone-targeted agents in multiple myeloma. Hematol Rep 2018; 10(1):7401. doi:10.4081/hr.2018.7401
  15. Ring ES, Lawson MA, Snowden JA, Jolley I, Chantry AD. New agents in the treatment of myeloma bone disease. Calcif Tissue Int 2018; 102(2):196–209. doi:10.1007/s00223-017-0351-7
Article PDF
Author and Disclosure Information

Michael A. Munoz, MD
Department of Internal Medicine, Hospital Medicine, Saint John’s Episcopal Hospital, Far Rockaway, NY

Zeeshan Zafar, MD, MBA
Saint John’s Episcopal Hospital, Far Rockaway, NY

Benson A. Babu, MD, MBA
Department of Internal Medicine, Hospital Medicine, Northwell Health, Plainview, NY

Address: Benson A. Babu, MD, MBA, FACP, Department of Internal Medicine, Northwell Health, 888 Old Country Road, Plainview, NY 11803; bensonbabumd@gmail.com

Issue
Cleveland Clinic Journal of Medicine - 86(11)
Publications
Topics
Page Number
719-723
Legacy Keywords
hypercalcemia, high calcium, abdominal pain, hyperparathyroidism, parathyroid hormone, PTH, PTH-related protein, PTHrP, RANK ligand, RANKL, bone scan, multiple myeloma, M spike, Bence Jones protein, plasma cell, osteolytic lesions, zolendronic acid, Michael Munoz, Zeeshan Zafar, Benson Babu
Sections
Author and Disclosure Information

Michael A. Munoz, MD
Department of Internal Medicine, Hospital Medicine, Saint John’s Episcopal Hospital, Far Rockaway, NY

Zeeshan Zafar, MD, MBA
Saint John’s Episcopal Hospital, Far Rockaway, NY

Benson A. Babu, MD, MBA
Department of Internal Medicine, Hospital Medicine, Northwell Health, Plainview, NY

Address: Benson A. Babu, MD, MBA, FACP, Department of Internal Medicine, Northwell Health, 888 Old Country Road, Plainview, NY 11803; bensonbabumd@gmail.com

Author and Disclosure Information

Michael A. Munoz, MD
Department of Internal Medicine, Hospital Medicine, Saint John’s Episcopal Hospital, Far Rockaway, NY

Zeeshan Zafar, MD, MBA
Saint John’s Episcopal Hospital, Far Rockaway, NY

Benson A. Babu, MD, MBA
Department of Internal Medicine, Hospital Medicine, Northwell Health, Plainview, NY

Address: Benson A. Babu, MD, MBA, FACP, Department of Internal Medicine, Northwell Health, 888 Old Country Road, Plainview, NY 11803; bensonbabumd@gmail.com

Article PDF
Article PDF
Related Articles

A morbidly obese 54-year-old woman presented to the emergency department after experiencing generalized abdominal pain for 3 days. She rated the pain as 5 on a scale of 10 and described it as dull, cramping, waxing and waning, not radiating, and not relieved with changes of position—in fact, not alleviated by anything she had tried. Her pain was associated with nausea and 1 episode of vomiting. She also experienced constipation before the onset of pain.

She denied recent trauma, recent travel, diarrhea, fevers, weakness, shortness of breath, chest pain, other muscle pains, or recent changes in diet. She also denied having this pain in the past. She said she had unintentionally lost some weight but was not certain how much. She denied tobacco, alcohol, or illicit drug use. She had no history of surgery.

Her medical history included hypertension, anemia, and uterine fibroids. Her current medications included losartan, hydrochlorothiazide, and albuterol. She had no family history of significant disease.

INITIAL EVALUATION AND MANAGEMENT

On admission, her temperature was 97.8°F (36.6°C), heart rate 100 beats per minute, blood pressure 136/64 mm Hg, respiratory rate 18 breaths per minute, oxygen saturation 97% on room air, weight 130.6 kg, and body mass index 35 kg/m2.

She was alert and oriented to person, place, and time. She was in mild discomfort but no distress. Her lungs were clear to auscultation, with no wheezing or crackles. Heart rate and rhythm were regular, with no extra heart sounds or murmurs. Bowel sounds were normal in all 4 quadrants, with tenderness to palpation of the epigastric area, but with no guarding or rebound tenderness.

Laboratory test results

Notable results of blood testing at presentation were as follows:

  • Hemoglobin 8.2 g/dL (reference range 12.3–15.3)
  • Hematocrit 26% (41–50)
  • Mean corpuscular volume 107 fL (80–100)
  • Blood urea nitrogen 33 mg/dL (8–21); 6 months earlier it was 16
  • Serum creatinine 3.6 mg/dL (0.58–0.96); 6 months earlier, it was 0.75
  • Albumin 3.3 g/dL (3.5–5)
  • Calcium 18.4 mg/dL (8.4–10.2); 6 months earlier, it was 9.6
  • Corrected calcium 19 mg/dL.

Findings on imaging, electrocardiography

Chest radiography showed no acute cardiopulmonary abnormalities. Abdominal computed tomography without contrast showed no abnormalities within the pancreas and no evidence of inflammation or obstruction. Electrocardiography showed sinus tachycardia.

DIFFERENTIAL DIAGNOSIS

1. Which is the most likely cause of this patient’s symptoms?

  • Primary hyperparathyroidism
  • Malignancy
  • Her drug therapy
  • Familial hypercalcemic hypocalciuria

720tbl1.jpg
The increase in this patient’s uncorrected calcium level from 9.6 to 18.4 mg/dL in 6 months indicates some form of increased calcium resorption or retention. Moreover, her hypercalcemia is very severe (Table 1).1 Patients with severe hypercalcemia can present with life-threatening arrhythmias and seizures, as well as volume depletion.2

In total, her laboratory results were consistent with macrocytic anemia, severe hypercalcemia, and acute kidney injury, and she had generalized symptoms.

Primary hyperparathyroidism

A main cause of hypercalcemia is primary hyperparathyroidism, and this needs to be ruled out. Benign adenomas are the most common cause of primary hyperparathyroidism, and a risk factor for benign adenoma is exposure to therapeutic levels of radiation.3

In hyperparathyroidism, there is an increased secretion of parathyroid hormone (PTH), which has multiple effects including increased reabsorption of calcium from the urine, increased excretion of phosphate, and increased expression of 1,25-hydroxyvitamin D hydroxylase to activate vitamin D. PTH also stimulates osteoclasts to increase their expression of receptor activator of nuclear factor kappa B ligand (RANKL), which has a downstream effect on osteoclast precursors to cause bone reabsorption.3

Inherited primary hyperparathyroidism tends to present at a younger age, with multiple overactive parathyroid glands.3 Given our patient’s age, inherited primary hyparathyroidism is thus less likely.

 

 

Malignancy

The probability that malignancy is causing the hypercalcemia increases with calcium levels greater than 13 mg/dL. Epidemiologically, in hospitalized patients with hypercalcemia, the source tends to be malignancy.4 Typically, patients who develop hypercalcemia from malignancy have a worse prognosis.5

Solid tumors and leukemias can cause hypercalcemia. The mechanisms include humoral factors secreted by the malignancy, local osteolysis due to tumor invasion of bone, and excessive absorption of calcium due to excess vitamin D produced by malignancies.5 The cancers that most frequently cause an increase in calcium resorption are lung cancer, renal cancer, breast cancer, and multiple myeloma.1

Solid tumors with no bone metastasis and non-Hodgkin lymphoma that release PTH-related protein (PTHrP) cause humoral hypercalcemia in malignancy. The patient is typically in an advanced stage of disease. PTHrP increases serum calcium levels by decreasing the kidney’s ability to excrete calcium and by increasing bone turnover. It has no effect on intestinal absorption because of its inability to stimulate activated vitamin D3. Thus, the increase in systemic calcium comes directly from breakdown of bone and inability to excrete the excess.

PTHrP has a unique role in breast cancer: it is released locally in areas where cancer cells have metastasized to bone, but it does not cause a systemic effect. Bone resorption occurs in areas of metastasis and results from an increase in expression of RANKL and RANK in osteoclasts in response to the effects of PTHrP, leading to an increase in the production of osteoclastic cells.1

Tamoxifen, an endocrine therapy often used in breast cancer, also causes a release of bone-reabsorbing factors from tumor cells, which can partially contribute to hypercal­cemia.5

Myeloma cells secrete RANKL, which stimulates osteoclastic activity, and they also  release interleukin 6 (IL-6) and activating macrophage inflammatory protein alpha. Serum testing usually shows low or normal intact PTH, PTHrP, and 1,25-dihydroxyvitamin D.1

Patients with multiple myeloma have a worse prognosis if they have a high red blood cell distribution width, a condition shown to correlate with malnutrition, leading to deficiencies in vitamin B12 and to poor response to treatment.6 Up to 14% of patients with multiple myeloma have vitamin B12 deficiency.7

Our patient’s recent weight loss and severe hypercalcemia raise suspicion of malignancy. Further, her obesity makes proper routine breast examination difficult and thus increases the chance of undiagnosed breast cancer.8 Her decrease in renal function and her anemia complicated by hypercalcemia also raise suspicion of multiple myeloma.

Hypercalcemia due to drug therapy

Thiazide diuretics, lithium, teriparatide, and vitamin A in excessive amounts can raise the serum calcium concentration.5 Our patient was taking a thiazide for hypertension, but her extremely high calcium level places drug-induced hypercalcemia as the sole cause lower on the differential list.

Familial hypercalcemic hypocalciuria

Familial hypercalcemic hypocalciuria is a rare autosomal-dominant cause of hypercalcemia in which the ability of the body (and especially the kidneys) to sense levels of calcium is impaired, leading to a decrease in excretion of calcium in the urine.3 Very high calcium levels are rare in hypercalcemic hypocalciuria.3 In our patient with a corrected calcium concentration of nearly 19 mg/dL, familial hypercalcemic hypocalciuria is very unlikely to be the cause of the hypercalcemia.

WHAT ARE THE NEXT STEPS IN THE WORKUP?

As hypercalcemia has been confirmed, the intact PTH level should be checked to determine whether the patient’s condition is PTH-mediated. If the PTH level is in the upper range of normal or is minimally elevated, primary hyperparathyroidism is likely. Elevated PTH confirms primary hyperparathyroidism. A low-normal or low intact PTH confirms a non-PTH-mediated process, and once this is confirmed, PTHrP levels should be checked. An elevated PTHrP suggests humoral hypercalcemia of malignancy. Serum protein electrophoresis, urine protein electrophoresis, and a serum light chain assay should be performed to rule out multiple myeloma.

Vitamin D toxicity is associated with high concentrations of 1,25-dihydroxyvitamin D and 25-hydroxyvitamin D metabolites. These levels should be checked in this patient.

Other disorders that cause hypercalcemia are vitamin A toxicity and hyperthyroidism, so vitamin A and thyroid-stimulating hormone levels should also be checked.5

CASE CONTINUED

After further questioning, the patient said that she had had lower back pain about 1 to 2 weeks before coming to the emergency room; her primary care doctor had said the pain was likely from muscle strain. The pain had almost resolved but was still present.

The results of further laboratory testing were as follows:

  • Serum PTH 11 pg/mL (15–65)
  • PTHrP 3.4 pmol/L (< 2.0)
  • Protein electrophoresis showed a monoclonal (M) spike of 0.2 g/dL (0)
  • Activated vitamin D < 5 ng/mL (19.9–79.3)
  • Vitamin A 7.2 mg/dL (33.1–100)
  • Vitamin B12 194 pg/mL (239–931)
  • Thyroid-stimulating hormone 1.21 mIU/ L (0.47–4.68
  • Free thyroxine 1.27 ng/dL (0.78–2.19)
  • Iron 103 µg/dL (37–170)
  • Total iron-binding capacity 335 µg/dL (265–497)
  • Transferrin 248 mg/dL (206–381)
  • Ferritin 66 ng/mL (11.1–264)
  • Urine protein (random) 100 mg/dL (0–20)
  • Urine microalbumin (random) 5.9 mg/dL (0–1.6)
  • Urine creatinine clearance 88.5 mL/min (88–128)
  • Urine albumin-creatinine ratio 66.66 mg/g (< 30).

 

 

Imaging reports

A nuclear bone scan showed increased bone uptake in the hip and both shoulders, consistent with arthritis, and increased activity in 2 of the lower left ribs, associated with rib fractures secondary to lytic lesions. A skeletal survey at a later date showed multiple well-circumscribed “punched-out” lytic lesions in both forearms and both femurs.

2. What should be the next step in this patient’s management?

  • Intravenous (IV) fluids
  • Calcitonin
  • Bisphosphonate treatment
  • Denosumab
  • Hemodialysis

Initial treatment of severe hypercalcemia includes the following:

Start IV isotonic fluids at a rate of 150 mL/h (if the patient is making urine) to maintain urine output at more than 100 mL/h. Closely monitor urine output.

Give calcitonin 4 IU/kg in combination with IV fluids to reduce calcium levels within the first 12 to 48 hours of treatment.

Give a bisphosphonate, eg, zoledronic acid 4 mg over 15 minutes, or pamidronate 60 to 90 mg over 2 hours. Zoledronic acid is preferred in malignancy-induced hypercal­cemia because it is more potent. Doses should be adjusted in patients with renal failure.

Give denosumab if hypercalcemia is refractory to bisphosphonates, or when bisphosphonates cannot be used in renal failure.9

Hemodialysis is performed in patients who have significant neurologic symptoms irrespective of acute renal insufficiency.

Our patient was started on 0.9% sodium chloride at a rate of 150 mL/h for severe hypercalcemia. Zoledronic acid 4 mg IV was given once. These measures lowered her calcium level and lessened her acute kidney injury.

ADDITIONAL FINDINGS

Urine testing was positive for Bence Jones protein. Immune electrophoresis, performed because of suspicion of multiple myeloma, showed an elevated level of kappa light chains at 806.7 mg/dL (0.33–1.94) and normal lambda light chains at 0.62 mg/dL (0.57–2.63). The immunoglobulin G level was low at 496 mg/dL (610–1,660). In patients with severe hypercalcemia, these results point to a diagnosis of malignancy. Bone marrow aspiration study showed greater than 10% plasma cells, confirming multiple myeloma.

MULTIPLE MYELOMA

The diagnosis of multiple myeloma is based in part on the presence of 10% or more of clonal bone marrow plasma cells10 and of specific end-organ damage (anemia, hypercalcemia, renal insufficiency, or bone lesions).9

Bone marrow clonality can be shown by the ratio of kappa to lambda light chains as  detected with immunohistochemistry, immunofluorescence, or flow cytometry.11 The normal ratio is 0.26 to 1.65 for a patient with normal kidney function. In this patient, however, the ratio was 1,301.08 (806.67 kappa to 0.62 lambda), which was extremely out of range. The patient’s bone marrow biopsy results revealed the presence of 15% clonal bone marrow plasma cells.

Multiple myeloma causes osteolytic lesions through increased activation of osteoclast activating factor that stimulates the growth of osteoclast precursors. At the same time, it inhibits osteoblast formation via multiple pathways, including the action of sclerostin.11 Our patient had lytic lesions in 2 left lower ribs and in both forearms and femurs.

Hypercalcemia in multiple myeloma is attributed to 2 main factors: bone breakdown and macrophage overactivation. Multiple myeloma cells increase the release of macrophage inflammatory protein 1-alpha and tumor necrosis factor, which are inflammatory proteins that cause an increase in macrophages, which cause an increase in calcitriol.11 As noted, our patient’s calcium level at presentation was 18.4 mg/dL uncorrected and 18.96 mg/dL corrected.

Cast nephropathy can occur in the distal tubules from the increased free light chains circulating and combining with Tamm-Horsfall protein, which in turn causes obstruction and local inflammation,12 leading to a rise in creatinine levels and resulting in acute kidney injury,12 as in our patient.

TREATMENT CONSIDERATIONS IN MULTIPLE MYELOMA

Our patient was referred to an oncologist for management.

In the management of multiple myeloma, the patient’s quality of life needs to be considered. With the development of new agents to combat the damages of the osteolytic effects, there is hope for improving quality of life.13,14 New agents under study include anabolic agents such as antisclerostin and anti-Dickkopf-1, which promote osteoblastogenesis, leading to bone formation, with the possibility of repairing existing damage.15

TAKE-HOME POINTS

  • If hypercalcemia is mild to moderate, consider primary hyperparathyroidism.
  • Identify patients with severe symptoms of hypercalcemia such as volume depletion, acute kidney injury, arrhythmia, or seizures.
  • Confirm severe cases of hypercalcemia and treat severe cases effectively.
  • Severe hypercalcemia may need further investigation into a potential underlying malignancy.

A morbidly obese 54-year-old woman presented to the emergency department after experiencing generalized abdominal pain for 3 days. She rated the pain as 5 on a scale of 10 and described it as dull, cramping, waxing and waning, not radiating, and not relieved with changes of position—in fact, not alleviated by anything she had tried. Her pain was associated with nausea and 1 episode of vomiting. She also experienced constipation before the onset of pain.

She denied recent trauma, recent travel, diarrhea, fevers, weakness, shortness of breath, chest pain, other muscle pains, or recent changes in diet. She also denied having this pain in the past. She said she had unintentionally lost some weight but was not certain how much. She denied tobacco, alcohol, or illicit drug use. She had no history of surgery.

Her medical history included hypertension, anemia, and uterine fibroids. Her current medications included losartan, hydrochlorothiazide, and albuterol. She had no family history of significant disease.

INITIAL EVALUATION AND MANAGEMENT

On admission, her temperature was 97.8°F (36.6°C), heart rate 100 beats per minute, blood pressure 136/64 mm Hg, respiratory rate 18 breaths per minute, oxygen saturation 97% on room air, weight 130.6 kg, and body mass index 35 kg/m2.

She was alert and oriented to person, place, and time. She was in mild discomfort but no distress. Her lungs were clear to auscultation, with no wheezing or crackles. Heart rate and rhythm were regular, with no extra heart sounds or murmurs. Bowel sounds were normal in all 4 quadrants, with tenderness to palpation of the epigastric area, but with no guarding or rebound tenderness.

Laboratory test results

Notable results of blood testing at presentation were as follows:

  • Hemoglobin 8.2 g/dL (reference range 12.3–15.3)
  • Hematocrit 26% (41–50)
  • Mean corpuscular volume 107 fL (80–100)
  • Blood urea nitrogen 33 mg/dL (8–21); 6 months earlier it was 16
  • Serum creatinine 3.6 mg/dL (0.58–0.96); 6 months earlier, it was 0.75
  • Albumin 3.3 g/dL (3.5–5)
  • Calcium 18.4 mg/dL (8.4–10.2); 6 months earlier, it was 9.6
  • Corrected calcium 19 mg/dL.

Findings on imaging, electrocardiography

Chest radiography showed no acute cardiopulmonary abnormalities. Abdominal computed tomography without contrast showed no abnormalities within the pancreas and no evidence of inflammation or obstruction. Electrocardiography showed sinus tachycardia.

DIFFERENTIAL DIAGNOSIS

1. Which is the most likely cause of this patient’s symptoms?

  • Primary hyperparathyroidism
  • Malignancy
  • Her drug therapy
  • Familial hypercalcemic hypocalciuria

720tbl1.jpg
The increase in this patient’s uncorrected calcium level from 9.6 to 18.4 mg/dL in 6 months indicates some form of increased calcium resorption or retention. Moreover, her hypercalcemia is very severe (Table 1).1 Patients with severe hypercalcemia can present with life-threatening arrhythmias and seizures, as well as volume depletion.2

In total, her laboratory results were consistent with macrocytic anemia, severe hypercalcemia, and acute kidney injury, and she had generalized symptoms.

Primary hyperparathyroidism

A main cause of hypercalcemia is primary hyperparathyroidism, and this needs to be ruled out. Benign adenomas are the most common cause of primary hyperparathyroidism, and a risk factor for benign adenoma is exposure to therapeutic levels of radiation.3

In hyperparathyroidism, there is an increased secretion of parathyroid hormone (PTH), which has multiple effects including increased reabsorption of calcium from the urine, increased excretion of phosphate, and increased expression of 1,25-hydroxyvitamin D hydroxylase to activate vitamin D. PTH also stimulates osteoclasts to increase their expression of receptor activator of nuclear factor kappa B ligand (RANKL), which has a downstream effect on osteoclast precursors to cause bone reabsorption.3

Inherited primary hyperparathyroidism tends to present at a younger age, with multiple overactive parathyroid glands.3 Given our patient’s age, inherited primary hyparathyroidism is thus less likely.

 

 

Malignancy

The probability that malignancy is causing the hypercalcemia increases with calcium levels greater than 13 mg/dL. Epidemiologically, in hospitalized patients with hypercalcemia, the source tends to be malignancy.4 Typically, patients who develop hypercalcemia from malignancy have a worse prognosis.5

Solid tumors and leukemias can cause hypercalcemia. The mechanisms include humoral factors secreted by the malignancy, local osteolysis due to tumor invasion of bone, and excessive absorption of calcium due to excess vitamin D produced by malignancies.5 The cancers that most frequently cause an increase in calcium resorption are lung cancer, renal cancer, breast cancer, and multiple myeloma.1

Solid tumors with no bone metastasis and non-Hodgkin lymphoma that release PTH-related protein (PTHrP) cause humoral hypercalcemia in malignancy. The patient is typically in an advanced stage of disease. PTHrP increases serum calcium levels by decreasing the kidney’s ability to excrete calcium and by increasing bone turnover. It has no effect on intestinal absorption because of its inability to stimulate activated vitamin D3. Thus, the increase in systemic calcium comes directly from breakdown of bone and inability to excrete the excess.

PTHrP has a unique role in breast cancer: it is released locally in areas where cancer cells have metastasized to bone, but it does not cause a systemic effect. Bone resorption occurs in areas of metastasis and results from an increase in expression of RANKL and RANK in osteoclasts in response to the effects of PTHrP, leading to an increase in the production of osteoclastic cells.1

Tamoxifen, an endocrine therapy often used in breast cancer, also causes a release of bone-reabsorbing factors from tumor cells, which can partially contribute to hypercal­cemia.5

Myeloma cells secrete RANKL, which stimulates osteoclastic activity, and they also  release interleukin 6 (IL-6) and activating macrophage inflammatory protein alpha. Serum testing usually shows low or normal intact PTH, PTHrP, and 1,25-dihydroxyvitamin D.1

Patients with multiple myeloma have a worse prognosis if they have a high red blood cell distribution width, a condition shown to correlate with malnutrition, leading to deficiencies in vitamin B12 and to poor response to treatment.6 Up to 14% of patients with multiple myeloma have vitamin B12 deficiency.7

Our patient’s recent weight loss and severe hypercalcemia raise suspicion of malignancy. Further, her obesity makes proper routine breast examination difficult and thus increases the chance of undiagnosed breast cancer.8 Her decrease in renal function and her anemia complicated by hypercalcemia also raise suspicion of multiple myeloma.

Hypercalcemia due to drug therapy

Thiazide diuretics, lithium, teriparatide, and vitamin A in excessive amounts can raise the serum calcium concentration.5 Our patient was taking a thiazide for hypertension, but her extremely high calcium level places drug-induced hypercalcemia as the sole cause lower on the differential list.

Familial hypercalcemic hypocalciuria

Familial hypercalcemic hypocalciuria is a rare autosomal-dominant cause of hypercalcemia in which the ability of the body (and especially the kidneys) to sense levels of calcium is impaired, leading to a decrease in excretion of calcium in the urine.3 Very high calcium levels are rare in hypercalcemic hypocalciuria.3 In our patient with a corrected calcium concentration of nearly 19 mg/dL, familial hypercalcemic hypocalciuria is very unlikely to be the cause of the hypercalcemia.

WHAT ARE THE NEXT STEPS IN THE WORKUP?

As hypercalcemia has been confirmed, the intact PTH level should be checked to determine whether the patient’s condition is PTH-mediated. If the PTH level is in the upper range of normal or is minimally elevated, primary hyperparathyroidism is likely. Elevated PTH confirms primary hyperparathyroidism. A low-normal or low intact PTH confirms a non-PTH-mediated process, and once this is confirmed, PTHrP levels should be checked. An elevated PTHrP suggests humoral hypercalcemia of malignancy. Serum protein electrophoresis, urine protein electrophoresis, and a serum light chain assay should be performed to rule out multiple myeloma.

Vitamin D toxicity is associated with high concentrations of 1,25-dihydroxyvitamin D and 25-hydroxyvitamin D metabolites. These levels should be checked in this patient.

Other disorders that cause hypercalcemia are vitamin A toxicity and hyperthyroidism, so vitamin A and thyroid-stimulating hormone levels should also be checked.5

CASE CONTINUED

After further questioning, the patient said that she had had lower back pain about 1 to 2 weeks before coming to the emergency room; her primary care doctor had said the pain was likely from muscle strain. The pain had almost resolved but was still present.

The results of further laboratory testing were as follows:

  • Serum PTH 11 pg/mL (15–65)
  • PTHrP 3.4 pmol/L (< 2.0)
  • Protein electrophoresis showed a monoclonal (M) spike of 0.2 g/dL (0)
  • Activated vitamin D < 5 ng/mL (19.9–79.3)
  • Vitamin A 7.2 mg/dL (33.1–100)
  • Vitamin B12 194 pg/mL (239–931)
  • Thyroid-stimulating hormone 1.21 mIU/ L (0.47–4.68
  • Free thyroxine 1.27 ng/dL (0.78–2.19)
  • Iron 103 µg/dL (37–170)
  • Total iron-binding capacity 335 µg/dL (265–497)
  • Transferrin 248 mg/dL (206–381)
  • Ferritin 66 ng/mL (11.1–264)
  • Urine protein (random) 100 mg/dL (0–20)
  • Urine microalbumin (random) 5.9 mg/dL (0–1.6)
  • Urine creatinine clearance 88.5 mL/min (88–128)
  • Urine albumin-creatinine ratio 66.66 mg/g (< 30).

 

 

Imaging reports

A nuclear bone scan showed increased bone uptake in the hip and both shoulders, consistent with arthritis, and increased activity in 2 of the lower left ribs, associated with rib fractures secondary to lytic lesions. A skeletal survey at a later date showed multiple well-circumscribed “punched-out” lytic lesions in both forearms and both femurs.

2. What should be the next step in this patient’s management?

  • Intravenous (IV) fluids
  • Calcitonin
  • Bisphosphonate treatment
  • Denosumab
  • Hemodialysis

Initial treatment of severe hypercalcemia includes the following:

Start IV isotonic fluids at a rate of 150 mL/h (if the patient is making urine) to maintain urine output at more than 100 mL/h. Closely monitor urine output.

Give calcitonin 4 IU/kg in combination with IV fluids to reduce calcium levels within the first 12 to 48 hours of treatment.

Give a bisphosphonate, eg, zoledronic acid 4 mg over 15 minutes, or pamidronate 60 to 90 mg over 2 hours. Zoledronic acid is preferred in malignancy-induced hypercal­cemia because it is more potent. Doses should be adjusted in patients with renal failure.

Give denosumab if hypercalcemia is refractory to bisphosphonates, or when bisphosphonates cannot be used in renal failure.9

Hemodialysis is performed in patients who have significant neurologic symptoms irrespective of acute renal insufficiency.

Our patient was started on 0.9% sodium chloride at a rate of 150 mL/h for severe hypercalcemia. Zoledronic acid 4 mg IV was given once. These measures lowered her calcium level and lessened her acute kidney injury.

ADDITIONAL FINDINGS

Urine testing was positive for Bence Jones protein. Immune electrophoresis, performed because of suspicion of multiple myeloma, showed an elevated level of kappa light chains at 806.7 mg/dL (0.33–1.94) and normal lambda light chains at 0.62 mg/dL (0.57–2.63). The immunoglobulin G level was low at 496 mg/dL (610–1,660). In patients with severe hypercalcemia, these results point to a diagnosis of malignancy. Bone marrow aspiration study showed greater than 10% plasma cells, confirming multiple myeloma.

MULTIPLE MYELOMA

The diagnosis of multiple myeloma is based in part on the presence of 10% or more of clonal bone marrow plasma cells10 and of specific end-organ damage (anemia, hypercalcemia, renal insufficiency, or bone lesions).9

Bone marrow clonality can be shown by the ratio of kappa to lambda light chains as  detected with immunohistochemistry, immunofluorescence, or flow cytometry.11 The normal ratio is 0.26 to 1.65 for a patient with normal kidney function. In this patient, however, the ratio was 1,301.08 (806.67 kappa to 0.62 lambda), which was extremely out of range. The patient’s bone marrow biopsy results revealed the presence of 15% clonal bone marrow plasma cells.

Multiple myeloma causes osteolytic lesions through increased activation of osteoclast activating factor that stimulates the growth of osteoclast precursors. At the same time, it inhibits osteoblast formation via multiple pathways, including the action of sclerostin.11 Our patient had lytic lesions in 2 left lower ribs and in both forearms and femurs.

Hypercalcemia in multiple myeloma is attributed to 2 main factors: bone breakdown and macrophage overactivation. Multiple myeloma cells increase the release of macrophage inflammatory protein 1-alpha and tumor necrosis factor, which are inflammatory proteins that cause an increase in macrophages, which cause an increase in calcitriol.11 As noted, our patient’s calcium level at presentation was 18.4 mg/dL uncorrected and 18.96 mg/dL corrected.

Cast nephropathy can occur in the distal tubules from the increased free light chains circulating and combining with Tamm-Horsfall protein, which in turn causes obstruction and local inflammation,12 leading to a rise in creatinine levels and resulting in acute kidney injury,12 as in our patient.

TREATMENT CONSIDERATIONS IN MULTIPLE MYELOMA

Our patient was referred to an oncologist for management.

In the management of multiple myeloma, the patient’s quality of life needs to be considered. With the development of new agents to combat the damages of the osteolytic effects, there is hope for improving quality of life.13,14 New agents under study include anabolic agents such as antisclerostin and anti-Dickkopf-1, which promote osteoblastogenesis, leading to bone formation, with the possibility of repairing existing damage.15

TAKE-HOME POINTS

  • If hypercalcemia is mild to moderate, consider primary hyperparathyroidism.
  • Identify patients with severe symptoms of hypercalcemia such as volume depletion, acute kidney injury, arrhythmia, or seizures.
  • Confirm severe cases of hypercalcemia and treat severe cases effectively.
  • Severe hypercalcemia may need further investigation into a potential underlying malignancy.
References
  1. Sternlicht H, Glezerman IG. Hypercalcemia of malignancy and new treatment options. Ther Clin Risk Manag 2015; 11:1779–1788. doi:10.2147/TCRM.S83681
  2. Ahmed R, Hashiba K. Reliability of QT intervals as indicators of clinical hypercalcemia. Clin Cardiol 1988; 11(6):395–400. doi:10.1002/clc.4960110607
  3. Bilezikian JP, Cusano NE, Khan AA, Liu JM, Marcocci C, Bandeira F. Primary hyperparathyroidism. Nat Rev Dis Primers 2016; 2:16033. doi:10.1038/nrdp.2016.33
  4. Kuchay MS, Kaur P, Mishra SK, Mithal A. The changing profile of hypercalcemia in a tertiary care setting in North India: an 18-month retrospective study. Clin Cases Miner Bone Metab 2017; 14(2):131–135. doi:10.11138/ccmbm/2017.14.1.131
  5. Rosner MH, Dalkin AC. Onco-nephrology: the pathophysiology and treatment of malignancy-associated hypercalcemia. Clin J Am Soc Nephrol 2012; 7(10):1722–1729. doi:10.2215/CJN.02470312
  6. Ai L, Mu S, Hu Y. Prognostic role of RDW in hematological malignancies: a systematic review and meta-analysis. Cancer Cell Int 2018; 18:61. doi:10.1186/s12935-018-0558-3
  7. Baz R, Alemany C, Green R, Hussein MA. Prevalence of vitamin B12 deficiency in patients with plasma cell dyscrasias: a retrospective review. Cancer 2004; 101(4):790–795. doi:10.1002/cncr.20441
  8. Elmore JG, Carney PA, Abraham LA, et al. The association between obesity and screening mammography accuracy. Arch Intern Med 2004; 164(10):1140–1147. doi:10.1001/archinte.164.10.1140
  9. Gerecke C, Fuhrmann S, Strifler S, Schmidt-Hieber M, Einsele H, Knop S. The diagnosis and treatment of multiple myeloma. Dtsch Arztebl Int 2016; 113(27–28):470–476. doi:10.3238/arztebl.2016.0470
  10. Rajkumar SV. Multiple myeloma: 2016 update on diagnosis, risk-stratification, and management. Am J Hematol 2016; 91(7):719–734. doi:10.1002/ajh.24402
  11. Silbermann R, Roodman GD. Myeloma bone disease: pathophysiology and management. J Bone Oncol 2013; 2(2):59–69. doi:10.1016/j.jbo.2013.04.001
  12. Doshi M, Lahoti A, Danesh FR, Batuman V, Sanders PW; American Society of Nephrology Onco-Nephrology Forum. Paraprotein-related kidney disease: kidney injury from paraproteins—what determines the site of injury? Clin J Am Soc Nephrol 2016; 11(12):2288–2294. doi:10.2215/CJN.02560316
  13. Reece D. Update on the initial therapy of multiple myeloma. Am Soc Clin Oncol Educ Book 2013. doi:10.1200/EdBook_AM.2013.33.e307
  14. Nishida H. Bone-targeted agents in multiple myeloma. Hematol Rep 2018; 10(1):7401. doi:10.4081/hr.2018.7401
  15. Ring ES, Lawson MA, Snowden JA, Jolley I, Chantry AD. New agents in the treatment of myeloma bone disease. Calcif Tissue Int 2018; 102(2):196–209. doi:10.1007/s00223-017-0351-7
References
  1. Sternlicht H, Glezerman IG. Hypercalcemia of malignancy and new treatment options. Ther Clin Risk Manag 2015; 11:1779–1788. doi:10.2147/TCRM.S83681
  2. Ahmed R, Hashiba K. Reliability of QT intervals as indicators of clinical hypercalcemia. Clin Cardiol 1988; 11(6):395–400. doi:10.1002/clc.4960110607
  3. Bilezikian JP, Cusano NE, Khan AA, Liu JM, Marcocci C, Bandeira F. Primary hyperparathyroidism. Nat Rev Dis Primers 2016; 2:16033. doi:10.1038/nrdp.2016.33
  4. Kuchay MS, Kaur P, Mishra SK, Mithal A. The changing profile of hypercalcemia in a tertiary care setting in North India: an 18-month retrospective study. Clin Cases Miner Bone Metab 2017; 14(2):131–135. doi:10.11138/ccmbm/2017.14.1.131
  5. Rosner MH, Dalkin AC. Onco-nephrology: the pathophysiology and treatment of malignancy-associated hypercalcemia. Clin J Am Soc Nephrol 2012; 7(10):1722–1729. doi:10.2215/CJN.02470312
  6. Ai L, Mu S, Hu Y. Prognostic role of RDW in hematological malignancies: a systematic review and meta-analysis. Cancer Cell Int 2018; 18:61. doi:10.1186/s12935-018-0558-3
  7. Baz R, Alemany C, Green R, Hussein MA. Prevalence of vitamin B12 deficiency in patients with plasma cell dyscrasias: a retrospective review. Cancer 2004; 101(4):790–795. doi:10.1002/cncr.20441
  8. Elmore JG, Carney PA, Abraham LA, et al. The association between obesity and screening mammography accuracy. Arch Intern Med 2004; 164(10):1140–1147. doi:10.1001/archinte.164.10.1140
  9. Gerecke C, Fuhrmann S, Strifler S, Schmidt-Hieber M, Einsele H, Knop S. The diagnosis and treatment of multiple myeloma. Dtsch Arztebl Int 2016; 113(27–28):470–476. doi:10.3238/arztebl.2016.0470
  10. Rajkumar SV. Multiple myeloma: 2016 update on diagnosis, risk-stratification, and management. Am J Hematol 2016; 91(7):719–734. doi:10.1002/ajh.24402
  11. Silbermann R, Roodman GD. Myeloma bone disease: pathophysiology and management. J Bone Oncol 2013; 2(2):59–69. doi:10.1016/j.jbo.2013.04.001
  12. Doshi M, Lahoti A, Danesh FR, Batuman V, Sanders PW; American Society of Nephrology Onco-Nephrology Forum. Paraprotein-related kidney disease: kidney injury from paraproteins—what determines the site of injury? Clin J Am Soc Nephrol 2016; 11(12):2288–2294. doi:10.2215/CJN.02560316
  13. Reece D. Update on the initial therapy of multiple myeloma. Am Soc Clin Oncol Educ Book 2013. doi:10.1200/EdBook_AM.2013.33.e307
  14. Nishida H. Bone-targeted agents in multiple myeloma. Hematol Rep 2018; 10(1):7401. doi:10.4081/hr.2018.7401
  15. Ring ES, Lawson MA, Snowden JA, Jolley I, Chantry AD. New agents in the treatment of myeloma bone disease. Calcif Tissue Int 2018; 102(2):196–209. doi:10.1007/s00223-017-0351-7
Issue
Cleveland Clinic Journal of Medicine - 86(11)
Issue
Cleveland Clinic Journal of Medicine - 86(11)
Page Number
719-723
Page Number
719-723
Publications
Publications
Topics
Article Type
Display Headline
Severe hypercalcemia in a 54-year-old woman
Display Headline
Severe hypercalcemia in a 54-year-old woman
Legacy Keywords
hypercalcemia, high calcium, abdominal pain, hyperparathyroidism, parathyroid hormone, PTH, PTH-related protein, PTHrP, RANK ligand, RANKL, bone scan, multiple myeloma, M spike, Bence Jones protein, plasma cell, osteolytic lesions, zolendronic acid, Michael Munoz, Zeeshan Zafar, Benson Babu
Legacy Keywords
hypercalcemia, high calcium, abdominal pain, hyperparathyroidism, parathyroid hormone, PTH, PTH-related protein, PTHrP, RANK ligand, RANKL, bone scan, multiple myeloma, M spike, Bence Jones protein, plasma cell, osteolytic lesions, zolendronic acid, Michael Munoz, Zeeshan Zafar, Benson Babu
Sections
Disallow All Ads
Content Gating
No Gating (article Unlocked/Free)
Alternative CME
Disqus Comments
Default
Gate On Date
Tue, 10/29/2019 - 06:15
Un-Gate On Date
Tue, 10/29/2019 - 06:15
Use ProPublica
CFC Schedule Remove Status
Tue, 10/29/2019 - 06:15
Hide sidebar & use full width
render the right sidebar.
Article PDF Media

A 66-year-old man with abnormal thyroid function tests

Article Type
Changed
Tue, 10/01/2019 - 08:52
Display Headline
A 66-year-old man with abnormal thyroid function tests

A 66-year-old man presented to the emergency department with increasing shortness of breath and productive cough, which had begun 5 days earlier. Three years previously, he had been diagnosed with chronic obstructive pulmonary disease (COPD).

One week before the current presentation, he developed a sore throat, rhinorrhea, and nasal congestion, and the shortness of breath had started 2 days after that. Although he could speak in sentences, he was breathless even at rest. His dyspnea was associated with noisy breathing and cough productive of yellowish sputum; there was no hemoptysis. He reported fever, but he had no chills, night sweats, chest pain, or paroxysmal nocturnal dyspnea. The review of other systems was unremarkable.

His COPD was known to be mild, in Global Initiative for Chronic Obstructive Lung Disease (GOLD) grade 1, group A. His postbronchodilator ratio of forced expiratory volume in 1 second (FEV1) to forced vital capacity (FVC) was less than 0.70, and his FEV1 was 84% of predicted. Apart from mild intermittent cough with white sputum, his COPD had been under good control with inhaled ipratropium 4 times daily and inhaled albuterol as needed. He said he did not have shortness of breath except when hurrying on level ground or walking up a slight hill (Modified Medical Research Council dyspnea scale grade 1; COPD Assessment Test score < 10). In the last 3 years, he had 2 exacerbations of COPD, 1 year apart, both requiring oral prednisone and antibiotic therapy.

Other relevant history included hypertension and dyslipidemia of 15-year duration, for which he was taking candesartan 16 mg twice daily and atorvastatin 20 mg daily. He was compliant with his medications.

Though he usually received an influenza vaccine every year, he did not get it the previous year. Also, 3 years previously, he received the 23-valent pneumococcal polysaccharide vaccine (PPSV23), and the year before that he received the pneumococcal conjugate vaccine (PCV13). In addition, he was immunized against herpes zoster and tetanus.

The patient had smoked 1 pack per day for the past 38 years. His primary care physician had advised him many times to quit smoking. He had enrolled in a smoking cessation program 2 years previously, in which he received varenicline in addition to behavioral counseling in the form of motivational interviewing and a telephone quit-line. Nevertheless, he continued to smoke.

He was a retired engineer. He did not drink alcohol or use illicit drugs.

PHYSICAL EXAMINATION

On physical examination, the patient was sitting up in bed, leaning forward. He was alert and oriented but was breathing rapidly and looked sick. He had no cyanosis, clubbing, pallor, or jaundice. His blood pressure was 145/90 mm Hg, heart rate 110 beats per minute and regular, respiratory rate 29 breaths per minute, and oral temperature 38.1°C (100.6°F). His oxygen saturation was 88% while breathing room air. His body mass index was 27.1 kg/m2.

His throat was mildly congested. His neck veins were flat, and there were no carotid bruits. His thyroid examination was normal, without goiter, nodules, or tenderness.

Intercostal retractions were noted around the anterolateral costal margins. He had no chest wall deformities. Chest expansion was reduced bilaterally. There was hyperresonance bilaterally. Expiratory wheezes were heard over both lungs, without crackles.

His heart had no murmurs or added sounds. There was no lower-limb edema or swelling. The rest of his physical examination was unremarkable.

alhalaseh_thyroidfunctiontests_t1.jpg
Chest radiography showed hyperinflation without infiltrates. Electrocardiography showed normal sinus rhythm, with a peaked P wave (P pulmonale) and evidence of right ventricular hypertrophy, but no ischemic changes.

Results of initial laboratory testing are shown in Table 1.

Assessment: A 66-year-old man with GOLD grade 1, group A COPD, presenting with a severe exacerbation, most likely due to viral bronchitis.

 

 

INITIAL MANAGEMENT

The patient was given oxygen 28% by Venturi mask, and his oxygen saturation went up to 90%. He was started on nebulized albuterol 2.5 mg with ipratropium bromide 500 µg every 4 hours, prednisone 40 mg orally daily for 5 days, and ceftriaxone 1 g intravenously every 24 hours. The first dose of each medication was given in the emergency department.

The patient was then admitted to a progressive care unit, where he was placed on noninvasive positive pressure ventilation, continuous cardiac monitoring, and pulse oximetry. He was started on enoxaparin 40 mg subcutaneously daily to prevent venous thromboembolism, and the oral medications he had been taking at home were continued. Because he was receiving a glucocorticoid, his blood glucose was monitored in the fasting state, 2 hours after each meal, and as needed.

Two hours after he started noninvasive positive pressure ventilation, his arterial blood gases were remeasured and showed the following results:

  • pH 7.35
  • Partial pressure of carbon dioxide (Paco2) 52 mm Hg
  • Bicarbonate 28 mmol/L
  • Partial pressure of oxygen (Pao2) 60 mm Hg
  • Oxygen saturation 90%.

HOSPITAL COURSE

On hospital day 3, his dyspnea had slightly improved. His respiratory rate was 26 to 28 breaths per minute. His oxygen saturation remained between 90% and 92%.

At 10:21 pm, his cardiac monitor showed an episode of focal atrial tachycardia at a rate of 129 beats per minute that lasted for 3 minutes and 21 seconds, terminating spontaneously. He denied any change in his clinical condition during the episode, with no chest pain, palpitation, or change in dyspnea. There was no change in his vital signs. He had another similar asymptomatic episode lasting 4 minutes and 9 seconds at 6:30 am of hospital day 4.

Because of these episodes, the attending physician ordered thyroid function tests.

THYROID FUNCTION TESTING

1. Which thyroid function test is most likely to be helpful in the assessment of this patient’s thyroid status?

  • Serum thyroid-stimulating hormone (TSH) alone
  • Serum TSH and total thyroxine (T4)
  • Serum TSH and total triiodothyronine (T3)
  • Serum TSH and free T4
  • Serum TSH and free T3

There are several tests to assess thyroid function: the serum TSH, total T4, free T4, total T3, and free T3 concentrations.1

In normal physiology, TSH from the pituitary stimulates the thyroid gland to produce and secrete T4 and T3, which in turn inhibit TSH secretion through negative feedback. A negative log-linear relation exists between serum free T4 and TSH levels.2 Thus, the serum free T4 level can remain within the normal reference range even if the TSH level is high or low. 

TSH assays can have different detection limits. A third-generation TSH assay with a detection limit of 0.01 mU/L is recommended for use in clinical practice.3

TSH testing alone. Given its superior sensitivity and specificity, serum TSH measurement is considered the best single test for assessing thyroid function in most cases.4 Nevertheless, measurement of the serum TSH level alone could be misleading in several situations, eg, hypothalamic or pituitary disorders, recent treatment of thyrotoxicosis, impaired sensitivity to thyroid hormone, and acute nonthyroidal illness.4

alhalaseh_thyroidfunctiontests_t2.jpg
Because our patient is acutely ill, measuring his serum TSH alone is not the most appropriate test of his thyroid function. Euthyroid patients who present with acute illness usually have different patterns of abnormal thyroid function test results, depending on the severity of their illness, its stage, the drugs they are receiving, and other factors. Thyroid function test abnormalities in those patients are shown in Table 2.5–7

Free vs total T4 and T3 levels

Serum total T4 includes a fraction that is bound, mainly to thyroxin-binding globulin, and a very small unbound (free) fraction. The same applies to T3. Only free thyroid hormones represent the “active” fraction available for interaction with their protein receptors in the nucleus.8 Patients with conditions that can affect the thyroid-binding protein concentrations usually have altered serum total T4 and T3 levels, whereas their free hormone concentrations remain normal. Accordingly, measurement of free hormone levels, especially free T4, is usually recommended.

Although equilibrium dialysis is the method most likely to provide an accurate serum free T4 measurement, it is not commonly used because of its limited availability and high cost. Thus, most commercial laboratories use “direct” free T4 measurement or, to a lesser degree, the free T4 index.9 However, none of the currently available free T4 tests actually measure free T4 directly; rather, they estimate it.10

Commercial laboratories can provide a direct free T3 estimate, but it may be less reliable than total T3. If serum T3 measurement is indicated, serum total T3 is usually measured. However, total T3 measurement is rarely indicated for patients with hypothyroidism because it usually remains within the normal reference range.11 Nevertheless, serum total T3 measurement could be useful in patients with T3 toxicosis and in those who are acutely ill.

Accordingly, in acutely ill hospitalized patients like ours, measuring serum TSH using a third-generation assay and free T4 is essential to assess thyroid function. Many clinicians also measure serum total T3.

 

 

CASE CONTINUED: LOW TSH, LOW-NORMAL FREE T4, LOW TOTAL T3

The attending physician ordered serum TSH, free T4, and total T3 measurements, which yielded the following:

  • TSH 0.1 mU/L (0.5–5.0)
  • Total T3 55 ng/dL (80–180)
  • Free T4 0.9 ng/dL (0.9–2.4).

2. Which best explains this patient’s abnormal thyroid test results?

  • His acute illness
  • Central hypothyroidism due to pituitary infarction
  • His albuterol therapy
  • Subclinical thyrotoxicosis
  • Hashimoto thyroiditis

Since euthyroid patients with an acute illness may have abnormal thyroid test results (Table 2),5–7 thyroid function testing is not recommended unless there is a strong indication for it, such as new-onset atrial fibrillation, atrial flutter, or focal atrial tachycardia.1 In such patients, it is important to know whether the test abnormalities represent true thyroid disorder or are the result of a nonthyroidal illness.

alhalaseh_thyroidfunctiontests_f1.jpg
%3Cp%3EFigure%201.%20Peripheral%20conversion%20of%20thyroxine%20(T%3Csub%3E4%3C%2Fsub%3E)%20to%20triiodothyronine%20(T%3Csub%3E3%3C%2Fsub%3E)%2C%20reverse%20T%3Csub%3E3%3C%2Fsub%3E%2C%20and%20diiodothyronine%20(T%3Csub%3E2%3C%2Fsub%3E)%20by%20deiodinase%20types%201%2C%202%2C%20and%203%20(D1%2C%20D2%2C%20D3)%20in%20healthy%20people%20and%20in%20patients%20with%20nonthyroidal%20illness.%3C%2Fp%3E
In healthy people, T4 is converted to T3 (the principal active hormone) by type 1 deiodinase (D1) mainly in the liver and kidneys, whereas this reaction is catalyzed by type 2 deiodinase (D2) in the hypothalamus and pituitary. Type 3 deiodinase (D3) converts T4 to reverse T3, a biologically inactive molecule.12 D1 also mediates conversion of reverse T3 to diiodothyronine (T2) (Figure 1).

alhalaseh_thyroidfunctiontests_t3.jpg
Several conditions and drugs can decrease D1 activity, resulting in low serum T3 concentrations (Table 3). In patients with nonthyroidal illness, decreased D1 activity can be observed as early as the first 24 hours after the onset of the illness and is attributed to increased inflammatory cytokines, free fatty acids, increased endogenous cortisol secretion, and use of certain drugs.13,14 In addition, the reduced D1 activity can decrease the conversion of reverse T3 to T2, resulting in elevated serum reverse T3. Increased D3 activity during an acute illness is another mechanism for elevated serum reverse T3 concentration.15

Thyroid function testing in patients with nonthyroidal illness usually shows low serum total T3, normal or low serum TSH, and normal, low, or high serum free T4. However, transient mild serum TSH elevation can be seen in some patients during the recovery period.16 These abnormalities with their mechanisms are shown in Table 2.5–7 In several commercial kits, serum direct free T4 can be falsely decreased or increased.8

THE DIFFERENTIAL DIAGNOSIS

Our patient had low serum TSH, low-normal serum direct free T4, and low serum total T3. This profile could be caused by a nonthyroidal illness, “true” central hypothyroidism, or his glucocorticoid treatment. The reason we use the term “true” in this setting is that some experts suggest that the thyroid function test abnormalities in patients with acute nonthyroidal illness represent a transient central hypothyroidism.17 The clinical presentation is key in differentiating true central hypothyroidism from nonthyroidal illness.

In addition, measuring serum cortisol may help to differentiate between the 2 states, as it would be elevated in patients with nonthyroidal illness as part of a stress response but low in patients with true central hypothyroidism, since it is usually part of combined pituitary hormone deficiency.18 Of note, some critically ill patients have low serum cortisol because of transient central adrenal insufficiency.19,20

The serum concentration of reverse T3 has been suggested as a way to differentiate between hypothyroidism (low) and nonthyroidal illness (high); however, further studies showed that it does not reliably differentiate between the conditions.21

GLUCOCORTICOIDS AND THYROID FUNCTION TESTS

By inhibiting D1, glucocorticoids can decrease peripheral conversion of T4 to T3 and thus decrease serum total T3. This effect depends on the type and dose of the glucocorticoid and the duration of therapy.

In one study,22 there was a significant reduction in serum total T3 concentration 24 hours after a single oral dose of dexamethasone 12 mg in normal participants. This effect lasted 48 hours, after which serum total T3 returned to its pretreatment level.

In another study,23 a daily oral dose of betamethasone 1.5 mg for 5 days did not significantly reduce the serum total T3 in healthy volunteers, but a daily dose of 3 mg did. This effect was more pronounced at a daily dose of 4.5 mg, whereas a dose of 6.0 mg had no further effect.

Long-term glucocorticoid therapy also decreases serum total T4 and total T3 by lowering serum thyroid-binding globulin.24

Finally, glucocorticoids can decrease TSH secretion by directly inhibiting thyrotropin-releasing hormone.25,26 However, chronic hypercortisolism, whether endogenous or exogenous, does not cause clinically central hypothyroidism, possibly because of the negative feedback mechanism of low thyroid hormones on the pituitary and the hypothalamus.27

Other drugs including dopamine, dopamine agonists, dobutamine, and somatostatin analogues can suppress serum TSH. As with glucocorticoids, these drugs do not cause clinically evident central hypothyroidism.28 Bexarotene, a retinoid X receptor ligand used in the treatment of cutaneous T-cell lymphoma, has been reported to cause clinically evident central hypothyroidism by suppressing TSH and increasing T4 clearance.29

 

 

BETA-BLOCKERS, BETA-AGONISTS AND THYROID FUNCTION

While there is general agreement that beta-adrenergic antagonists (beta-blockers) do not affect the serum TSH concentration, conflicting data have been reported concerning their effect on other thyroid function tests. This may be due to several factors, including dose, duration of therapy, the patient’s thyroid status, and differences in laboratory methodology.30

In studies of propranolol, serum total T4 concentrations did not change or were increased with daily doses of 160 mg or more in both euthyroid participants and hyperthyroid patients31–33; serum total T3 concentrations did not change or were decreased with 40 mg or more daily34; and serum reverse T3 concentrations were increased with daily doses of 80 mg or more.31 It is most likely that propranolol exerts these changes by inhibiting D1 activity in peripheral tissues.

Furthermore, a significant decrease in serum total T3 concentrations was observed in hyperthyroid patients treated with atenolol 100 mg daily, metoprolol 100 mg daily, and alprenolol 100 mg daily, but not with sotalol 80 mg daily or nadolol (up to 240 mg daily).35,36

On the other hand, beta-adrenergic agonists have not been reported to cause significant changes in thyroid function tests.37

SUBCLINICAL THYROTOXICOSIS OR HASHIMOTO THYROIDITIS?

Our patient’s thyroid function test results are more likely due to his nonthyroidal illness and glucocorticoid therapy, as there is no clinical evidence to point to a hypothalamic-pituitary disorder accounting for true central hypothyroidism.

The other options mentioned in question 2 are unlikely to explain our patient’s thyroid function test results.

Subclinical thyrotoxicosis is characterized by suppressed serum TSH, but both serum free T4 and total T3 remain within the normal reference ranges. In addition, the serum TSH level may help to differentiate between thyrotoxicosis and nonthyroidal illness. In the former, serum TSH is usually suppressed (< 0.01 mU/L), whereas in the latter it is usually low but detectable (0.05– 0.3 mU/L).38,39

Hashimoto thyroiditis is a chronic autoimmune thyroid disease characterized by diffuse lymphocytic infiltration of the thyroid gland. Almost all patients with Hashimoto thyroiditis have elevated levels of antibodies to thyroid peroxidase or thyroglobulin.40 Clinically, patients with Hashimoto thyroiditis can either be hypothyroid or have normal thyroid function, which is not the case in our patient.

CASE CONTINUED

An endocrinologist, consulted for a second opinion, agreed that the patient’s thyroid function test results were most likely due to his nonthyroidal illness and glucocorticoid therapy.

3. In view of the endocrinologist’s opinion, which should be the next step in the management of the patient’s thyroid condition?

  • Start levothyroxine (T4) therapy
  • Start liothyronine (T3) therapy
  • Start N-acetylcysteine therapy
  • Start thyrotropin-releasing hormone therapy
  • Remeasure thyroid hormones after full recovery from his acute illness

It is not clear whether the changes in thyroid hormone levels during an acute illness are a pathologic alteration for which thyroid hormone therapy may be beneficial, or a physiologic adaptation for which such therapy would not be indicated.41

However, current data argue against thyroid hormone therapy using T4 or T3 for patients with nonthyroidal illness syndrome (also called euthyroid sick syndrome).42 Indeed, several randomized controlled trials showed that thyroid hormone therapy is not beneficial in such patients and may be detrimental.41,43

Therapies other than thyroid hormone have been investigated to ameliorate thyroid hormone abnormalities in patients with nonthyroidal illness. These include N-acetylcysteine, thyrotropin-releasing hormone therapy, and nutritional support.

Some studies showed that giving N-acetyl­cysteine, an antioxidant, increased serum T3 and decreased serum reverse T3 concentrations in patients with acute myocardial infarction.44 Nevertheless, the mortality rate and length of hospitalization were not affected. Further studies are needed to know whether N-acetylcysteine therapy is beneficial for such patients.

Similarly, a study using a thyrotropin-releasing hormone analogue along with growth hormone-releasing peptide 2 showed an increase in serum TSH, T4, and T3 levels in critically ill patients.45 The benefit of this therapy has yet to be determined. On the other hand, early nutritional support was reported to prevent thyroid hormonal changes in patients postoperatively.46

Measuring thyroid hormone levels after full recovery is the most appropriate next step in our patient, as the changes in thyroid hormone concentrations subside as the acute illness resolves.47

 

 

CASE CONTINUED

The patient continued to improve. On hospital day 6, he was feeling better but still had mild respiratory distress. There had been no further episodes of arrhythmia since day 4. His blood pressure was 136/86 mm Hg, heart rate 88 beats per minute and regular, respiratory rate 18 breaths per minute, and oral temperature 37.1°C. His oxygen saturation was 92% on room air.

Before discharge, he was encouraged to quit smoking. He was offered behavioral counseling and medication therapy, but he only said that he would think about it. He was discharged on oral cefixime for 4 more days and was instructed to switch to a long-acting bronchodilator along with his other home medications and to return in 1 week to have his thyroid hormones checked.

One week later, his laboratory results were:

  • TSH 11.2 mU/L (reference range 0.5–5.0)
  • Free T4 1.2 ng/dL (0.9–2.4)
  • Total T3 92 ng/dL (80–180).

Clinically, the patient was euthyroid, and examination of his thyroid was unremarkable.

4. Based on these last test results, which statement is correct?

  • Levothyroxine therapy should be started
  • His serum TSH elevation is most likely transient
  • Thyroid ultrasonography is strongly indicated
  • A radioactive iodine uptake study should be performed
  • Measurement of thyroid-stimulating immunoglobulins is indicated

During recovery from nonthyroidal illness, some patients may have elevated serum TSH levels that are usually transient and modest (< 20 mU/L).48 Normalization of the thyroid function tests including serum TSH may take several weeks49 or months.50 However, a systematic review found that the likelihood of permanent primary hypothyroidism is high in patients with serum TSH levels higher than 20 mU/L during the recovery phase of their nonthyroidal illness.51

Ultrasonography is useful for evaluating patients with thyroid nodules or goiter but is of little benefit for patients like ours, in whom the thyroid is normal on examination.

Similarly, a radioactive iodine uptake study is not indicated, as it is principally used to help differentiate between types of thyrotoxicosis. (Radioactive iodine is also used to treat differentiated thyroid cancer.)

Thyroid-stimulating immunoglobins are TSH receptor-stimulating antibodies that cause Graves disease. Nevertheless, measuring them is not routinely indicated for its diagnosis. However, their measurement is of significant help in the diagnosis of Graves disease if a radioactive iodine uptake study cannot be performed (as in pregnancy) and in atypical presentations such as euthyroid Graves ophthalmopathy.52 Other indications for thyroid-stimulating immunoglobin measurement are beyond the scope of the article. Our patient’s test results are not consistent with hyperthyroidism, so measuring thyroid-stimulating immunoglobins is not indicated.

CASE CONCLUSION: BETTER, BUT STILL SMOKING

The patient missed his 1-month clinic follow-up, but he visited the clinic for follow-up 3 months later. He was feeling well with no complaints. Test results including serum TSH, free T4, and total T3 were within normal ranges. His COPD was under control, with an FEV1 88% of predicted.

He was again encouraged to quit smoking and was offered drug therapy and behavioral counseling, but he declined. In addition, he was instructed to adhere to his annual influenza vaccination.

KEY POINTS

  • In patients with acute illness, it is recommended that thyroid function not be assessed unless there is a strong indication.
  • If thyroid function assessment is indicated for critically ill patients, serum TSH and free T4 concentrations should be measured. Some clinicians also measure serum total T3 level.
  • Thyroid function testing in critically ill patients usually shows low serum total T3, normal or low serum TSH, and normal or low serum free T4.
  • Many drugs can alter thyroid hormone levels.
  • Thyroid hormone therapy is not recommended for critically ill patients with low T3, low T4, or both.
  • During recovery from nonthyroidal illness, some patients may have mild elevation in serum TSH levels (< 20 mU/L).
  • Thyroid hormone levels may take several weeks or months to return to normal after the acute illness.
  • Patients with serum TSH levels higher than 20 mU/L during the recovery phase of their nonthyroidal illness are more likely to have permanent primary hypothyroidism.
References
  1. Lamb EJ, Martin J. Thyroid function tests: often justified in the acutely ill. Ann Clin Biochem 2000; 37(pt 2):158–164. doi:10.1258/0004563001899159
  2. Spencer CA, LoPresti JS, Patel A, et al. Applications of a new chemiluminometric thyrotropin assay to subnormal measurement. J Clin Endocrinol Metab 1990; 70(2):453–460. doi:10.1210/jcem-70-2-453
  3. Ross DS, Ardisson LJ, Meskell MJ. Measurement of thyrotropin in clinical and subclinical hyperthyroidism using a new chemiluminescent assay. J Clin Endocrinol Metab 1989; 69(3):684–688. doi:10.1210/jcem-69-3-684
  4. Koulouri O, Moran C, Halsall D, Chatterjee K, Gurnell M. Pitfalls in the measurement and interpretation of thyroid function tests. Best Pract Res Clin Endocrinol Metab 2013; 27(6):745–762. doi:10.1016/j.beem.2013.10.003
  5. Lechan RM, Fekete C. Role of thyroid hormone deiodination in the hypothalamus. Thyroid 2005; 15(8):883–897. doi:10.1089/thy.2005.15.883
  6. Chopra IJ, Hershman JM, Pardridge WM, Nicoloff JT. Thyroid function in nonthyroidal ilnesses. Ann Intern Med 1983; 98(6):946–957. doi:10.7326/0003-4819-98-6-946
  7. Chopra IJ, Solomon DH, Hepner HW, Mortenstein AA. Misleadingly low free thyroxine index and usefulness of reverse triiodothyronine measurement in nonthyroidal illnesses. Ann Intern Med 1979; 90(6):905–912. doi:10.7326/0003-4819-90-6-905
  8. Pontecorvi A, Robbins J. The plasma membrane and thyroid hormone entry into cells. Trends Endocrinol Metab 1989; 1(2):90–94. pmid:18411097
  9. Hennemann G, Krenning EP. Pitfalls in the interpretation of thyroid function tests in old age and non-thyroidal illness. Horm Res 1987; 26(1–4):100–104. doi:10.1159/000180688
  10. Baloch Z, Carayon P, Conte-Devolx B, et al; Guidelines Committee, National Academy of Clinical Biochemistry. Laboratory medicine practice guidelines. Laboratory support for the diagnosis and monitoring of thyroid disease. Thyroid 2003; 13(1):3–126. doi:10.1089/105072503321086962
  11. Lum S, Nicoloff JT, Spencer CA, Kaptein EM. Peripheral tissue mechanism for maintenance of serum triiodothyronine values in a thyroxine-deficient state in man. J Clin Invest 1984; 73(2):570–575. doi:10.1172/JCI111245
  12. Ortiga-Carvalho TM, Chiamolera MI, Pazos-Moura CC, Wondisford FE. Hypothalamus-pituitary-thyroid axis. Compr Physiol 2016; 6(3):1387–1428. doi:10.1002/cphy.c150027
  13. de Vries EM, Fliers E, Boelen A. The molecular basis of the non-thyroidal illness syndrome. J Endocrinol 2015; 225(3):R67–R81. doi:10.1530/JOE-15-0133
  14. Chopra IJ, Huang TS, Beredo A, Solomon DH, Teco GN, Mean JF. Evidence for an inhibitor of extrathyroidal conversion of thyroxine to 3, 5, 3'-triiodothyronine in sera of patients with nonthyroidal illnesses. J Clin Endocrinol Metab 1985; 60(4):666–672. doi:10.1210/jcem-60-4-666
  15. Peeters RP, Debaveye Y, Fliers E, Visser TJ. Changes within the thyroid axis during critical illness. Crit Care Clin 2006; 22(1):41–55. doi:10.1016/j.ccc.2005.08.006
  16. Spencer C, Eigen A, Shen D, et al. Specificity of sensitive assays of thyrotropin (TSH) used to screen for thyroid disease in hospitalized patients. Clin Chem 1987; 33(8):1391–1396. pmid:3301067
  17. Adler SM, Wartofsky L. The nonthyroidal illness syndrome. Endocrinol Metab Clin North Am 2007; 36(3):657–672. doi:10.1016/j.ecl.2007.04.007
  18. Persani L. Central hypothyroidism: pathogenic, diagnostic, and therapeutic challenges. J Clin Endocrinol Metab 2012; 97(9):3068–3078. doi:10.1210/jc.2012-1616
  19. Kidess AI, Caplan RH, Reynertson RH, Wickus GG, Goodnough DE. Transient corticotropin deficiency in critical illness. Mayo Clin Proc 1993; 68(5):435–441. doi:10.1016/s0025-6196(12)60188-8
  20. Lamberts SW, Bruining HA, De Jong FH. Corticosteroid therapy in severe illness. N Engl J Med 1997; 337(18):1285–1292. doi:10.1056/NEJM199710303371807
  21. Burmeister LA. Reverse T3 does not reliably differentiate hypothyroid sick syndrome from euthyroid sick syndrome. Thyroid 1995; 5(6):435–441. doi:10.1089/thy.1995.5.435
  22. Duick DS, Warren DW, Nicoloff JT, Otis CL, Croxson MS. Effect of single dose dexamethasone on the concentration of serum triiodothyronine in man. J Clin Endocrinol Metab 1974; 39(6):1151–1154. doi:10.1210/jcem-39-6-1151
  23. Gamstedt A, Järnerot G, Kågedal B. Dose related effects of betamethasone on iodothyronines and thyroid hormone-binding proteins in serum. Acta Endocrinol (Copenh) 1981; 96(4):484–490. doi:10.1530/acta.0.0960484
  24. Wartofsky L, Burman KD. Alterations in thyroid function in patients with systemic illness: the “euthyroid sick syndrome.” Endocr Rev 1982; 3(2):164–217. doi:10.1210/edrv-3-2-164
  25. Wilber JF, Utiger RD. The effect of glucocorticoids on thyrotropin secretion. J Clin Invest 1969; 48(11):2096–2103. doi:10.1172/JCI106176
  26. Nicoloff JT, Fisher DA, Appleman MD Jr. The role of glucocorticoids in the regulation of thyroid function in man. J Clin Invest 1970; 49(10):1922–1929. doi:10.1172/JCI106411
  27. Surks MI, Sievert R. Drugs and thyroid function. N Engl J Med 1995; 333(25):1688–1694. doi:10.1056/NEJM199512213332507
  28. Haugen BR. Drugs that suppress TSH or cause central hypothyroidism. Best Pract Res Clin Endocrinol Metab 2009; 23(6):793–800. doi:10.1016/j.beem.2009.08.003
  29. Sherman SI, Gopal J, Haugen BR, et al. Central hypothyroidism associated with retinoid X receptor–selective ligands. N Engl J Med 1999; 340(14):1075–1079. doi:10.1056/NEJM199904083401404
  30. Murchison LE, How J, Bewsher PD. Comparison of propranolol and metoprolol in the management of hyperthyroidism. Br J Clin Pharmacol 1979; 8(6):581–587. doi:10.1111/j.1365-2125.1979.tb01048.x
  31. Faber J, Friis T, Kirkegaard C, et al. Serum T4, T3 and reverse T3 during treatment with propranolol in hyperthyroidism, L-T4 treated myxedema and in normal man. Horm Metab Res 1979; 11(1):34–36. doi:10.1055/s-0028-1092678
  32. Kristensen BO, Weeke J. Propranolol-induced increments in total and free serum thyroxine in patients with essential hypertension. Clin Pharmacol Ther 1977; 22(6):864–867. doi:10.1002/cpt1977226864
  33. Murchison LE, Bewsher PD, Chesters MI, Ferrier WR. Comparison of propranolol and practolol in the management of hyperthyroidism. Br J Clin Pharmacol 1976; 3(2):273–277. doi:10.1111/j.1365-2125.1976.tb00603.x
  34. Lotti G, Delitala G, Devilla L, Alagna S, Masala A. Reduction of plasma triiodothyronine (T3) induced by propranolol. Clin Endocrinol 1977; 6(6):405–410. doi:10.1111/j.1365-2265.1977.tb03322.x
  35. Perrild H, Hansen JM, Skovsted L, Christensen LK. Different effects of propranolol, alprenolol, sotalol, atenolol and metoprolol on serum T3 and serum rT3 in hyperthyroidism. Clin Endocrinol (Oxf) 1983; 18(2):139–142. pmid:6133659
  36. Reeves RA, From GL, Paul W, Leenen FH. Nadolol, propranolol, and thyroid hormones: evidence for a membrane-stabilizing action of propranolol. Clin Pharmacol Ther 1985; 37(2):157–161. doi:10.1038/clpt.1985.28
  37. Walker N, Jung RT, Jennings G, James WP. The effect of a beta-receptor agonist (salbutamol) on peripheral thyroid metabolism in euthyroid subjects. Horm Metab Res 1981; 13(10):590–591. doi:10.1055/s-2007-1019346
  38. Melmed S, Geola FL, Reed AW, Pekary AE, Park J, Hershman JM. A comparison of methods for assessing thyroid function in nonthyroidal illness. J Clin Endocrinol Metab 1982; 54(2):300–306. doi:10.1210/jcem-54-2-300
  39. Docter R, Krenning E, De Jong M, Hennemann G. The sick euthyroid syndrome: changes in thyroid hormone serum parameters and hormone metabolism. Clin Endocrinol (Oxf) 1993; 39(5):499–518. pmid:8252737
  40. Mariotti S, Caturegli P, Piccolo P, Barbesino G, Pinchera A. Antithyroid peroxidase autoantibodies in thyroid diseases. J Clin Endocrinol Metab 1990; 71(3):661–669. doi:10.1210/jcem-71-3-661
  41. De Groot LJ. Non-thyroidal illness syndrome is a manifestation of hypothalamic-pituitary dysfunction, and in view of current evidence, should be treated with appropriate replacement therapies. Crit Care Clin 2006; 22(1):57–86. doi:10.1016/j.ccc.2005.10.001
  42. Jonklaas J, Bianco AC, Bauer AJ, et al; American Thyroid Association Task Force on Thyroid Hormone Replacement. Guidelines for the treatment of hypothyroidism: prepared by the American Thyroid Association Task Force on Thyroid Hormone Replacement. Thyroid 2014; 24(12):1670–1751. doi:10.1089/thy.2014.0028
  43. Kaptein EM, Beale E, Chan LS. Thyroid hormone therapy for obesity and nonthyroidal illnesses: a systematic review. J Clin Endocrinol Metab 2009; 94(10):3663–3675. doi:10.1210/jc.2009-0899
  44. Vidart J, Wajner SM, Leite RS, et al. N-acetylcysteine administration prevents nonthyroidal illness syndrome in patients with acute myocardial infarction: a randomized clinical trial. J Clin Endocrinol Metab 2014; 99(12):4537–4545. doi:10.1210/jc.2014-2192
  45. Van den Berghe G, Wouters P, Weekers F, et al. Reactivation of pituitary hormone release and metabolic improvement by infusion of growth hormone-releasing peptide and thyrotropin-releasing hormone in patients with protracted critical illness. J Clin Endocrinol Metab 1999; 84(4):1311–1323. doi:10.1210/jcem.84.4.5636
  46. Langouche L, Vander Perre S, Marques M, et al. Impact of early nutrient restriction during critical illness on the nonthyroidal illness syndrome and its relation with outcome: a randomized, controlled clinical study. J Clin Endocrinol Metab 2013; 98(3):1006–1013. doi:10.1210/jc.2012-2809
  47. Economidou F, Douka E, Tzanela M, Nanas S, Kotanidou A. Thyroid function during critical illness. Hormones (Athens) 2011; 10(2):117–124. doi:10.14310/horm.2002.1301
  48. Hamblin PS, Dyer SA, Mohr VS, et al. Relationship between thyrotropin and thyroxine changes during recovery from severe hypothyroxinemia of critical illness. J Clin Endocrinol Metab 1986; 62(4):717–722. doi:10.1210/jcem-62-4-717
  49. Iglesias P, Diez JJ. Thyroid dysfunction and kidney disease. Eur J Endocrinol 2009; 160(4):503–515. doi:10.1530/EJE-08-0837
  50. Spencer CA. Clinical utility and cost-effectiveness of sensitive thyrotropin assays in ambulatory and hospitalized patients. Mayo Clin Proc 1988; 63(12):1214–1222. doi:10.1016/s0025-6196(12)65408-1
  51. Attia J, Margetts P, Guyatt G. Diagnosis of thyroid disease in hospitalized patients: a systematic review. Arch Intern Med 1999; 159(7):658–665. pmid:10218744
  52. Barbesino G, Tomer Y. Clinical review: clinical utility of TSH receptor antibodies. J Clin Endocrinol Metab 2013; 98(6):2247–2255. doi:10.1210/jc.2012-4309
Article PDF
Author and Disclosure Information

Yazan N. Alhalaseh, MD
Department of Internal Medicine, King Hussein Cancer Center, Amman, Jordan

Zaid A. Abdulelah, MD
Istishari Hospital, Amman, Jordan

Ahmad O. Armouti, MD
King Hussein Medical Center, Amman, Jordan

Ayman A. Zayed, MD, MSc, FACE, FACP
Professor of Medicine and Chief, Division of Endocrinology, Diabetes, and Metabolism, Department of Internal Medicine, Jordan University Hospital, Amman, Jordan

Address: Ayman A. Zayed, MD, MSc, FACE, FACP, Department of Internal Medicine, Jordan University Hospital, Queen Rania Street, Amman, Jordan, 11942; baraaayman@gmail.com

Issue
Cleveland Clinic Journal of Medicine - 86(10)
Publications
Topics
Page Number
666-675
Legacy Keywords
thyroid, thyroid function tests, hypothyroid, chronic obstructive pulmonary disease, COPD, thyroid-stimulating hormone, TSH, thyroxine, T4, triiodothyronine, T3, reverse T3, deiodinase, D1, euthyroid sick syndrome, nonthyroidal illness syndrome, Yazan Alhalaseh, Zaid Abdulelah, Ahmad Armouti, Ayman Zayed
Sections
Author and Disclosure Information

Yazan N. Alhalaseh, MD
Department of Internal Medicine, King Hussein Cancer Center, Amman, Jordan

Zaid A. Abdulelah, MD
Istishari Hospital, Amman, Jordan

Ahmad O. Armouti, MD
King Hussein Medical Center, Amman, Jordan

Ayman A. Zayed, MD, MSc, FACE, FACP
Professor of Medicine and Chief, Division of Endocrinology, Diabetes, and Metabolism, Department of Internal Medicine, Jordan University Hospital, Amman, Jordan

Address: Ayman A. Zayed, MD, MSc, FACE, FACP, Department of Internal Medicine, Jordan University Hospital, Queen Rania Street, Amman, Jordan, 11942; baraaayman@gmail.com

Author and Disclosure Information

Yazan N. Alhalaseh, MD
Department of Internal Medicine, King Hussein Cancer Center, Amman, Jordan

Zaid A. Abdulelah, MD
Istishari Hospital, Amman, Jordan

Ahmad O. Armouti, MD
King Hussein Medical Center, Amman, Jordan

Ayman A. Zayed, MD, MSc, FACE, FACP
Professor of Medicine and Chief, Division of Endocrinology, Diabetes, and Metabolism, Department of Internal Medicine, Jordan University Hospital, Amman, Jordan

Address: Ayman A. Zayed, MD, MSc, FACE, FACP, Department of Internal Medicine, Jordan University Hospital, Queen Rania Street, Amman, Jordan, 11942; baraaayman@gmail.com

Article PDF
Article PDF
Related Articles

A 66-year-old man presented to the emergency department with increasing shortness of breath and productive cough, which had begun 5 days earlier. Three years previously, he had been diagnosed with chronic obstructive pulmonary disease (COPD).

One week before the current presentation, he developed a sore throat, rhinorrhea, and nasal congestion, and the shortness of breath had started 2 days after that. Although he could speak in sentences, he was breathless even at rest. His dyspnea was associated with noisy breathing and cough productive of yellowish sputum; there was no hemoptysis. He reported fever, but he had no chills, night sweats, chest pain, or paroxysmal nocturnal dyspnea. The review of other systems was unremarkable.

His COPD was known to be mild, in Global Initiative for Chronic Obstructive Lung Disease (GOLD) grade 1, group A. His postbronchodilator ratio of forced expiratory volume in 1 second (FEV1) to forced vital capacity (FVC) was less than 0.70, and his FEV1 was 84% of predicted. Apart from mild intermittent cough with white sputum, his COPD had been under good control with inhaled ipratropium 4 times daily and inhaled albuterol as needed. He said he did not have shortness of breath except when hurrying on level ground or walking up a slight hill (Modified Medical Research Council dyspnea scale grade 1; COPD Assessment Test score < 10). In the last 3 years, he had 2 exacerbations of COPD, 1 year apart, both requiring oral prednisone and antibiotic therapy.

Other relevant history included hypertension and dyslipidemia of 15-year duration, for which he was taking candesartan 16 mg twice daily and atorvastatin 20 mg daily. He was compliant with his medications.

Though he usually received an influenza vaccine every year, he did not get it the previous year. Also, 3 years previously, he received the 23-valent pneumococcal polysaccharide vaccine (PPSV23), and the year before that he received the pneumococcal conjugate vaccine (PCV13). In addition, he was immunized against herpes zoster and tetanus.

The patient had smoked 1 pack per day for the past 38 years. His primary care physician had advised him many times to quit smoking. He had enrolled in a smoking cessation program 2 years previously, in which he received varenicline in addition to behavioral counseling in the form of motivational interviewing and a telephone quit-line. Nevertheless, he continued to smoke.

He was a retired engineer. He did not drink alcohol or use illicit drugs.

PHYSICAL EXAMINATION

On physical examination, the patient was sitting up in bed, leaning forward. He was alert and oriented but was breathing rapidly and looked sick. He had no cyanosis, clubbing, pallor, or jaundice. His blood pressure was 145/90 mm Hg, heart rate 110 beats per minute and regular, respiratory rate 29 breaths per minute, and oral temperature 38.1°C (100.6°F). His oxygen saturation was 88% while breathing room air. His body mass index was 27.1 kg/m2.

His throat was mildly congested. His neck veins were flat, and there were no carotid bruits. His thyroid examination was normal, without goiter, nodules, or tenderness.

Intercostal retractions were noted around the anterolateral costal margins. He had no chest wall deformities. Chest expansion was reduced bilaterally. There was hyperresonance bilaterally. Expiratory wheezes were heard over both lungs, without crackles.

His heart had no murmurs or added sounds. There was no lower-limb edema or swelling. The rest of his physical examination was unremarkable.

alhalaseh_thyroidfunctiontests_t1.jpg
Chest radiography showed hyperinflation without infiltrates. Electrocardiography showed normal sinus rhythm, with a peaked P wave (P pulmonale) and evidence of right ventricular hypertrophy, but no ischemic changes.

Results of initial laboratory testing are shown in Table 1.

Assessment: A 66-year-old man with GOLD grade 1, group A COPD, presenting with a severe exacerbation, most likely due to viral bronchitis.

 

 

INITIAL MANAGEMENT

The patient was given oxygen 28% by Venturi mask, and his oxygen saturation went up to 90%. He was started on nebulized albuterol 2.5 mg with ipratropium bromide 500 µg every 4 hours, prednisone 40 mg orally daily for 5 days, and ceftriaxone 1 g intravenously every 24 hours. The first dose of each medication was given in the emergency department.

The patient was then admitted to a progressive care unit, where he was placed on noninvasive positive pressure ventilation, continuous cardiac monitoring, and pulse oximetry. He was started on enoxaparin 40 mg subcutaneously daily to prevent venous thromboembolism, and the oral medications he had been taking at home were continued. Because he was receiving a glucocorticoid, his blood glucose was monitored in the fasting state, 2 hours after each meal, and as needed.

Two hours after he started noninvasive positive pressure ventilation, his arterial blood gases were remeasured and showed the following results:

  • pH 7.35
  • Partial pressure of carbon dioxide (Paco2) 52 mm Hg
  • Bicarbonate 28 mmol/L
  • Partial pressure of oxygen (Pao2) 60 mm Hg
  • Oxygen saturation 90%.

HOSPITAL COURSE

On hospital day 3, his dyspnea had slightly improved. His respiratory rate was 26 to 28 breaths per minute. His oxygen saturation remained between 90% and 92%.

At 10:21 pm, his cardiac monitor showed an episode of focal atrial tachycardia at a rate of 129 beats per minute that lasted for 3 minutes and 21 seconds, terminating spontaneously. He denied any change in his clinical condition during the episode, with no chest pain, palpitation, or change in dyspnea. There was no change in his vital signs. He had another similar asymptomatic episode lasting 4 minutes and 9 seconds at 6:30 am of hospital day 4.

Because of these episodes, the attending physician ordered thyroid function tests.

THYROID FUNCTION TESTING

1. Which thyroid function test is most likely to be helpful in the assessment of this patient’s thyroid status?

  • Serum thyroid-stimulating hormone (TSH) alone
  • Serum TSH and total thyroxine (T4)
  • Serum TSH and total triiodothyronine (T3)
  • Serum TSH and free T4
  • Serum TSH and free T3

There are several tests to assess thyroid function: the serum TSH, total T4, free T4, total T3, and free T3 concentrations.1

In normal physiology, TSH from the pituitary stimulates the thyroid gland to produce and secrete T4 and T3, which in turn inhibit TSH secretion through negative feedback. A negative log-linear relation exists between serum free T4 and TSH levels.2 Thus, the serum free T4 level can remain within the normal reference range even if the TSH level is high or low. 

TSH assays can have different detection limits. A third-generation TSH assay with a detection limit of 0.01 mU/L is recommended for use in clinical practice.3

TSH testing alone. Given its superior sensitivity and specificity, serum TSH measurement is considered the best single test for assessing thyroid function in most cases.4 Nevertheless, measurement of the serum TSH level alone could be misleading in several situations, eg, hypothalamic or pituitary disorders, recent treatment of thyrotoxicosis, impaired sensitivity to thyroid hormone, and acute nonthyroidal illness.4

alhalaseh_thyroidfunctiontests_t2.jpg
Because our patient is acutely ill, measuring his serum TSH alone is not the most appropriate test of his thyroid function. Euthyroid patients who present with acute illness usually have different patterns of abnormal thyroid function test results, depending on the severity of their illness, its stage, the drugs they are receiving, and other factors. Thyroid function test abnormalities in those patients are shown in Table 2.5–7

Free vs total T4 and T3 levels

Serum total T4 includes a fraction that is bound, mainly to thyroxin-binding globulin, and a very small unbound (free) fraction. The same applies to T3. Only free thyroid hormones represent the “active” fraction available for interaction with their protein receptors in the nucleus.8 Patients with conditions that can affect the thyroid-binding protein concentrations usually have altered serum total T4 and T3 levels, whereas their free hormone concentrations remain normal. Accordingly, measurement of free hormone levels, especially free T4, is usually recommended.

Although equilibrium dialysis is the method most likely to provide an accurate serum free T4 measurement, it is not commonly used because of its limited availability and high cost. Thus, most commercial laboratories use “direct” free T4 measurement or, to a lesser degree, the free T4 index.9 However, none of the currently available free T4 tests actually measure free T4 directly; rather, they estimate it.10

Commercial laboratories can provide a direct free T3 estimate, but it may be less reliable than total T3. If serum T3 measurement is indicated, serum total T3 is usually measured. However, total T3 measurement is rarely indicated for patients with hypothyroidism because it usually remains within the normal reference range.11 Nevertheless, serum total T3 measurement could be useful in patients with T3 toxicosis and in those who are acutely ill.

Accordingly, in acutely ill hospitalized patients like ours, measuring serum TSH using a third-generation assay and free T4 is essential to assess thyroid function. Many clinicians also measure serum total T3.

 

 

CASE CONTINUED: LOW TSH, LOW-NORMAL FREE T4, LOW TOTAL T3

The attending physician ordered serum TSH, free T4, and total T3 measurements, which yielded the following:

  • TSH 0.1 mU/L (0.5–5.0)
  • Total T3 55 ng/dL (80–180)
  • Free T4 0.9 ng/dL (0.9–2.4).

2. Which best explains this patient’s abnormal thyroid test results?

  • His acute illness
  • Central hypothyroidism due to pituitary infarction
  • His albuterol therapy
  • Subclinical thyrotoxicosis
  • Hashimoto thyroiditis

Since euthyroid patients with an acute illness may have abnormal thyroid test results (Table 2),5–7 thyroid function testing is not recommended unless there is a strong indication for it, such as new-onset atrial fibrillation, atrial flutter, or focal atrial tachycardia.1 In such patients, it is important to know whether the test abnormalities represent true thyroid disorder or are the result of a nonthyroidal illness.

alhalaseh_thyroidfunctiontests_f1.jpg
%3Cp%3EFigure%201.%20Peripheral%20conversion%20of%20thyroxine%20(T%3Csub%3E4%3C%2Fsub%3E)%20to%20triiodothyronine%20(T%3Csub%3E3%3C%2Fsub%3E)%2C%20reverse%20T%3Csub%3E3%3C%2Fsub%3E%2C%20and%20diiodothyronine%20(T%3Csub%3E2%3C%2Fsub%3E)%20by%20deiodinase%20types%201%2C%202%2C%20and%203%20(D1%2C%20D2%2C%20D3)%20in%20healthy%20people%20and%20in%20patients%20with%20nonthyroidal%20illness.%3C%2Fp%3E
In healthy people, T4 is converted to T3 (the principal active hormone) by type 1 deiodinase (D1) mainly in the liver and kidneys, whereas this reaction is catalyzed by type 2 deiodinase (D2) in the hypothalamus and pituitary. Type 3 deiodinase (D3) converts T4 to reverse T3, a biologically inactive molecule.12 D1 also mediates conversion of reverse T3 to diiodothyronine (T2) (Figure 1).

alhalaseh_thyroidfunctiontests_t3.jpg
Several conditions and drugs can decrease D1 activity, resulting in low serum T3 concentrations (Table 3). In patients with nonthyroidal illness, decreased D1 activity can be observed as early as the first 24 hours after the onset of the illness and is attributed to increased inflammatory cytokines, free fatty acids, increased endogenous cortisol secretion, and use of certain drugs.13,14 In addition, the reduced D1 activity can decrease the conversion of reverse T3 to T2, resulting in elevated serum reverse T3. Increased D3 activity during an acute illness is another mechanism for elevated serum reverse T3 concentration.15

Thyroid function testing in patients with nonthyroidal illness usually shows low serum total T3, normal or low serum TSH, and normal, low, or high serum free T4. However, transient mild serum TSH elevation can be seen in some patients during the recovery period.16 These abnormalities with their mechanisms are shown in Table 2.5–7 In several commercial kits, serum direct free T4 can be falsely decreased or increased.8

THE DIFFERENTIAL DIAGNOSIS

Our patient had low serum TSH, low-normal serum direct free T4, and low serum total T3. This profile could be caused by a nonthyroidal illness, “true” central hypothyroidism, or his glucocorticoid treatment. The reason we use the term “true” in this setting is that some experts suggest that the thyroid function test abnormalities in patients with acute nonthyroidal illness represent a transient central hypothyroidism.17 The clinical presentation is key in differentiating true central hypothyroidism from nonthyroidal illness.

In addition, measuring serum cortisol may help to differentiate between the 2 states, as it would be elevated in patients with nonthyroidal illness as part of a stress response but low in patients with true central hypothyroidism, since it is usually part of combined pituitary hormone deficiency.18 Of note, some critically ill patients have low serum cortisol because of transient central adrenal insufficiency.19,20

The serum concentration of reverse T3 has been suggested as a way to differentiate between hypothyroidism (low) and nonthyroidal illness (high); however, further studies showed that it does not reliably differentiate between the conditions.21

GLUCOCORTICOIDS AND THYROID FUNCTION TESTS

By inhibiting D1, glucocorticoids can decrease peripheral conversion of T4 to T3 and thus decrease serum total T3. This effect depends on the type and dose of the glucocorticoid and the duration of therapy.

In one study,22 there was a significant reduction in serum total T3 concentration 24 hours after a single oral dose of dexamethasone 12 mg in normal participants. This effect lasted 48 hours, after which serum total T3 returned to its pretreatment level.

In another study,23 a daily oral dose of betamethasone 1.5 mg for 5 days did not significantly reduce the serum total T3 in healthy volunteers, but a daily dose of 3 mg did. This effect was more pronounced at a daily dose of 4.5 mg, whereas a dose of 6.0 mg had no further effect.

Long-term glucocorticoid therapy also decreases serum total T4 and total T3 by lowering serum thyroid-binding globulin.24

Finally, glucocorticoids can decrease TSH secretion by directly inhibiting thyrotropin-releasing hormone.25,26 However, chronic hypercortisolism, whether endogenous or exogenous, does not cause clinically central hypothyroidism, possibly because of the negative feedback mechanism of low thyroid hormones on the pituitary and the hypothalamus.27

Other drugs including dopamine, dopamine agonists, dobutamine, and somatostatin analogues can suppress serum TSH. As with glucocorticoids, these drugs do not cause clinically evident central hypothyroidism.28 Bexarotene, a retinoid X receptor ligand used in the treatment of cutaneous T-cell lymphoma, has been reported to cause clinically evident central hypothyroidism by suppressing TSH and increasing T4 clearance.29

 

 

BETA-BLOCKERS, BETA-AGONISTS AND THYROID FUNCTION

While there is general agreement that beta-adrenergic antagonists (beta-blockers) do not affect the serum TSH concentration, conflicting data have been reported concerning their effect on other thyroid function tests. This may be due to several factors, including dose, duration of therapy, the patient’s thyroid status, and differences in laboratory methodology.30

In studies of propranolol, serum total T4 concentrations did not change or were increased with daily doses of 160 mg or more in both euthyroid participants and hyperthyroid patients31–33; serum total T3 concentrations did not change or were decreased with 40 mg or more daily34; and serum reverse T3 concentrations were increased with daily doses of 80 mg or more.31 It is most likely that propranolol exerts these changes by inhibiting D1 activity in peripheral tissues.

Furthermore, a significant decrease in serum total T3 concentrations was observed in hyperthyroid patients treated with atenolol 100 mg daily, metoprolol 100 mg daily, and alprenolol 100 mg daily, but not with sotalol 80 mg daily or nadolol (up to 240 mg daily).35,36

On the other hand, beta-adrenergic agonists have not been reported to cause significant changes in thyroid function tests.37

SUBCLINICAL THYROTOXICOSIS OR HASHIMOTO THYROIDITIS?

Our patient’s thyroid function test results are more likely due to his nonthyroidal illness and glucocorticoid therapy, as there is no clinical evidence to point to a hypothalamic-pituitary disorder accounting for true central hypothyroidism.

The other options mentioned in question 2 are unlikely to explain our patient’s thyroid function test results.

Subclinical thyrotoxicosis is characterized by suppressed serum TSH, but both serum free T4 and total T3 remain within the normal reference ranges. In addition, the serum TSH level may help to differentiate between thyrotoxicosis and nonthyroidal illness. In the former, serum TSH is usually suppressed (< 0.01 mU/L), whereas in the latter it is usually low but detectable (0.05– 0.3 mU/L).38,39

Hashimoto thyroiditis is a chronic autoimmune thyroid disease characterized by diffuse lymphocytic infiltration of the thyroid gland. Almost all patients with Hashimoto thyroiditis have elevated levels of antibodies to thyroid peroxidase or thyroglobulin.40 Clinically, patients with Hashimoto thyroiditis can either be hypothyroid or have normal thyroid function, which is not the case in our patient.

CASE CONTINUED

An endocrinologist, consulted for a second opinion, agreed that the patient’s thyroid function test results were most likely due to his nonthyroidal illness and glucocorticoid therapy.

3. In view of the endocrinologist’s opinion, which should be the next step in the management of the patient’s thyroid condition?

  • Start levothyroxine (T4) therapy
  • Start liothyronine (T3) therapy
  • Start N-acetylcysteine therapy
  • Start thyrotropin-releasing hormone therapy
  • Remeasure thyroid hormones after full recovery from his acute illness

It is not clear whether the changes in thyroid hormone levels during an acute illness are a pathologic alteration for which thyroid hormone therapy may be beneficial, or a physiologic adaptation for which such therapy would not be indicated.41

However, current data argue against thyroid hormone therapy using T4 or T3 for patients with nonthyroidal illness syndrome (also called euthyroid sick syndrome).42 Indeed, several randomized controlled trials showed that thyroid hormone therapy is not beneficial in such patients and may be detrimental.41,43

Therapies other than thyroid hormone have been investigated to ameliorate thyroid hormone abnormalities in patients with nonthyroidal illness. These include N-acetylcysteine, thyrotropin-releasing hormone therapy, and nutritional support.

Some studies showed that giving N-acetyl­cysteine, an antioxidant, increased serum T3 and decreased serum reverse T3 concentrations in patients with acute myocardial infarction.44 Nevertheless, the mortality rate and length of hospitalization were not affected. Further studies are needed to know whether N-acetylcysteine therapy is beneficial for such patients.

Similarly, a study using a thyrotropin-releasing hormone analogue along with growth hormone-releasing peptide 2 showed an increase in serum TSH, T4, and T3 levels in critically ill patients.45 The benefit of this therapy has yet to be determined. On the other hand, early nutritional support was reported to prevent thyroid hormonal changes in patients postoperatively.46

Measuring thyroid hormone levels after full recovery is the most appropriate next step in our patient, as the changes in thyroid hormone concentrations subside as the acute illness resolves.47

 

 

CASE CONTINUED

The patient continued to improve. On hospital day 6, he was feeling better but still had mild respiratory distress. There had been no further episodes of arrhythmia since day 4. His blood pressure was 136/86 mm Hg, heart rate 88 beats per minute and regular, respiratory rate 18 breaths per minute, and oral temperature 37.1°C. His oxygen saturation was 92% on room air.

Before discharge, he was encouraged to quit smoking. He was offered behavioral counseling and medication therapy, but he only said that he would think about it. He was discharged on oral cefixime for 4 more days and was instructed to switch to a long-acting bronchodilator along with his other home medications and to return in 1 week to have his thyroid hormones checked.

One week later, his laboratory results were:

  • TSH 11.2 mU/L (reference range 0.5–5.0)
  • Free T4 1.2 ng/dL (0.9–2.4)
  • Total T3 92 ng/dL (80–180).

Clinically, the patient was euthyroid, and examination of his thyroid was unremarkable.

4. Based on these last test results, which statement is correct?

  • Levothyroxine therapy should be started
  • His serum TSH elevation is most likely transient
  • Thyroid ultrasonography is strongly indicated
  • A radioactive iodine uptake study should be performed
  • Measurement of thyroid-stimulating immunoglobulins is indicated

During recovery from nonthyroidal illness, some patients may have elevated serum TSH levels that are usually transient and modest (< 20 mU/L).48 Normalization of the thyroid function tests including serum TSH may take several weeks49 or months.50 However, a systematic review found that the likelihood of permanent primary hypothyroidism is high in patients with serum TSH levels higher than 20 mU/L during the recovery phase of their nonthyroidal illness.51

Ultrasonography is useful for evaluating patients with thyroid nodules or goiter but is of little benefit for patients like ours, in whom the thyroid is normal on examination.

Similarly, a radioactive iodine uptake study is not indicated, as it is principally used to help differentiate between types of thyrotoxicosis. (Radioactive iodine is also used to treat differentiated thyroid cancer.)

Thyroid-stimulating immunoglobins are TSH receptor-stimulating antibodies that cause Graves disease. Nevertheless, measuring them is not routinely indicated for its diagnosis. However, their measurement is of significant help in the diagnosis of Graves disease if a radioactive iodine uptake study cannot be performed (as in pregnancy) and in atypical presentations such as euthyroid Graves ophthalmopathy.52 Other indications for thyroid-stimulating immunoglobin measurement are beyond the scope of the article. Our patient’s test results are not consistent with hyperthyroidism, so measuring thyroid-stimulating immunoglobins is not indicated.

CASE CONCLUSION: BETTER, BUT STILL SMOKING

The patient missed his 1-month clinic follow-up, but he visited the clinic for follow-up 3 months later. He was feeling well with no complaints. Test results including serum TSH, free T4, and total T3 were within normal ranges. His COPD was under control, with an FEV1 88% of predicted.

He was again encouraged to quit smoking and was offered drug therapy and behavioral counseling, but he declined. In addition, he was instructed to adhere to his annual influenza vaccination.

KEY POINTS

  • In patients with acute illness, it is recommended that thyroid function not be assessed unless there is a strong indication.
  • If thyroid function assessment is indicated for critically ill patients, serum TSH and free T4 concentrations should be measured. Some clinicians also measure serum total T3 level.
  • Thyroid function testing in critically ill patients usually shows low serum total T3, normal or low serum TSH, and normal or low serum free T4.
  • Many drugs can alter thyroid hormone levels.
  • Thyroid hormone therapy is not recommended for critically ill patients with low T3, low T4, or both.
  • During recovery from nonthyroidal illness, some patients may have mild elevation in serum TSH levels (< 20 mU/L).
  • Thyroid hormone levels may take several weeks or months to return to normal after the acute illness.
  • Patients with serum TSH levels higher than 20 mU/L during the recovery phase of their nonthyroidal illness are more likely to have permanent primary hypothyroidism.

A 66-year-old man presented to the emergency department with increasing shortness of breath and productive cough, which had begun 5 days earlier. Three years previously, he had been diagnosed with chronic obstructive pulmonary disease (COPD).

One week before the current presentation, he developed a sore throat, rhinorrhea, and nasal congestion, and the shortness of breath had started 2 days after that. Although he could speak in sentences, he was breathless even at rest. His dyspnea was associated with noisy breathing and cough productive of yellowish sputum; there was no hemoptysis. He reported fever, but he had no chills, night sweats, chest pain, or paroxysmal nocturnal dyspnea. The review of other systems was unremarkable.

His COPD was known to be mild, in Global Initiative for Chronic Obstructive Lung Disease (GOLD) grade 1, group A. His postbronchodilator ratio of forced expiratory volume in 1 second (FEV1) to forced vital capacity (FVC) was less than 0.70, and his FEV1 was 84% of predicted. Apart from mild intermittent cough with white sputum, his COPD had been under good control with inhaled ipratropium 4 times daily and inhaled albuterol as needed. He said he did not have shortness of breath except when hurrying on level ground or walking up a slight hill (Modified Medical Research Council dyspnea scale grade 1; COPD Assessment Test score < 10). In the last 3 years, he had 2 exacerbations of COPD, 1 year apart, both requiring oral prednisone and antibiotic therapy.

Other relevant history included hypertension and dyslipidemia of 15-year duration, for which he was taking candesartan 16 mg twice daily and atorvastatin 20 mg daily. He was compliant with his medications.

Though he usually received an influenza vaccine every year, he did not get it the previous year. Also, 3 years previously, he received the 23-valent pneumococcal polysaccharide vaccine (PPSV23), and the year before that he received the pneumococcal conjugate vaccine (PCV13). In addition, he was immunized against herpes zoster and tetanus.

The patient had smoked 1 pack per day for the past 38 years. His primary care physician had advised him many times to quit smoking. He had enrolled in a smoking cessation program 2 years previously, in which he received varenicline in addition to behavioral counseling in the form of motivational interviewing and a telephone quit-line. Nevertheless, he continued to smoke.

He was a retired engineer. He did not drink alcohol or use illicit drugs.

PHYSICAL EXAMINATION

On physical examination, the patient was sitting up in bed, leaning forward. He was alert and oriented but was breathing rapidly and looked sick. He had no cyanosis, clubbing, pallor, or jaundice. His blood pressure was 145/90 mm Hg, heart rate 110 beats per minute and regular, respiratory rate 29 breaths per minute, and oral temperature 38.1°C (100.6°F). His oxygen saturation was 88% while breathing room air. His body mass index was 27.1 kg/m2.

His throat was mildly congested. His neck veins were flat, and there were no carotid bruits. His thyroid examination was normal, without goiter, nodules, or tenderness.

Intercostal retractions were noted around the anterolateral costal margins. He had no chest wall deformities. Chest expansion was reduced bilaterally. There was hyperresonance bilaterally. Expiratory wheezes were heard over both lungs, without crackles.

His heart had no murmurs or added sounds. There was no lower-limb edema or swelling. The rest of his physical examination was unremarkable.

alhalaseh_thyroidfunctiontests_t1.jpg
Chest radiography showed hyperinflation without infiltrates. Electrocardiography showed normal sinus rhythm, with a peaked P wave (P pulmonale) and evidence of right ventricular hypertrophy, but no ischemic changes.

Results of initial laboratory testing are shown in Table 1.

Assessment: A 66-year-old man with GOLD grade 1, group A COPD, presenting with a severe exacerbation, most likely due to viral bronchitis.

 

 

INITIAL MANAGEMENT

The patient was given oxygen 28% by Venturi mask, and his oxygen saturation went up to 90%. He was started on nebulized albuterol 2.5 mg with ipratropium bromide 500 µg every 4 hours, prednisone 40 mg orally daily for 5 days, and ceftriaxone 1 g intravenously every 24 hours. The first dose of each medication was given in the emergency department.

The patient was then admitted to a progressive care unit, where he was placed on noninvasive positive pressure ventilation, continuous cardiac monitoring, and pulse oximetry. He was started on enoxaparin 40 mg subcutaneously daily to prevent venous thromboembolism, and the oral medications he had been taking at home were continued. Because he was receiving a glucocorticoid, his blood glucose was monitored in the fasting state, 2 hours after each meal, and as needed.

Two hours after he started noninvasive positive pressure ventilation, his arterial blood gases were remeasured and showed the following results:

  • pH 7.35
  • Partial pressure of carbon dioxide (Paco2) 52 mm Hg
  • Bicarbonate 28 mmol/L
  • Partial pressure of oxygen (Pao2) 60 mm Hg
  • Oxygen saturation 90%.

HOSPITAL COURSE

On hospital day 3, his dyspnea had slightly improved. His respiratory rate was 26 to 28 breaths per minute. His oxygen saturation remained between 90% and 92%.

At 10:21 pm, his cardiac monitor showed an episode of focal atrial tachycardia at a rate of 129 beats per minute that lasted for 3 minutes and 21 seconds, terminating spontaneously. He denied any change in his clinical condition during the episode, with no chest pain, palpitation, or change in dyspnea. There was no change in his vital signs. He had another similar asymptomatic episode lasting 4 minutes and 9 seconds at 6:30 am of hospital day 4.

Because of these episodes, the attending physician ordered thyroid function tests.

THYROID FUNCTION TESTING

1. Which thyroid function test is most likely to be helpful in the assessment of this patient’s thyroid status?

  • Serum thyroid-stimulating hormone (TSH) alone
  • Serum TSH and total thyroxine (T4)
  • Serum TSH and total triiodothyronine (T3)
  • Serum TSH and free T4
  • Serum TSH and free T3

There are several tests to assess thyroid function: the serum TSH, total T4, free T4, total T3, and free T3 concentrations.1

In normal physiology, TSH from the pituitary stimulates the thyroid gland to produce and secrete T4 and T3, which in turn inhibit TSH secretion through negative feedback. A negative log-linear relation exists between serum free T4 and TSH levels.2 Thus, the serum free T4 level can remain within the normal reference range even if the TSH level is high or low. 

TSH assays can have different detection limits. A third-generation TSH assay with a detection limit of 0.01 mU/L is recommended for use in clinical practice.3

TSH testing alone. Given its superior sensitivity and specificity, serum TSH measurement is considered the best single test for assessing thyroid function in most cases.4 Nevertheless, measurement of the serum TSH level alone could be misleading in several situations, eg, hypothalamic or pituitary disorders, recent treatment of thyrotoxicosis, impaired sensitivity to thyroid hormone, and acute nonthyroidal illness.4

alhalaseh_thyroidfunctiontests_t2.jpg
Because our patient is acutely ill, measuring his serum TSH alone is not the most appropriate test of his thyroid function. Euthyroid patients who present with acute illness usually have different patterns of abnormal thyroid function test results, depending on the severity of their illness, its stage, the drugs they are receiving, and other factors. Thyroid function test abnormalities in those patients are shown in Table 2.5–7

Free vs total T4 and T3 levels

Serum total T4 includes a fraction that is bound, mainly to thyroxin-binding globulin, and a very small unbound (free) fraction. The same applies to T3. Only free thyroid hormones represent the “active” fraction available for interaction with their protein receptors in the nucleus.8 Patients with conditions that can affect the thyroid-binding protein concentrations usually have altered serum total T4 and T3 levels, whereas their free hormone concentrations remain normal. Accordingly, measurement of free hormone levels, especially free T4, is usually recommended.

Although equilibrium dialysis is the method most likely to provide an accurate serum free T4 measurement, it is not commonly used because of its limited availability and high cost. Thus, most commercial laboratories use “direct” free T4 measurement or, to a lesser degree, the free T4 index.9 However, none of the currently available free T4 tests actually measure free T4 directly; rather, they estimate it.10

Commercial laboratories can provide a direct free T3 estimate, but it may be less reliable than total T3. If serum T3 measurement is indicated, serum total T3 is usually measured. However, total T3 measurement is rarely indicated for patients with hypothyroidism because it usually remains within the normal reference range.11 Nevertheless, serum total T3 measurement could be useful in patients with T3 toxicosis and in those who are acutely ill.

Accordingly, in acutely ill hospitalized patients like ours, measuring serum TSH using a third-generation assay and free T4 is essential to assess thyroid function. Many clinicians also measure serum total T3.

 

 

CASE CONTINUED: LOW TSH, LOW-NORMAL FREE T4, LOW TOTAL T3

The attending physician ordered serum TSH, free T4, and total T3 measurements, which yielded the following:

  • TSH 0.1 mU/L (0.5–5.0)
  • Total T3 55 ng/dL (80–180)
  • Free T4 0.9 ng/dL (0.9–2.4).

2. Which best explains this patient’s abnormal thyroid test results?

  • His acute illness
  • Central hypothyroidism due to pituitary infarction
  • His albuterol therapy
  • Subclinical thyrotoxicosis
  • Hashimoto thyroiditis

Since euthyroid patients with an acute illness may have abnormal thyroid test results (Table 2),5–7 thyroid function testing is not recommended unless there is a strong indication for it, such as new-onset atrial fibrillation, atrial flutter, or focal atrial tachycardia.1 In such patients, it is important to know whether the test abnormalities represent true thyroid disorder or are the result of a nonthyroidal illness.

alhalaseh_thyroidfunctiontests_f1.jpg
%3Cp%3EFigure%201.%20Peripheral%20conversion%20of%20thyroxine%20(T%3Csub%3E4%3C%2Fsub%3E)%20to%20triiodothyronine%20(T%3Csub%3E3%3C%2Fsub%3E)%2C%20reverse%20T%3Csub%3E3%3C%2Fsub%3E%2C%20and%20diiodothyronine%20(T%3Csub%3E2%3C%2Fsub%3E)%20by%20deiodinase%20types%201%2C%202%2C%20and%203%20(D1%2C%20D2%2C%20D3)%20in%20healthy%20people%20and%20in%20patients%20with%20nonthyroidal%20illness.%3C%2Fp%3E
In healthy people, T4 is converted to T3 (the principal active hormone) by type 1 deiodinase (D1) mainly in the liver and kidneys, whereas this reaction is catalyzed by type 2 deiodinase (D2) in the hypothalamus and pituitary. Type 3 deiodinase (D3) converts T4 to reverse T3, a biologically inactive molecule.12 D1 also mediates conversion of reverse T3 to diiodothyronine (T2) (Figure 1).

alhalaseh_thyroidfunctiontests_t3.jpg
Several conditions and drugs can decrease D1 activity, resulting in low serum T3 concentrations (Table 3). In patients with nonthyroidal illness, decreased D1 activity can be observed as early as the first 24 hours after the onset of the illness and is attributed to increased inflammatory cytokines, free fatty acids, increased endogenous cortisol secretion, and use of certain drugs.13,14 In addition, the reduced D1 activity can decrease the conversion of reverse T3 to T2, resulting in elevated serum reverse T3. Increased D3 activity during an acute illness is another mechanism for elevated serum reverse T3 concentration.15

Thyroid function testing in patients with nonthyroidal illness usually shows low serum total T3, normal or low serum TSH, and normal, low, or high serum free T4. However, transient mild serum TSH elevation can be seen in some patients during the recovery period.16 These abnormalities with their mechanisms are shown in Table 2.5–7 In several commercial kits, serum direct free T4 can be falsely decreased or increased.8

THE DIFFERENTIAL DIAGNOSIS

Our patient had low serum TSH, low-normal serum direct free T4, and low serum total T3. This profile could be caused by a nonthyroidal illness, “true” central hypothyroidism, or his glucocorticoid treatment. The reason we use the term “true” in this setting is that some experts suggest that the thyroid function test abnormalities in patients with acute nonthyroidal illness represent a transient central hypothyroidism.17 The clinical presentation is key in differentiating true central hypothyroidism from nonthyroidal illness.

In addition, measuring serum cortisol may help to differentiate between the 2 states, as it would be elevated in patients with nonthyroidal illness as part of a stress response but low in patients with true central hypothyroidism, since it is usually part of combined pituitary hormone deficiency.18 Of note, some critically ill patients have low serum cortisol because of transient central adrenal insufficiency.19,20

The serum concentration of reverse T3 has been suggested as a way to differentiate between hypothyroidism (low) and nonthyroidal illness (high); however, further studies showed that it does not reliably differentiate between the conditions.21

GLUCOCORTICOIDS AND THYROID FUNCTION TESTS

By inhibiting D1, glucocorticoids can decrease peripheral conversion of T4 to T3 and thus decrease serum total T3. This effect depends on the type and dose of the glucocorticoid and the duration of therapy.

In one study,22 there was a significant reduction in serum total T3 concentration 24 hours after a single oral dose of dexamethasone 12 mg in normal participants. This effect lasted 48 hours, after which serum total T3 returned to its pretreatment level.

In another study,23 a daily oral dose of betamethasone 1.5 mg for 5 days did not significantly reduce the serum total T3 in healthy volunteers, but a daily dose of 3 mg did. This effect was more pronounced at a daily dose of 4.5 mg, whereas a dose of 6.0 mg had no further effect.

Long-term glucocorticoid therapy also decreases serum total T4 and total T3 by lowering serum thyroid-binding globulin.24

Finally, glucocorticoids can decrease TSH secretion by directly inhibiting thyrotropin-releasing hormone.25,26 However, chronic hypercortisolism, whether endogenous or exogenous, does not cause clinically central hypothyroidism, possibly because of the negative feedback mechanism of low thyroid hormones on the pituitary and the hypothalamus.27

Other drugs including dopamine, dopamine agonists, dobutamine, and somatostatin analogues can suppress serum TSH. As with glucocorticoids, these drugs do not cause clinically evident central hypothyroidism.28 Bexarotene, a retinoid X receptor ligand used in the treatment of cutaneous T-cell lymphoma, has been reported to cause clinically evident central hypothyroidism by suppressing TSH and increasing T4 clearance.29

 

 

BETA-BLOCKERS, BETA-AGONISTS AND THYROID FUNCTION

While there is general agreement that beta-adrenergic antagonists (beta-blockers) do not affect the serum TSH concentration, conflicting data have been reported concerning their effect on other thyroid function tests. This may be due to several factors, including dose, duration of therapy, the patient’s thyroid status, and differences in laboratory methodology.30

In studies of propranolol, serum total T4 concentrations did not change or were increased with daily doses of 160 mg or more in both euthyroid participants and hyperthyroid patients31–33; serum total T3 concentrations did not change or were decreased with 40 mg or more daily34; and serum reverse T3 concentrations were increased with daily doses of 80 mg or more.31 It is most likely that propranolol exerts these changes by inhibiting D1 activity in peripheral tissues.

Furthermore, a significant decrease in serum total T3 concentrations was observed in hyperthyroid patients treated with atenolol 100 mg daily, metoprolol 100 mg daily, and alprenolol 100 mg daily, but not with sotalol 80 mg daily or nadolol (up to 240 mg daily).35,36

On the other hand, beta-adrenergic agonists have not been reported to cause significant changes in thyroid function tests.37

SUBCLINICAL THYROTOXICOSIS OR HASHIMOTO THYROIDITIS?

Our patient’s thyroid function test results are more likely due to his nonthyroidal illness and glucocorticoid therapy, as there is no clinical evidence to point to a hypothalamic-pituitary disorder accounting for true central hypothyroidism.

The other options mentioned in question 2 are unlikely to explain our patient’s thyroid function test results.

Subclinical thyrotoxicosis is characterized by suppressed serum TSH, but both serum free T4 and total T3 remain within the normal reference ranges. In addition, the serum TSH level may help to differentiate between thyrotoxicosis and nonthyroidal illness. In the former, serum TSH is usually suppressed (< 0.01 mU/L), whereas in the latter it is usually low but detectable (0.05– 0.3 mU/L).38,39

Hashimoto thyroiditis is a chronic autoimmune thyroid disease characterized by diffuse lymphocytic infiltration of the thyroid gland. Almost all patients with Hashimoto thyroiditis have elevated levels of antibodies to thyroid peroxidase or thyroglobulin.40 Clinically, patients with Hashimoto thyroiditis can either be hypothyroid or have normal thyroid function, which is not the case in our patient.

CASE CONTINUED

An endocrinologist, consulted for a second opinion, agreed that the patient’s thyroid function test results were most likely due to his nonthyroidal illness and glucocorticoid therapy.

3. In view of the endocrinologist’s opinion, which should be the next step in the management of the patient’s thyroid condition?

  • Start levothyroxine (T4) therapy
  • Start liothyronine (T3) therapy
  • Start N-acetylcysteine therapy
  • Start thyrotropin-releasing hormone therapy
  • Remeasure thyroid hormones after full recovery from his acute illness

It is not clear whether the changes in thyroid hormone levels during an acute illness are a pathologic alteration for which thyroid hormone therapy may be beneficial, or a physiologic adaptation for which such therapy would not be indicated.41

However, current data argue against thyroid hormone therapy using T4 or T3 for patients with nonthyroidal illness syndrome (also called euthyroid sick syndrome).42 Indeed, several randomized controlled trials showed that thyroid hormone therapy is not beneficial in such patients and may be detrimental.41,43

Therapies other than thyroid hormone have been investigated to ameliorate thyroid hormone abnormalities in patients with nonthyroidal illness. These include N-acetylcysteine, thyrotropin-releasing hormone therapy, and nutritional support.

Some studies showed that giving N-acetyl­cysteine, an antioxidant, increased serum T3 and decreased serum reverse T3 concentrations in patients with acute myocardial infarction.44 Nevertheless, the mortality rate and length of hospitalization were not affected. Further studies are needed to know whether N-acetylcysteine therapy is beneficial for such patients.

Similarly, a study using a thyrotropin-releasing hormone analogue along with growth hormone-releasing peptide 2 showed an increase in serum TSH, T4, and T3 levels in critically ill patients.45 The benefit of this therapy has yet to be determined. On the other hand, early nutritional support was reported to prevent thyroid hormonal changes in patients postoperatively.46

Measuring thyroid hormone levels after full recovery is the most appropriate next step in our patient, as the changes in thyroid hormone concentrations subside as the acute illness resolves.47

 

 

CASE CONTINUED

The patient continued to improve. On hospital day 6, he was feeling better but still had mild respiratory distress. There had been no further episodes of arrhythmia since day 4. His blood pressure was 136/86 mm Hg, heart rate 88 beats per minute and regular, respiratory rate 18 breaths per minute, and oral temperature 37.1°C. His oxygen saturation was 92% on room air.

Before discharge, he was encouraged to quit smoking. He was offered behavioral counseling and medication therapy, but he only said that he would think about it. He was discharged on oral cefixime for 4 more days and was instructed to switch to a long-acting bronchodilator along with his other home medications and to return in 1 week to have his thyroid hormones checked.

One week later, his laboratory results were:

  • TSH 11.2 mU/L (reference range 0.5–5.0)
  • Free T4 1.2 ng/dL (0.9–2.4)
  • Total T3 92 ng/dL (80–180).

Clinically, the patient was euthyroid, and examination of his thyroid was unremarkable.

4. Based on these last test results, which statement is correct?

  • Levothyroxine therapy should be started
  • His serum TSH elevation is most likely transient
  • Thyroid ultrasonography is strongly indicated
  • A radioactive iodine uptake study should be performed
  • Measurement of thyroid-stimulating immunoglobulins is indicated

During recovery from nonthyroidal illness, some patients may have elevated serum TSH levels that are usually transient and modest (< 20 mU/L).48 Normalization of the thyroid function tests including serum TSH may take several weeks49 or months.50 However, a systematic review found that the likelihood of permanent primary hypothyroidism is high in patients with serum TSH levels higher than 20 mU/L during the recovery phase of their nonthyroidal illness.51

Ultrasonography is useful for evaluating patients with thyroid nodules or goiter but is of little benefit for patients like ours, in whom the thyroid is normal on examination.

Similarly, a radioactive iodine uptake study is not indicated, as it is principally used to help differentiate between types of thyrotoxicosis. (Radioactive iodine is also used to treat differentiated thyroid cancer.)

Thyroid-stimulating immunoglobins are TSH receptor-stimulating antibodies that cause Graves disease. Nevertheless, measuring them is not routinely indicated for its diagnosis. However, their measurement is of significant help in the diagnosis of Graves disease if a radioactive iodine uptake study cannot be performed (as in pregnancy) and in atypical presentations such as euthyroid Graves ophthalmopathy.52 Other indications for thyroid-stimulating immunoglobin measurement are beyond the scope of the article. Our patient’s test results are not consistent with hyperthyroidism, so measuring thyroid-stimulating immunoglobins is not indicated.

CASE CONCLUSION: BETTER, BUT STILL SMOKING

The patient missed his 1-month clinic follow-up, but he visited the clinic for follow-up 3 months later. He was feeling well with no complaints. Test results including serum TSH, free T4, and total T3 were within normal ranges. His COPD was under control, with an FEV1 88% of predicted.

He was again encouraged to quit smoking and was offered drug therapy and behavioral counseling, but he declined. In addition, he was instructed to adhere to his annual influenza vaccination.

KEY POINTS

  • In patients with acute illness, it is recommended that thyroid function not be assessed unless there is a strong indication.
  • If thyroid function assessment is indicated for critically ill patients, serum TSH and free T4 concentrations should be measured. Some clinicians also measure serum total T3 level.
  • Thyroid function testing in critically ill patients usually shows low serum total T3, normal or low serum TSH, and normal or low serum free T4.
  • Many drugs can alter thyroid hormone levels.
  • Thyroid hormone therapy is not recommended for critically ill patients with low T3, low T4, or both.
  • During recovery from nonthyroidal illness, some patients may have mild elevation in serum TSH levels (< 20 mU/L).
  • Thyroid hormone levels may take several weeks or months to return to normal after the acute illness.
  • Patients with serum TSH levels higher than 20 mU/L during the recovery phase of their nonthyroidal illness are more likely to have permanent primary hypothyroidism.
References
  1. Lamb EJ, Martin J. Thyroid function tests: often justified in the acutely ill. Ann Clin Biochem 2000; 37(pt 2):158–164. doi:10.1258/0004563001899159
  2. Spencer CA, LoPresti JS, Patel A, et al. Applications of a new chemiluminometric thyrotropin assay to subnormal measurement. J Clin Endocrinol Metab 1990; 70(2):453–460. doi:10.1210/jcem-70-2-453
  3. Ross DS, Ardisson LJ, Meskell MJ. Measurement of thyrotropin in clinical and subclinical hyperthyroidism using a new chemiluminescent assay. J Clin Endocrinol Metab 1989; 69(3):684–688. doi:10.1210/jcem-69-3-684
  4. Koulouri O, Moran C, Halsall D, Chatterjee K, Gurnell M. Pitfalls in the measurement and interpretation of thyroid function tests. Best Pract Res Clin Endocrinol Metab 2013; 27(6):745–762. doi:10.1016/j.beem.2013.10.003
  5. Lechan RM, Fekete C. Role of thyroid hormone deiodination in the hypothalamus. Thyroid 2005; 15(8):883–897. doi:10.1089/thy.2005.15.883
  6. Chopra IJ, Hershman JM, Pardridge WM, Nicoloff JT. Thyroid function in nonthyroidal ilnesses. Ann Intern Med 1983; 98(6):946–957. doi:10.7326/0003-4819-98-6-946
  7. Chopra IJ, Solomon DH, Hepner HW, Mortenstein AA. Misleadingly low free thyroxine index and usefulness of reverse triiodothyronine measurement in nonthyroidal illnesses. Ann Intern Med 1979; 90(6):905–912. doi:10.7326/0003-4819-90-6-905
  8. Pontecorvi A, Robbins J. The plasma membrane and thyroid hormone entry into cells. Trends Endocrinol Metab 1989; 1(2):90–94. pmid:18411097
  9. Hennemann G, Krenning EP. Pitfalls in the interpretation of thyroid function tests in old age and non-thyroidal illness. Horm Res 1987; 26(1–4):100–104. doi:10.1159/000180688
  10. Baloch Z, Carayon P, Conte-Devolx B, et al; Guidelines Committee, National Academy of Clinical Biochemistry. Laboratory medicine practice guidelines. Laboratory support for the diagnosis and monitoring of thyroid disease. Thyroid 2003; 13(1):3–126. doi:10.1089/105072503321086962
  11. Lum S, Nicoloff JT, Spencer CA, Kaptein EM. Peripheral tissue mechanism for maintenance of serum triiodothyronine values in a thyroxine-deficient state in man. J Clin Invest 1984; 73(2):570–575. doi:10.1172/JCI111245
  12. Ortiga-Carvalho TM, Chiamolera MI, Pazos-Moura CC, Wondisford FE. Hypothalamus-pituitary-thyroid axis. Compr Physiol 2016; 6(3):1387–1428. doi:10.1002/cphy.c150027
  13. de Vries EM, Fliers E, Boelen A. The molecular basis of the non-thyroidal illness syndrome. J Endocrinol 2015; 225(3):R67–R81. doi:10.1530/JOE-15-0133
  14. Chopra IJ, Huang TS, Beredo A, Solomon DH, Teco GN, Mean JF. Evidence for an inhibitor of extrathyroidal conversion of thyroxine to 3, 5, 3'-triiodothyronine in sera of patients with nonthyroidal illnesses. J Clin Endocrinol Metab 1985; 60(4):666–672. doi:10.1210/jcem-60-4-666
  15. Peeters RP, Debaveye Y, Fliers E, Visser TJ. Changes within the thyroid axis during critical illness. Crit Care Clin 2006; 22(1):41–55. doi:10.1016/j.ccc.2005.08.006
  16. Spencer C, Eigen A, Shen D, et al. Specificity of sensitive assays of thyrotropin (TSH) used to screen for thyroid disease in hospitalized patients. Clin Chem 1987; 33(8):1391–1396. pmid:3301067
  17. Adler SM, Wartofsky L. The nonthyroidal illness syndrome. Endocrinol Metab Clin North Am 2007; 36(3):657–672. doi:10.1016/j.ecl.2007.04.007
  18. Persani L. Central hypothyroidism: pathogenic, diagnostic, and therapeutic challenges. J Clin Endocrinol Metab 2012; 97(9):3068–3078. doi:10.1210/jc.2012-1616
  19. Kidess AI, Caplan RH, Reynertson RH, Wickus GG, Goodnough DE. Transient corticotropin deficiency in critical illness. Mayo Clin Proc 1993; 68(5):435–441. doi:10.1016/s0025-6196(12)60188-8
  20. Lamberts SW, Bruining HA, De Jong FH. Corticosteroid therapy in severe illness. N Engl J Med 1997; 337(18):1285–1292. doi:10.1056/NEJM199710303371807
  21. Burmeister LA. Reverse T3 does not reliably differentiate hypothyroid sick syndrome from euthyroid sick syndrome. Thyroid 1995; 5(6):435–441. doi:10.1089/thy.1995.5.435
  22. Duick DS, Warren DW, Nicoloff JT, Otis CL, Croxson MS. Effect of single dose dexamethasone on the concentration of serum triiodothyronine in man. J Clin Endocrinol Metab 1974; 39(6):1151–1154. doi:10.1210/jcem-39-6-1151
  23. Gamstedt A, Järnerot G, Kågedal B. Dose related effects of betamethasone on iodothyronines and thyroid hormone-binding proteins in serum. Acta Endocrinol (Copenh) 1981; 96(4):484–490. doi:10.1530/acta.0.0960484
  24. Wartofsky L, Burman KD. Alterations in thyroid function in patients with systemic illness: the “euthyroid sick syndrome.” Endocr Rev 1982; 3(2):164–217. doi:10.1210/edrv-3-2-164
  25. Wilber JF, Utiger RD. The effect of glucocorticoids on thyrotropin secretion. J Clin Invest 1969; 48(11):2096–2103. doi:10.1172/JCI106176
  26. Nicoloff JT, Fisher DA, Appleman MD Jr. The role of glucocorticoids in the regulation of thyroid function in man. J Clin Invest 1970; 49(10):1922–1929. doi:10.1172/JCI106411
  27. Surks MI, Sievert R. Drugs and thyroid function. N Engl J Med 1995; 333(25):1688–1694. doi:10.1056/NEJM199512213332507
  28. Haugen BR. Drugs that suppress TSH or cause central hypothyroidism. Best Pract Res Clin Endocrinol Metab 2009; 23(6):793–800. doi:10.1016/j.beem.2009.08.003
  29. Sherman SI, Gopal J, Haugen BR, et al. Central hypothyroidism associated with retinoid X receptor–selective ligands. N Engl J Med 1999; 340(14):1075–1079. doi:10.1056/NEJM199904083401404
  30. Murchison LE, How J, Bewsher PD. Comparison of propranolol and metoprolol in the management of hyperthyroidism. Br J Clin Pharmacol 1979; 8(6):581–587. doi:10.1111/j.1365-2125.1979.tb01048.x
  31. Faber J, Friis T, Kirkegaard C, et al. Serum T4, T3 and reverse T3 during treatment with propranolol in hyperthyroidism, L-T4 treated myxedema and in normal man. Horm Metab Res 1979; 11(1):34–36. doi:10.1055/s-0028-1092678
  32. Kristensen BO, Weeke J. Propranolol-induced increments in total and free serum thyroxine in patients with essential hypertension. Clin Pharmacol Ther 1977; 22(6):864–867. doi:10.1002/cpt1977226864
  33. Murchison LE, Bewsher PD, Chesters MI, Ferrier WR. Comparison of propranolol and practolol in the management of hyperthyroidism. Br J Clin Pharmacol 1976; 3(2):273–277. doi:10.1111/j.1365-2125.1976.tb00603.x
  34. Lotti G, Delitala G, Devilla L, Alagna S, Masala A. Reduction of plasma triiodothyronine (T3) induced by propranolol. Clin Endocrinol 1977; 6(6):405–410. doi:10.1111/j.1365-2265.1977.tb03322.x
  35. Perrild H, Hansen JM, Skovsted L, Christensen LK. Different effects of propranolol, alprenolol, sotalol, atenolol and metoprolol on serum T3 and serum rT3 in hyperthyroidism. Clin Endocrinol (Oxf) 1983; 18(2):139–142. pmid:6133659
  36. Reeves RA, From GL, Paul W, Leenen FH. Nadolol, propranolol, and thyroid hormones: evidence for a membrane-stabilizing action of propranolol. Clin Pharmacol Ther 1985; 37(2):157–161. doi:10.1038/clpt.1985.28
  37. Walker N, Jung RT, Jennings G, James WP. The effect of a beta-receptor agonist (salbutamol) on peripheral thyroid metabolism in euthyroid subjects. Horm Metab Res 1981; 13(10):590–591. doi:10.1055/s-2007-1019346
  38. Melmed S, Geola FL, Reed AW, Pekary AE, Park J, Hershman JM. A comparison of methods for assessing thyroid function in nonthyroidal illness. J Clin Endocrinol Metab 1982; 54(2):300–306. doi:10.1210/jcem-54-2-300
  39. Docter R, Krenning E, De Jong M, Hennemann G. The sick euthyroid syndrome: changes in thyroid hormone serum parameters and hormone metabolism. Clin Endocrinol (Oxf) 1993; 39(5):499–518. pmid:8252737
  40. Mariotti S, Caturegli P, Piccolo P, Barbesino G, Pinchera A. Antithyroid peroxidase autoantibodies in thyroid diseases. J Clin Endocrinol Metab 1990; 71(3):661–669. doi:10.1210/jcem-71-3-661
  41. De Groot LJ. Non-thyroidal illness syndrome is a manifestation of hypothalamic-pituitary dysfunction, and in view of current evidence, should be treated with appropriate replacement therapies. Crit Care Clin 2006; 22(1):57–86. doi:10.1016/j.ccc.2005.10.001
  42. Jonklaas J, Bianco AC, Bauer AJ, et al; American Thyroid Association Task Force on Thyroid Hormone Replacement. Guidelines for the treatment of hypothyroidism: prepared by the American Thyroid Association Task Force on Thyroid Hormone Replacement. Thyroid 2014; 24(12):1670–1751. doi:10.1089/thy.2014.0028
  43. Kaptein EM, Beale E, Chan LS. Thyroid hormone therapy for obesity and nonthyroidal illnesses: a systematic review. J Clin Endocrinol Metab 2009; 94(10):3663–3675. doi:10.1210/jc.2009-0899
  44. Vidart J, Wajner SM, Leite RS, et al. N-acetylcysteine administration prevents nonthyroidal illness syndrome in patients with acute myocardial infarction: a randomized clinical trial. J Clin Endocrinol Metab 2014; 99(12):4537–4545. doi:10.1210/jc.2014-2192
  45. Van den Berghe G, Wouters P, Weekers F, et al. Reactivation of pituitary hormone release and metabolic improvement by infusion of growth hormone-releasing peptide and thyrotropin-releasing hormone in patients with protracted critical illness. J Clin Endocrinol Metab 1999; 84(4):1311–1323. doi:10.1210/jcem.84.4.5636
  46. Langouche L, Vander Perre S, Marques M, et al. Impact of early nutrient restriction during critical illness on the nonthyroidal illness syndrome and its relation with outcome: a randomized, controlled clinical study. J Clin Endocrinol Metab 2013; 98(3):1006–1013. doi:10.1210/jc.2012-2809
  47. Economidou F, Douka E, Tzanela M, Nanas S, Kotanidou A. Thyroid function during critical illness. Hormones (Athens) 2011; 10(2):117–124. doi:10.14310/horm.2002.1301
  48. Hamblin PS, Dyer SA, Mohr VS, et al. Relationship between thyrotropin and thyroxine changes during recovery from severe hypothyroxinemia of critical illness. J Clin Endocrinol Metab 1986; 62(4):717–722. doi:10.1210/jcem-62-4-717
  49. Iglesias P, Diez JJ. Thyroid dysfunction and kidney disease. Eur J Endocrinol 2009; 160(4):503–515. doi:10.1530/EJE-08-0837
  50. Spencer CA. Clinical utility and cost-effectiveness of sensitive thyrotropin assays in ambulatory and hospitalized patients. Mayo Clin Proc 1988; 63(12):1214–1222. doi:10.1016/s0025-6196(12)65408-1
  51. Attia J, Margetts P, Guyatt G. Diagnosis of thyroid disease in hospitalized patients: a systematic review. Arch Intern Med 1999; 159(7):658–665. pmid:10218744
  52. Barbesino G, Tomer Y. Clinical review: clinical utility of TSH receptor antibodies. J Clin Endocrinol Metab 2013; 98(6):2247–2255. doi:10.1210/jc.2012-4309
References
  1. Lamb EJ, Martin J. Thyroid function tests: often justified in the acutely ill. Ann Clin Biochem 2000; 37(pt 2):158–164. doi:10.1258/0004563001899159
  2. Spencer CA, LoPresti JS, Patel A, et al. Applications of a new chemiluminometric thyrotropin assay to subnormal measurement. J Clin Endocrinol Metab 1990; 70(2):453–460. doi:10.1210/jcem-70-2-453
  3. Ross DS, Ardisson LJ, Meskell MJ. Measurement of thyrotropin in clinical and subclinical hyperthyroidism using a new chemiluminescent assay. J Clin Endocrinol Metab 1989; 69(3):684–688. doi:10.1210/jcem-69-3-684
  4. Koulouri O, Moran C, Halsall D, Chatterjee K, Gurnell M. Pitfalls in the measurement and interpretation of thyroid function tests. Best Pract Res Clin Endocrinol Metab 2013; 27(6):745–762. doi:10.1016/j.beem.2013.10.003
  5. Lechan RM, Fekete C. Role of thyroid hormone deiodination in the hypothalamus. Thyroid 2005; 15(8):883–897. doi:10.1089/thy.2005.15.883
  6. Chopra IJ, Hershman JM, Pardridge WM, Nicoloff JT. Thyroid function in nonthyroidal ilnesses. Ann Intern Med 1983; 98(6):946–957. doi:10.7326/0003-4819-98-6-946
  7. Chopra IJ, Solomon DH, Hepner HW, Mortenstein AA. Misleadingly low free thyroxine index and usefulness of reverse triiodothyronine measurement in nonthyroidal illnesses. Ann Intern Med 1979; 90(6):905–912. doi:10.7326/0003-4819-90-6-905
  8. Pontecorvi A, Robbins J. The plasma membrane and thyroid hormone entry into cells. Trends Endocrinol Metab 1989; 1(2):90–94. pmid:18411097
  9. Hennemann G, Krenning EP. Pitfalls in the interpretation of thyroid function tests in old age and non-thyroidal illness. Horm Res 1987; 26(1–4):100–104. doi:10.1159/000180688
  10. Baloch Z, Carayon P, Conte-Devolx B, et al; Guidelines Committee, National Academy of Clinical Biochemistry. Laboratory medicine practice guidelines. Laboratory support for the diagnosis and monitoring of thyroid disease. Thyroid 2003; 13(1):3–126. doi:10.1089/105072503321086962
  11. Lum S, Nicoloff JT, Spencer CA, Kaptein EM. Peripheral tissue mechanism for maintenance of serum triiodothyronine values in a thyroxine-deficient state in man. J Clin Invest 1984; 73(2):570–575. doi:10.1172/JCI111245
  12. Ortiga-Carvalho TM, Chiamolera MI, Pazos-Moura CC, Wondisford FE. Hypothalamus-pituitary-thyroid axis. Compr Physiol 2016; 6(3):1387–1428. doi:10.1002/cphy.c150027
  13. de Vries EM, Fliers E, Boelen A. The molecular basis of the non-thyroidal illness syndrome. J Endocrinol 2015; 225(3):R67–R81. doi:10.1530/JOE-15-0133
  14. Chopra IJ, Huang TS, Beredo A, Solomon DH, Teco GN, Mean JF. Evidence for an inhibitor of extrathyroidal conversion of thyroxine to 3, 5, 3'-triiodothyronine in sera of patients with nonthyroidal illnesses. J Clin Endocrinol Metab 1985; 60(4):666–672. doi:10.1210/jcem-60-4-666
  15. Peeters RP, Debaveye Y, Fliers E, Visser TJ. Changes within the thyroid axis during critical illness. Crit Care Clin 2006; 22(1):41–55. doi:10.1016/j.ccc.2005.08.006
  16. Spencer C, Eigen A, Shen D, et al. Specificity of sensitive assays of thyrotropin (TSH) used to screen for thyroid disease in hospitalized patients. Clin Chem 1987; 33(8):1391–1396. pmid:3301067
  17. Adler SM, Wartofsky L. The nonthyroidal illness syndrome. Endocrinol Metab Clin North Am 2007; 36(3):657–672. doi:10.1016/j.ecl.2007.04.007
  18. Persani L. Central hypothyroidism: pathogenic, diagnostic, and therapeutic challenges. J Clin Endocrinol Metab 2012; 97(9):3068–3078. doi:10.1210/jc.2012-1616
  19. Kidess AI, Caplan RH, Reynertson RH, Wickus GG, Goodnough DE. Transient corticotropin deficiency in critical illness. Mayo Clin Proc 1993; 68(5):435–441. doi:10.1016/s0025-6196(12)60188-8
  20. Lamberts SW, Bruining HA, De Jong FH. Corticosteroid therapy in severe illness. N Engl J Med 1997; 337(18):1285–1292. doi:10.1056/NEJM199710303371807
  21. Burmeister LA. Reverse T3 does not reliably differentiate hypothyroid sick syndrome from euthyroid sick syndrome. Thyroid 1995; 5(6):435–441. doi:10.1089/thy.1995.5.435
  22. Duick DS, Warren DW, Nicoloff JT, Otis CL, Croxson MS. Effect of single dose dexamethasone on the concentration of serum triiodothyronine in man. J Clin Endocrinol Metab 1974; 39(6):1151–1154. doi:10.1210/jcem-39-6-1151
  23. Gamstedt A, Järnerot G, Kågedal B. Dose related effects of betamethasone on iodothyronines and thyroid hormone-binding proteins in serum. Acta Endocrinol (Copenh) 1981; 96(4):484–490. doi:10.1530/acta.0.0960484
  24. Wartofsky L, Burman KD. Alterations in thyroid function in patients with systemic illness: the “euthyroid sick syndrome.” Endocr Rev 1982; 3(2):164–217. doi:10.1210/edrv-3-2-164
  25. Wilber JF, Utiger RD. The effect of glucocorticoids on thyrotropin secretion. J Clin Invest 1969; 48(11):2096–2103. doi:10.1172/JCI106176
  26. Nicoloff JT, Fisher DA, Appleman MD Jr. The role of glucocorticoids in the regulation of thyroid function in man. J Clin Invest 1970; 49(10):1922–1929. doi:10.1172/JCI106411
  27. Surks MI, Sievert R. Drugs and thyroid function. N Engl J Med 1995; 333(25):1688–1694. doi:10.1056/NEJM199512213332507
  28. Haugen BR. Drugs that suppress TSH or cause central hypothyroidism. Best Pract Res Clin Endocrinol Metab 2009; 23(6):793–800. doi:10.1016/j.beem.2009.08.003
  29. Sherman SI, Gopal J, Haugen BR, et al. Central hypothyroidism associated with retinoid X receptor–selective ligands. N Engl J Med 1999; 340(14):1075–1079. doi:10.1056/NEJM199904083401404
  30. Murchison LE, How J, Bewsher PD. Comparison of propranolol and metoprolol in the management of hyperthyroidism. Br J Clin Pharmacol 1979; 8(6):581–587. doi:10.1111/j.1365-2125.1979.tb01048.x
  31. Faber J, Friis T, Kirkegaard C, et al. Serum T4, T3 and reverse T3 during treatment with propranolol in hyperthyroidism, L-T4 treated myxedema and in normal man. Horm Metab Res 1979; 11(1):34–36. doi:10.1055/s-0028-1092678
  32. Kristensen BO, Weeke J. Propranolol-induced increments in total and free serum thyroxine in patients with essential hypertension. Clin Pharmacol Ther 1977; 22(6):864–867. doi:10.1002/cpt1977226864
  33. Murchison LE, Bewsher PD, Chesters MI, Ferrier WR. Comparison of propranolol and practolol in the management of hyperthyroidism. Br J Clin Pharmacol 1976; 3(2):273–277. doi:10.1111/j.1365-2125.1976.tb00603.x
  34. Lotti G, Delitala G, Devilla L, Alagna S, Masala A. Reduction of plasma triiodothyronine (T3) induced by propranolol. Clin Endocrinol 1977; 6(6):405–410. doi:10.1111/j.1365-2265.1977.tb03322.x
  35. Perrild H, Hansen JM, Skovsted L, Christensen LK. Different effects of propranolol, alprenolol, sotalol, atenolol and metoprolol on serum T3 and serum rT3 in hyperthyroidism. Clin Endocrinol (Oxf) 1983; 18(2):139–142. pmid:6133659
  36. Reeves RA, From GL, Paul W, Leenen FH. Nadolol, propranolol, and thyroid hormones: evidence for a membrane-stabilizing action of propranolol. Clin Pharmacol Ther 1985; 37(2):157–161. doi:10.1038/clpt.1985.28
  37. Walker N, Jung RT, Jennings G, James WP. The effect of a beta-receptor agonist (salbutamol) on peripheral thyroid metabolism in euthyroid subjects. Horm Metab Res 1981; 13(10):590–591. doi:10.1055/s-2007-1019346
  38. Melmed S, Geola FL, Reed AW, Pekary AE, Park J, Hershman JM. A comparison of methods for assessing thyroid function in nonthyroidal illness. J Clin Endocrinol Metab 1982; 54(2):300–306. doi:10.1210/jcem-54-2-300
  39. Docter R, Krenning E, De Jong M, Hennemann G. The sick euthyroid syndrome: changes in thyroid hormone serum parameters and hormone metabolism. Clin Endocrinol (Oxf) 1993; 39(5):499–518. pmid:8252737
  40. Mariotti S, Caturegli P, Piccolo P, Barbesino G, Pinchera A. Antithyroid peroxidase autoantibodies in thyroid diseases. J Clin Endocrinol Metab 1990; 71(3):661–669. doi:10.1210/jcem-71-3-661
  41. De Groot LJ. Non-thyroidal illness syndrome is a manifestation of hypothalamic-pituitary dysfunction, and in view of current evidence, should be treated with appropriate replacement therapies. Crit Care Clin 2006; 22(1):57–86. doi:10.1016/j.ccc.2005.10.001
  42. Jonklaas J, Bianco AC, Bauer AJ, et al; American Thyroid Association Task Force on Thyroid Hormone Replacement. Guidelines for the treatment of hypothyroidism: prepared by the American Thyroid Association Task Force on Thyroid Hormone Replacement. Thyroid 2014; 24(12):1670–1751. doi:10.1089/thy.2014.0028
  43. Kaptein EM, Beale E, Chan LS. Thyroid hormone therapy for obesity and nonthyroidal illnesses: a systematic review. J Clin Endocrinol Metab 2009; 94(10):3663–3675. doi:10.1210/jc.2009-0899
  44. Vidart J, Wajner SM, Leite RS, et al. N-acetylcysteine administration prevents nonthyroidal illness syndrome in patients with acute myocardial infarction: a randomized clinical trial. J Clin Endocrinol Metab 2014; 99(12):4537–4545. doi:10.1210/jc.2014-2192
  45. Van den Berghe G, Wouters P, Weekers F, et al. Reactivation of pituitary hormone release and metabolic improvement by infusion of growth hormone-releasing peptide and thyrotropin-releasing hormone in patients with protracted critical illness. J Clin Endocrinol Metab 1999; 84(4):1311–1323. doi:10.1210/jcem.84.4.5636
  46. Langouche L, Vander Perre S, Marques M, et al. Impact of early nutrient restriction during critical illness on the nonthyroidal illness syndrome and its relation with outcome: a randomized, controlled clinical study. J Clin Endocrinol Metab 2013; 98(3):1006–1013. doi:10.1210/jc.2012-2809
  47. Economidou F, Douka E, Tzanela M, Nanas S, Kotanidou A. Thyroid function during critical illness. Hormones (Athens) 2011; 10(2):117–124. doi:10.14310/horm.2002.1301
  48. Hamblin PS, Dyer SA, Mohr VS, et al. Relationship between thyrotropin and thyroxine changes during recovery from severe hypothyroxinemia of critical illness. J Clin Endocrinol Metab 1986; 62(4):717–722. doi:10.1210/jcem-62-4-717
  49. Iglesias P, Diez JJ. Thyroid dysfunction and kidney disease. Eur J Endocrinol 2009; 160(4):503–515. doi:10.1530/EJE-08-0837
  50. Spencer CA. Clinical utility and cost-effectiveness of sensitive thyrotropin assays in ambulatory and hospitalized patients. Mayo Clin Proc 1988; 63(12):1214–1222. doi:10.1016/s0025-6196(12)65408-1
  51. Attia J, Margetts P, Guyatt G. Diagnosis of thyroid disease in hospitalized patients: a systematic review. Arch Intern Med 1999; 159(7):658–665. pmid:10218744
  52. Barbesino G, Tomer Y. Clinical review: clinical utility of TSH receptor antibodies. J Clin Endocrinol Metab 2013; 98(6):2247–2255. doi:10.1210/jc.2012-4309
Issue
Cleveland Clinic Journal of Medicine - 86(10)
Issue
Cleveland Clinic Journal of Medicine - 86(10)
Page Number
666-675
Page Number
666-675
Publications
Publications
Topics
Article Type
Display Headline
A 66-year-old man with abnormal thyroid function tests
Display Headline
A 66-year-old man with abnormal thyroid function tests
Legacy Keywords
thyroid, thyroid function tests, hypothyroid, chronic obstructive pulmonary disease, COPD, thyroid-stimulating hormone, TSH, thyroxine, T4, triiodothyronine, T3, reverse T3, deiodinase, D1, euthyroid sick syndrome, nonthyroidal illness syndrome, Yazan Alhalaseh, Zaid Abdulelah, Ahmad Armouti, Ayman Zayed
Legacy Keywords
thyroid, thyroid function tests, hypothyroid, chronic obstructive pulmonary disease, COPD, thyroid-stimulating hormone, TSH, thyroxine, T4, triiodothyronine, T3, reverse T3, deiodinase, D1, euthyroid sick syndrome, nonthyroidal illness syndrome, Yazan Alhalaseh, Zaid Abdulelah, Ahmad Armouti, Ayman Zayed
Sections
Disallow All Ads
Content Gating
No Gating (article Unlocked/Free)
Alternative CME
Disqus Comments
Default
Gate On Date
Mon, 09/23/2019 - 15:15
Un-Gate On Date
Mon, 09/23/2019 - 15:15
Use ProPublica
CFC Schedule Remove Status
Mon, 09/23/2019 - 15:15
Hide sidebar & use full width
render the right sidebar.
Article PDF Media

A young man with acute chest pain

Article Type
Changed
Tue, 09/03/2019 - 08:23
Display Headline
A young man with acute chest pain

An 18-year-old man without any significant medical history was transferred from another hospital for higher-level care after presenting with unremitting chest pain. He had been in his usual state of good health until 7 days before presentation, when he developed mild rhinorrhea and a sore throat, but not a cough. He went to an outpatient clinic, where a rapid test for group A streptococci was done; the result was negative, and he was sent home on supportive measures.

On the day of admission, he awoke with severe, pressure-like, midsternal, nonradiating pain, which he rated 10 on a scale of 10. The pain intensified in the supine position and improved with sitting. A complete review of systems was otherwise negative. He denied having had similar symptoms in the past, as well as sick contacts, recent travel, toxin exposure, illicit substance abuse, pets at home, or tick bites. His family history was negative for cardiac arrhythmias, premature coronary artery disease, thoracic aneurysms or dissection, and infiltrative disorders. His surgical and social histories were unremarkable. He said he had no drug allergies.

farid_myocarditis_f1.jpg
%3Cp%3EFigure%201.%20The%20patient%E2%80%99s%20electrocardiogram%20on%20presentation%20shows%20ST-segment%20elevation%20(arrows)%20over%20the%20lateral%20and%20inferior%20distribution%20(V%3Csub%3E4%3C%2Fsub%3E%E2%80%93V%3Csub%3E6%3C%2Fsub%3E%2C%20II%2C%20III%2C%20and%20aVF).%3C%2Fp%3E
An electrocardiogram was obtained (Figure 1). His troponin I level was 7.0 ng/mL (reference range < 0.04 ng/mL).

On examination, his temperature was 38.1°C (100.6°F), heart rate 101 beats per minute, blood pressure 142/78 mm Hg, respiratory rate 16 breaths per minute, and oxygen saturation 98% on room air. He appeared anxious but was in no acute distress. Neck examination showed no elevation in jugular venous pulsation, bruits, thyromegaly, or lymphadenopathy. Cardiac examination revealed tachycardia without murmurs, rubs, or gallops. Lungs were clear to auscultation. Examination of all 4 extremities found 2+ pulses (on a scale of 0 to 4+) throughout and no cyanosis, clubbing, or edema. Abdominal, neurologic, and dermatologic examinations were unremarkable.

Further blood testing revealed the following:

  • Troponin I (3 hours after the first level) 15.5 ng/mL
  • B-type natriuretic peptide 200 mg/dL (reference range 0–100 mg/dL)
  • C-reactive protein 0.9 mg/dL (reference range 0.0–0.8 mg/dL)
  • Erythrocyte sedimentation rate 10 mm/h (reference range < 15 mm/h).

Metabolic and hematologic assessments were unremarkable. A toxicology screen for drugs of abuse was negative. Viral serologic testing was not done.

A chest radiograph showed no acute cardiopulmonary processes.

Given his presenting symptoms, persistent tachycardia, rapidly rising troponin I level, and electrocardiogram showing diffuse ST elevation, he was taken for urgent cardiac catheterization. Coronary angiography revealed no evidence of atherosclerotic disease, acute thrombosis, dissection, or aneurysm. Echocardiography 2 hours after the procedure showed a normal ejection fraction and no regional wall-motion abnormalities or valvular heart disease.

 

 

FURTHER TESTING

1. Which test should be done next to further evaluate this patient’s chest pain?

  • Serum viral serologic testing
  • Serum free light chain assay
  • Nuclear myocardial perfusion study
  • Cardiac magnetic resonance imaging (MRI)
  • Endomyocardial biopsy

In this patient without ischemic coronary disease or valvular heart disease, the recent upper respiratory tract prodrome, active positional chest pain, and diffuse electrocardiographic changes raise the possibility of myocarditis with pericardial involvement.

Viral serologic tests

Viral serologic tests are often obtained in the workup of myocarditis as a noninvasive means of detecting an infectious cause.

However, this approach has several problems. First, a positive serologic result is a signal of the peripheral immune response to a pathogen but does not necessarily indicate active myocardial inflammation. Additionally, circulating immunoglobulin G against cardiotropic viruses is commonly found, even in the absence of myocarditis.1 This is often the result of a high prevalence and exposure to these viruses in the general population. Further, trials have shown no correlation between serologic results and organisms identified by endomyocardial biopsy.2

Thus, serologic testing seems to be of limited utility, reserved for testing for infection with Borrelia burgdorferi (Lyme disease) in endemic areas, hepatitis C virus, human immunodeficiency virus in patients at high risk, Rickettsia conorii, and Rickettsia rickettsii.3

Serum free light chain testing for amyloidosis

Serum free light chain testing is replacing serum and urine protein electrophoresis in the workup of cardiac amyloidosis,4 as electrophoresis has poor sensitivity.4,5

Cardiac amyloidosis often affects older persons, although in rare cases it can affect young patients who carry mutations in the transthyretin gene (ATTR amyloidosis).6 This diagnosis is unlikely in our patient, as he has no other affected organ systems (amyloidosis often affects the renal and neurologic systems), normal QRS voltages on electrocardiography (which are often but not always low in amyloidosis), and no left ventricular hypertrophy or diastolic dysfunction on echocardiography (which are often seen in amyloidosis).4

Nuclear perfusion imaging for sarcoidosis

Nuclear imaging has a limited role in evaluating myocarditis,3 but positron-emission tomography with fluorine-18 fluorodeoxyglucose has a diagnostic role in sarcoidosis, an immune-mediated cause of myocarditis.7

Based on the acuity of the patient’s presentation, preceded by upper respiratory tract symptoms, sarcoidosis is less likely. Sarcoidosis is difficult to diagnose, although when it is the cause of myocarditis, some clues exist, as patients usually present with heart failure symptoms, a second- or third-degree atrioventricular block, or a dilated left ventricle on echocardiography.3 All of these were absent in our patient.

Cardiac MRI

Cardiac MRI has undergone many advances, making it an extremely useful noninvasive test. It has excellent utility as a stand-alone test in diagnosing myocarditis and has synergistic value when combined with endomyocardial biopsy.8 It is indicated in hemodynamically stable patients with a clinical suspicion of myocarditis, persistent symptoms, absence of heart failure, and when imaging findings will change management. It is particularly useful to help elucidate a cause and guide tailored therapy.9 Therefore, it is a reasonable next step in the diagnostic pathway for this patient.10

Cardiac MRI also allows for concurrent assessment of scar. In myocardial infarction, the late gadolinium enhancement is subendocardial or transmural. In myocarditis, the pattern differs, being found in the subepicardial lateral free wall (in most patients with parvovirus B19) and mid-myocardial septum (in most patients with herpesvirus 6).9,11 Cardiac MRI also confers prognostic information for patients with suspected myocarditis.12

The Lake Louise criteria9 for the diagnosis of myocarditis require 2 of the following:

  • Evidence of myocardial edema
  • Increased ratio of early gadolinium enhancement between myocardium and skeletal muscle (indicates hyperemia)
  • At least 1 focal lesion with nonischemic late gadolinium enhancement (indicates cardiac myocyte injury or scarring).

The Lake Louise criteria may be replaced by T1 and T2 mapping, which was found to be considerably better for diagnosing myocarditis when the 2 were compared.9,13,14

Endomyocardial biopsy

Endomyocardial biopsy should not be delayed while waiting for cardiac MRI in patients who are hemodynamically unstable or present with life-threatening features (ventricular arrhythmia, left ventricular failure, or resuscitation after sudden cardiac death).3,10

The indications for endomyocardial biopsy have been highly debated. The 2013 guidelines from the European Society of Cardiology (ESC) recommending endomyocardial biopsy  in all clinically suspected cases of myocarditis have only heightened the controversy.3 The American Heart Association (AHA) guidelines reserve biopsy for patients with suspected myocarditis who have acute or subacute heart failure symptoms or who do not respond to standard medical therapy.15 Other reasonable indications may include the following: myocarditis with life-threatening ventricular arrhythmias, suspicion of giant cell myocarditis, necrotizing eosinophilic myocarditis, or cardiac sarcoidosis.16

Endomyocardial biopsy is the only way to make a definitive diagnosis of myocarditis.3 However, given the patchy distribution of myocardial involvement, a negative result does not rule out myocarditis. The diagnostic utility can be improved by increasing the number of samples taken (at least 3 but up to 10), obtaining samples from both ventricles, and using cardiac MRI data to determine which sites to biopsy.3,13,17,18

Noninvasive testing such as cardiac MRI does not distinguish cell type or etiology (viral vs nonviral).3 Further, endomyocardial biopsy must be performed before immunosuppressive therapy can be safely started.3,16 At experienced centers, the complication rate is 0% to 0.8%.3 The addition of immunohistochemical testing and viral genomic detection by polymerase chain reaction testing have increased the sensitivity of this technique.19 Finally, endomyocardial biopsy can help rule out some of the other possibilities in the differential diagnosis for myocarditis, including infiltrative and storage diseases, and possibly cardiac tumors.3

Of additional note, the diffuse ST-segment elevation seen on the patient’s electrocardiogram (Figure 1) is indicative of subepicardial inflammation. Since the distribution involves more than one epicardial coronary territory, this helps to differentiate the changes from those that occur with myocardial infarction.20

 

 

CASE CONTINUED

farid_myocarditis_f2.jpg
%3Cp%3EFigure%202.%20Cardiac%20magnetic%20resonance%20imaging%20shows%20areas%20of%20patchy%20subepicardial%20late%20gadolinium%20enhancement%20(arrows).%3C%2Fp%3E
The patient underwent cardiac MRI, which showed myocardial edema and patchy areas of late gadolinium enhancement, raising suspicion for myocarditis (Figure 2).

Causes of myocarditis are numerous (Table 1),3,21,22 but viral and postinfectious etiologies remain the most common causes of acute myocarditis.23

farid_myocarditis_t1.jpg
2. What is the most likely causative infectious agent?

  • Parvovirus B19
  • Coxsackievirus B
  • Adenovirus species
  • Human herpesvirus 6
  • Staphylococcus aureus
  • Corynebacterium diphtheria
  • Trypanosoma cruzi
  • Influenza H1/N1

INFECTIOUS CAUSES OF MYOCARDITIS

Coxsackievirus B was the agent most often linked to this condition from the 1950s through the 1990s. However, in the last 2 decades, adenovirus species and human herpesvirus 6 have been increasingly encountered, and recently, parvovirus B19 has been credited as the most common culprit,11,23 at least in the Western world. In developing nations, T cruzi and C diphtheria are the most common offenders.21

S aureus is a common cause of endocarditis, but it rarely plays a role in myocarditis. When it does, the myocarditis is often the sequela of profound bacteremia. This was much more common before antibiotics were invented.24,25

Influenza H1/N1 is not among the most common causes of viral myocarditis, but it should be considered during flu season, given its ability to result in fulminant myocarditis.3,26

TREATMENT FOR MYOCARDITIS

3. Which treatment is the most appropriate at this time?

  • Intravenous immunoglobulin
  • Interferon beta
  • Acyclovir
  • Prednisone
  • Colchicine

Treatment for myocarditis depends on the cause but always includes supportive care to address the constellation of presenting symptoms. Standard therapies for tachy- or bradyarrhythmias, heart failure, and hemodynamic derangement should be started.

Supportive care

In patients with severe left ventricular dysfunction, an implantable cardiac electronic device, left ventricular assist device, or heart transplant may ultimately be needed. However, if possible these should be deferred for several months to determine response to treatment, since the myocardium can possibly recover.16

Diuretics, beta-blockers, angiotensin II receptor blockers, angiotensin-converting enzyme inhibitors, and aldosterone antagonists should be given as part of guideline-directed medical therapy for patients with heart failure and reduced ejection fraction.3,27 However, whether and how the patient should be weaned from these agents after disease recovery are unknown.3

Intravenous immunoglobulin

Intravenous immunoglobulin in high doses has had mixed results. Its efficacy is well documented in children,21 but limited supportive data are available in adults.3 As such, recent ESC guidelines do not provide recommendations regarding its use in adults.3

Interferon beta

Interferon beta has shown promise in improving New York Heart Association class and left ventricular ejection fraction.3 This is attributed to its effects on eliminating adenoviral species and enteroviruses. Treatment of enteroviral organisms in particular has been associated with improved 10-year prognosis.3 Interferon beta also has in vitro data showing efficacy at diminishing apoptosis from parvovirus B19.28

Nucleoside analogues

Empiric treatment with nucleoside analogues (acyclovir, ganciclovir, and valacyclovir) has been tried for patients in whom human herpesvirus is suspected as the causative organism, although with unconfirmed effects.3 Consultation with an infectious disease specialist is recommended before starting these agents, and biopsy is often needed beforehand.3

Immunosuppressive agents

Immunosuppressive agents such as prednisone, azathioprine, and cyclosporine can be used in cases of biopsy-proven disease with manifestations of severe heart failure, especially if biopsy results reveal sarcoidosis, giant cell myocarditis, or necrotizing eosinophilic myocarditis. Although the results were neutral in the Myocarditis Treatment Trial,29 the cause of myocarditis in this trial was unknown. Therapy with such agents should be initiated after active infection is ruled out, which also would require a biopsy.

Colchicine

Mechanisms of chest pain in myocarditis include associated pericarditis and coronary artery vasospasm.3,23 Our patient’s chest pain changed when he changed position, possibly indicating associated pericarditis. In myocarditis with accompanying pericarditis symptoms, colchicine (1–2 mg as an initial dose and then 0.6 mg daily for up to 3 months) can be helpful in alleviating symptoms.21,30 Thus, starting this agent in a patient who presents with myocarditis in absence of heart failure, arrhythmias, or left ventricular dysfunction is prudent.

Colchicine is used mainly to address the pain associated with pericarditis. For patients who present with pericarditis without myocarditis, nonsteroidal anti-inflammatory drugs (NSAIDs) remain the first-line treatment, with the addition of colchicine leading to faster symptom resolution.30 The benefit of colchicine for isolated myocarditis is not well established, with only limited data showing some clinical effects.31

 

 

CASE CONTINUED

The patient was given colchicine 1.2 mg on the first day and then 0.6 mg daily. Within 2 days, his chest pain had resolved. He did not receive any immunosuppressive agents.

DISCHARGE INSTRUCTIONS

4. Before discharge, this patient should be instructed to do which of the following?

  • Take over-the-counter NSAIDs to supplement the effects of colchicine
  • Avoid competitive sports and athletics for at least 6 months
  • Call to schedule repeat cardiac MRI
  • No further instruction is needed

NSAIDs are used by themselves or in combination with colchicine in the treatment of pericarditis, but their use may be associated with worse outcomes in myocarditis.3,21 Thus, their use is not recommended in most cases.3

Excessive physical activity should be avoided for at least 6 months after the clinical syndrome resolves. This recommendation is included in the most recent ESC guidelines but is based mainly on expert opinion and murine models with coxsackievirus B.3 Periodic reassessment is indicated with exercise stress testing before return to strenuous activity.3,16,32 Testing should look for exercise tolerance, and exercise electrocardiography also helps to evaluate for clinically relevant arrythmias.

Cardiac MRI can help clarify the prognosis in myocarditis, but the role of repeat testing in guiding therapy is limited.3 Indications for repeat cardiac MRI include presence of 0 or 1 of the Lake Louise criteria (recall that 2 are necessary to make the diagnosis) with recurrence of symptoms and a high suspicion for myocardial inflammation.3,9 Repeat cardiac MRI was not performed for our patient.

CASE CONCLUDED

The patient was evaluated in the cardiology clinic within 1 week of discharge. At that time, he was in sinus tachycardia with a heart rate of 102 bpm, and he was instructed to avoid any exercise until further notice.

At 6-month follow-up, the sinus tachycardia had resolved. However, because persistent tachycardia had been noted at the first postdischarge visit, and in view of the extent of myocardial involvement, he underwent exercise treadmill testing to evaluate for ventricular arrhythmias. The study did show premature ventricular complexes and 1 ventricular couplet at submaximal exercise levels. As this indicated a higher risk of exercise-induced arrhythmias, he was asked to continue normal activity levels but to abstain from exercise until the next evaluation.

During his 1-year follow-up, a repeat treadmill test showed no ventricular ectopy. Holter monitoring was ordered and showed no premature ventricular complexes, supraventricular arrhythmias, or atrioventricular block within the 48-hour period.

At his 2-year evaluation, he had returned to playing basketball and soccer on weekends and reported no recurrence of his initial symptoms.

KEY POINTS

  • farid_myocarditis_f3.jpg
    %3Cp%3EFigure%203.%20Our%20suggested%20approach%20to%20suspected%20acute%20myocarditis.%3C%2Fp%3E
    Cardiac MRI has emerged as an excellent noninvasive imaging modality for the diagnosis of myocarditis.
  • Treatment of myocarditis depends on the cause and severity of the patient’s presentation, spanning the spectrum from conservative care to immunosuppressive agents and even heart failure therapy.
  • Excessive physical activity should be avoided for the first 6 months after disease diagnosis and treatment.
  • If myocarditis is associated with pericardial involvement, colchicine is the agent of choice, and NSAIDs should be avoided.

Our suggested strategy for approaching myocarditis is shown in Figure 3.

References
  1. Dennert R, Crijns HJ, Heymans S. Acute viral myocarditis. Eur Heart J 2008; 29(17):2073–2082. doi:10.1093/eurheartj/ehn296
  2. Mahfoud F, Gärtner B, Kindermann M, et al. Virus serology in patients with suspected myocarditis: utility or futility? Eur Heart J 2011; 32(7):897–903. doi:10.1093/eurheartj/ehq493
  3. Caforio AL, Pankuweit S, Arbustini E, et al; European Society of Cardiology Working Group on Myocardial and Pericardial Diseases. Current state of knowledge on aetiology, diagnosis, management, and therapy of myocarditis: a position statement of the European Society of Cardiology Working Group on Myocardial and Pericardial Diseases. Eur Heart J 2013; 34(33):2636–2648, 2648a–2648d. doi:10.1093/eurheartj/eht210
  4. Donnelly JP, Hanna M. Cardiac amyloidosis: an update on diagnosis and treatment. Cleve Clin J Med 2017; 84(12 suppl 3):12–26. doi:10.3949/ccjm.84.s3.02
  5. Siddiqi OK, Ruberg FL. Cardiac amyloidosis: an update on pathophysiology, diagnosis, and treatment. Trends Cardiovasc Med 2018; 28(1):10–21. doi:10.1016/j.tcm.2017.07.004
  6. Gertz MA, Benson MD, Dyck PJ, et al. Diagnosis, prognosis, and therapy of transthyretin amyloidosis. J Am Coll Cardiol 2015; 66(21):2451–2466. doi:10.1016/j.jacc.2015.09.075
  7. Blankstein R, Osborne M, Naya M, et al. Cardiac positron emission tomography enhances prognostic assessments of patients with suspected cardiac sarcoidosis. J Am Coll Cardiol 2014; 63(4):329–336. doi:10.1016/j.jacc.2013.09.022
  8. Baccouche H, Mahrholtz H, Meinhardt G, et al. Diagnostic synergy of non-invasive cardiovascular magnetic resonance and invasive endomyocardial biopsy in troponin-positive patients without coronary artery disease. Eur Heart J 2009; 30(23):2869–2879. doi:10.1093/eurheartj/ehp328
  9. Friedrich MG, Sechtem U, Schulz-Menger J, et al; International Consensus Group on Cardiovascular Magnetic Resonance in Myocarditis. Cardiovascular magnetic resonance in myocarditis: a JACC white paper. J Am Coll Cardiol 2009; 53(17):1475–1487. doi:10.1016/j.jacc.2009.02.007
  10. Kindermann I, Barth C, Mahfoud F, et al. Update on myocarditis. J Am Coll Cardiol 2012; 59(9):779–792. doi:10.1016/j.jacc.2011.09.074
  11. Mahrholdt H, Wagner A, Deluigi CC, et al. Presentation, patterns of myocardial damage, and clinical course of viral myocarditis. Circulation 2006; 114(15):1581–1590. doi:10.1161/CIRCULATIONAHA.105.606509
  12. Gräni C, Eichhorn C, Bière L, et al. Prognostic value of cardiac magnetic resonance tissue characterization in risk stratifying patients with suspected myocarditis. J Am Coll Cardiol 2017; 70(16):1964–1976. doi:10.1016/j.jacc.2017.08.050
  13. Lurz P, Luecke C, Eitel I, et al. Comprehensive cardiac magnetic resonance imaging in patients with suspected myocarditis: the MyoRacer-Trial. J Am Coll Cardiol 2016; 67(15):1800–1811. doi:10.1016/j.jacc.2016.02.013
  14. Gannon MP, Schaub E, Griens CL, Saba SG. State of the art: evaluation and prognostication of myocarditis using cardiac MRI. J Magn Reson Imaging 2019; 49(7):e122–e131. doi:10.1002/jmri.26611
  15. Cooper LT, Baughman KL, Feldman AM, et al. The role of endomyocardial biopsy in the management of cardiovascular disease: a scientific statement from the American Heart Association, the American College of Cardiology, and the European Society of Cardiology endorsed by the Heart Failure Society of America and the Heart Failure Association of the European Society of Cardiology. Eur Heart J 2007; 28(24):3076–3093. doi:10.1093/eurheartj/ehm456
  16. Sinagra G, Anzini M, Pereira NL, et al. Myocarditis in clinical practice. Mayo Clin Proc 2016; 91(9):1256–1266. doi:10.1016/j.mayocp.2016.05.013
  17. Cooper LT, Baughman KL, Feldman AM, et al; American Heart Association; American College of Cardiology; European Society of Cardiology. The role of endomyocardial biopsy in the management of cardiovascular disease: a scientific statement from the American Heart Association, the American College of Cardiology, and the European Society of Cardiology. Circulation 2007; 116(19):2216–2233. doi:10.1161/CIRCULATIONAHA.107.186093
  18. Leone O, Veinot JP, Angelini A, et al. 2011 consensus statement on endomyocardial biopsy from the Association for European Cardiovascular Pathology and the Society for Cardiovascular Pathology. Cardiovasc Pathol 2012; 21(4):245–274. doi:10.1016/j.carpath.2011.10.001
  19. Baughman KL. Diagnosis of myocarditis: death of Dallas criteria. Circulation 2006; 113(4):593–595. doi:10.1161/CIRCULATIONAHA.105.589663
  20. Alraies MC, Klein AL. Should we still use electrocardiography to diagnose pericardial disease? Cleve Clin J Med 2013; 80(2):97–100. doi:10.3949/ccjm.80a.11144
  21. Sagar S, Liu PP, Cooper LT Jr. Myocarditis. Lancet 2012; 379(9817):738–747. doi:10.1016/S0140-6736(11)60648-X
  22. Caforio AL, Marcolongo R, Basso C, Iliceto S. Clinical presentation and diagnosis of myocarditis. Heart 2015; 101(16):1332–1344. doi:10.1136/heartjnl-2014-306363
  23. Cooper LT Jr. Myocarditis. N Engl J Med 2009; 360(15):1526–1538. doi:10.1056/NEJMra0800028
  24. LeLeiko RM, Bower DJ, Larsen CP. MRSA-associated bacterial myocarditis causing ruptured ventricle and tamponade. Cardiology 2008; 111(3):188–190. doi:10.1159/000121602
  25. Wasi F, Shuter J. Primary bacterial infection of the myocardium. Front Biosci 2003; 8:s228–s231. pmid:12700039
  26. Al-Amoodi M, Rao K, Rao S, Brewer JH, Magalski A, Chhatriwalla AK. Fulminant myocarditis due to H1N1 influenza. Circ Heart Fail 2010; 3(3):e7–e9. doi:10.1161/CIRCHEARTFAILURE.110.938506
  27. Yancy CW, Jessup M, Bozkurt B, et al. 2016 ACC/AHA/HFSA focused update on new pharmacological therapy for heart failure: an update of the 2013 ACCF/AHA guideline for the management of heart failure: a report of the American College of Cardiology/American Heart Association Task Force on Clinical Practice Guidelines and the Heart Failure Society of America. J Am Coll Cardiol 2016; 68(13):1476–1488. doi:10.1016/j.jacc.2016.05.011
  28. Schmidt-Lucke C, Spillmann F, Bock T, et al. Interferon beta modulates endothelial damage in patients with cardiac persistence of human parvovirus b19 infection. J Infect Dis 2010; 201(6):936–945. doi:10.1086/650700
  29. Mason JW, O’Connell JB, Herskowitz A, et al. A clinical trial of immunosuppressive therapy for myocarditis: the Myocarditis Treatment Trial Investigators. N Engl J Med 1995; 333(5):269–275. doi:10.1056/NEJM199508033330501
  30. Imazio M, Bobbio M, Cecchi E, et al. Colchicine in addition to conventional therapy for acute pericarditis: results of the COlchicine for acute PEricarditis (COPE) trial. Circulation 2005; 112(13):2012–2016. doi:10.1161/CIRCULATIONAHA.105.542738
  31. Morgenstern D, Lisko J, Boniface NC, Mikolich BM, Mikolich JR. Myocarditis and colchicine: a new perspective from cardiac MRI. J Cardiovasc Magn Reson 2016; 18(suppl 1):0100.
  32. Maron BJ, Zipes DP, Kovacs RJ. Eligibility and disqualification recommendations for competitive athletes with cardiovascular abnormalities: preamble, principles, and general considerations: a scientific statement from the American Heart Association and American College of Cardiology. J Am Coll Cardiol 2015; 66(21):2343–2349. doi:10.1016/j.jacc.2015.09.032
Article PDF
Author and Disclosure Information

Amir Farid, MD
Department of Cardiology, University of California Davis Medical Center, Sacramento

Neil Beri, MD
Department of Cardiology, University of California Davis Medical Center, Sacramento

David Torres-Barba, MD, PhD
Department of Cardiology, University of California San Diego

Charles Whitcomb, MD
Department of Cardiology, University of California Davis Medical Center, Sacramento

Address: David Torres-Barba, MD, PhD, Department of Internal Medicine, University of California, Davis, 4150 V. Street, Sacramento, CA 95817; davidtorresbarba@gmail.com

Issue
Cleveland Clinic Journal of Medicine - 86(9)
Publications
Topics
Page Number
586-594
Legacy Keywords
chest pain, angina, myocarditis, pericarditis, ST-segment elevation, serologic testing, light chain, myocardial perfusion, magnetic resonance imaging, MRI, biopsy, amyloidosis, sarcoidosis, parvovirus B19, colchicine, Amir Farid, Neil Beri, David Torres-Barba, Charles Whitcomb
Sections
Author and Disclosure Information

Amir Farid, MD
Department of Cardiology, University of California Davis Medical Center, Sacramento

Neil Beri, MD
Department of Cardiology, University of California Davis Medical Center, Sacramento

David Torres-Barba, MD, PhD
Department of Cardiology, University of California San Diego

Charles Whitcomb, MD
Department of Cardiology, University of California Davis Medical Center, Sacramento

Address: David Torres-Barba, MD, PhD, Department of Internal Medicine, University of California, Davis, 4150 V. Street, Sacramento, CA 95817; davidtorresbarba@gmail.com

Author and Disclosure Information

Amir Farid, MD
Department of Cardiology, University of California Davis Medical Center, Sacramento

Neil Beri, MD
Department of Cardiology, University of California Davis Medical Center, Sacramento

David Torres-Barba, MD, PhD
Department of Cardiology, University of California San Diego

Charles Whitcomb, MD
Department of Cardiology, University of California Davis Medical Center, Sacramento

Address: David Torres-Barba, MD, PhD, Department of Internal Medicine, University of California, Davis, 4150 V. Street, Sacramento, CA 95817; davidtorresbarba@gmail.com

Article PDF
Article PDF
Related Articles

An 18-year-old man without any significant medical history was transferred from another hospital for higher-level care after presenting with unremitting chest pain. He had been in his usual state of good health until 7 days before presentation, when he developed mild rhinorrhea and a sore throat, but not a cough. He went to an outpatient clinic, where a rapid test for group A streptococci was done; the result was negative, and he was sent home on supportive measures.

On the day of admission, he awoke with severe, pressure-like, midsternal, nonradiating pain, which he rated 10 on a scale of 10. The pain intensified in the supine position and improved with sitting. A complete review of systems was otherwise negative. He denied having had similar symptoms in the past, as well as sick contacts, recent travel, toxin exposure, illicit substance abuse, pets at home, or tick bites. His family history was negative for cardiac arrhythmias, premature coronary artery disease, thoracic aneurysms or dissection, and infiltrative disorders. His surgical and social histories were unremarkable. He said he had no drug allergies.

farid_myocarditis_f1.jpg
%3Cp%3EFigure%201.%20The%20patient%E2%80%99s%20electrocardiogram%20on%20presentation%20shows%20ST-segment%20elevation%20(arrows)%20over%20the%20lateral%20and%20inferior%20distribution%20(V%3Csub%3E4%3C%2Fsub%3E%E2%80%93V%3Csub%3E6%3C%2Fsub%3E%2C%20II%2C%20III%2C%20and%20aVF).%3C%2Fp%3E
An electrocardiogram was obtained (Figure 1). His troponin I level was 7.0 ng/mL (reference range < 0.04 ng/mL).

On examination, his temperature was 38.1°C (100.6°F), heart rate 101 beats per minute, blood pressure 142/78 mm Hg, respiratory rate 16 breaths per minute, and oxygen saturation 98% on room air. He appeared anxious but was in no acute distress. Neck examination showed no elevation in jugular venous pulsation, bruits, thyromegaly, or lymphadenopathy. Cardiac examination revealed tachycardia without murmurs, rubs, or gallops. Lungs were clear to auscultation. Examination of all 4 extremities found 2+ pulses (on a scale of 0 to 4+) throughout and no cyanosis, clubbing, or edema. Abdominal, neurologic, and dermatologic examinations were unremarkable.

Further blood testing revealed the following:

  • Troponin I (3 hours after the first level) 15.5 ng/mL
  • B-type natriuretic peptide 200 mg/dL (reference range 0–100 mg/dL)
  • C-reactive protein 0.9 mg/dL (reference range 0.0–0.8 mg/dL)
  • Erythrocyte sedimentation rate 10 mm/h (reference range < 15 mm/h).

Metabolic and hematologic assessments were unremarkable. A toxicology screen for drugs of abuse was negative. Viral serologic testing was not done.

A chest radiograph showed no acute cardiopulmonary processes.

Given his presenting symptoms, persistent tachycardia, rapidly rising troponin I level, and electrocardiogram showing diffuse ST elevation, he was taken for urgent cardiac catheterization. Coronary angiography revealed no evidence of atherosclerotic disease, acute thrombosis, dissection, or aneurysm. Echocardiography 2 hours after the procedure showed a normal ejection fraction and no regional wall-motion abnormalities or valvular heart disease.

 

 

FURTHER TESTING

1. Which test should be done next to further evaluate this patient’s chest pain?

  • Serum viral serologic testing
  • Serum free light chain assay
  • Nuclear myocardial perfusion study
  • Cardiac magnetic resonance imaging (MRI)
  • Endomyocardial biopsy

In this patient without ischemic coronary disease or valvular heart disease, the recent upper respiratory tract prodrome, active positional chest pain, and diffuse electrocardiographic changes raise the possibility of myocarditis with pericardial involvement.

Viral serologic tests

Viral serologic tests are often obtained in the workup of myocarditis as a noninvasive means of detecting an infectious cause.

However, this approach has several problems. First, a positive serologic result is a signal of the peripheral immune response to a pathogen but does not necessarily indicate active myocardial inflammation. Additionally, circulating immunoglobulin G against cardiotropic viruses is commonly found, even in the absence of myocarditis.1 This is often the result of a high prevalence and exposure to these viruses in the general population. Further, trials have shown no correlation between serologic results and organisms identified by endomyocardial biopsy.2

Thus, serologic testing seems to be of limited utility, reserved for testing for infection with Borrelia burgdorferi (Lyme disease) in endemic areas, hepatitis C virus, human immunodeficiency virus in patients at high risk, Rickettsia conorii, and Rickettsia rickettsii.3

Serum free light chain testing for amyloidosis

Serum free light chain testing is replacing serum and urine protein electrophoresis in the workup of cardiac amyloidosis,4 as electrophoresis has poor sensitivity.4,5

Cardiac amyloidosis often affects older persons, although in rare cases it can affect young patients who carry mutations in the transthyretin gene (ATTR amyloidosis).6 This diagnosis is unlikely in our patient, as he has no other affected organ systems (amyloidosis often affects the renal and neurologic systems), normal QRS voltages on electrocardiography (which are often but not always low in amyloidosis), and no left ventricular hypertrophy or diastolic dysfunction on echocardiography (which are often seen in amyloidosis).4

Nuclear perfusion imaging for sarcoidosis

Nuclear imaging has a limited role in evaluating myocarditis,3 but positron-emission tomography with fluorine-18 fluorodeoxyglucose has a diagnostic role in sarcoidosis, an immune-mediated cause of myocarditis.7

Based on the acuity of the patient’s presentation, preceded by upper respiratory tract symptoms, sarcoidosis is less likely. Sarcoidosis is difficult to diagnose, although when it is the cause of myocarditis, some clues exist, as patients usually present with heart failure symptoms, a second- or third-degree atrioventricular block, or a dilated left ventricle on echocardiography.3 All of these were absent in our patient.

Cardiac MRI

Cardiac MRI has undergone many advances, making it an extremely useful noninvasive test. It has excellent utility as a stand-alone test in diagnosing myocarditis and has synergistic value when combined with endomyocardial biopsy.8 It is indicated in hemodynamically stable patients with a clinical suspicion of myocarditis, persistent symptoms, absence of heart failure, and when imaging findings will change management. It is particularly useful to help elucidate a cause and guide tailored therapy.9 Therefore, it is a reasonable next step in the diagnostic pathway for this patient.10

Cardiac MRI also allows for concurrent assessment of scar. In myocardial infarction, the late gadolinium enhancement is subendocardial or transmural. In myocarditis, the pattern differs, being found in the subepicardial lateral free wall (in most patients with parvovirus B19) and mid-myocardial septum (in most patients with herpesvirus 6).9,11 Cardiac MRI also confers prognostic information for patients with suspected myocarditis.12

The Lake Louise criteria9 for the diagnosis of myocarditis require 2 of the following:

  • Evidence of myocardial edema
  • Increased ratio of early gadolinium enhancement between myocardium and skeletal muscle (indicates hyperemia)
  • At least 1 focal lesion with nonischemic late gadolinium enhancement (indicates cardiac myocyte injury or scarring).

The Lake Louise criteria may be replaced by T1 and T2 mapping, which was found to be considerably better for diagnosing myocarditis when the 2 were compared.9,13,14

Endomyocardial biopsy

Endomyocardial biopsy should not be delayed while waiting for cardiac MRI in patients who are hemodynamically unstable or present with life-threatening features (ventricular arrhythmia, left ventricular failure, or resuscitation after sudden cardiac death).3,10

The indications for endomyocardial biopsy have been highly debated. The 2013 guidelines from the European Society of Cardiology (ESC) recommending endomyocardial biopsy  in all clinically suspected cases of myocarditis have only heightened the controversy.3 The American Heart Association (AHA) guidelines reserve biopsy for patients with suspected myocarditis who have acute or subacute heart failure symptoms or who do not respond to standard medical therapy.15 Other reasonable indications may include the following: myocarditis with life-threatening ventricular arrhythmias, suspicion of giant cell myocarditis, necrotizing eosinophilic myocarditis, or cardiac sarcoidosis.16

Endomyocardial biopsy is the only way to make a definitive diagnosis of myocarditis.3 However, given the patchy distribution of myocardial involvement, a negative result does not rule out myocarditis. The diagnostic utility can be improved by increasing the number of samples taken (at least 3 but up to 10), obtaining samples from both ventricles, and using cardiac MRI data to determine which sites to biopsy.3,13,17,18

Noninvasive testing such as cardiac MRI does not distinguish cell type or etiology (viral vs nonviral).3 Further, endomyocardial biopsy must be performed before immunosuppressive therapy can be safely started.3,16 At experienced centers, the complication rate is 0% to 0.8%.3 The addition of immunohistochemical testing and viral genomic detection by polymerase chain reaction testing have increased the sensitivity of this technique.19 Finally, endomyocardial biopsy can help rule out some of the other possibilities in the differential diagnosis for myocarditis, including infiltrative and storage diseases, and possibly cardiac tumors.3

Of additional note, the diffuse ST-segment elevation seen on the patient’s electrocardiogram (Figure 1) is indicative of subepicardial inflammation. Since the distribution involves more than one epicardial coronary territory, this helps to differentiate the changes from those that occur with myocardial infarction.20

 

 

CASE CONTINUED

farid_myocarditis_f2.jpg
%3Cp%3EFigure%202.%20Cardiac%20magnetic%20resonance%20imaging%20shows%20areas%20of%20patchy%20subepicardial%20late%20gadolinium%20enhancement%20(arrows).%3C%2Fp%3E
The patient underwent cardiac MRI, which showed myocardial edema and patchy areas of late gadolinium enhancement, raising suspicion for myocarditis (Figure 2).

Causes of myocarditis are numerous (Table 1),3,21,22 but viral and postinfectious etiologies remain the most common causes of acute myocarditis.23

farid_myocarditis_t1.jpg
2. What is the most likely causative infectious agent?

  • Parvovirus B19
  • Coxsackievirus B
  • Adenovirus species
  • Human herpesvirus 6
  • Staphylococcus aureus
  • Corynebacterium diphtheria
  • Trypanosoma cruzi
  • Influenza H1/N1

INFECTIOUS CAUSES OF MYOCARDITIS

Coxsackievirus B was the agent most often linked to this condition from the 1950s through the 1990s. However, in the last 2 decades, adenovirus species and human herpesvirus 6 have been increasingly encountered, and recently, parvovirus B19 has been credited as the most common culprit,11,23 at least in the Western world. In developing nations, T cruzi and C diphtheria are the most common offenders.21

S aureus is a common cause of endocarditis, but it rarely plays a role in myocarditis. When it does, the myocarditis is often the sequela of profound bacteremia. This was much more common before antibiotics were invented.24,25

Influenza H1/N1 is not among the most common causes of viral myocarditis, but it should be considered during flu season, given its ability to result in fulminant myocarditis.3,26

TREATMENT FOR MYOCARDITIS

3. Which treatment is the most appropriate at this time?

  • Intravenous immunoglobulin
  • Interferon beta
  • Acyclovir
  • Prednisone
  • Colchicine

Treatment for myocarditis depends on the cause but always includes supportive care to address the constellation of presenting symptoms. Standard therapies for tachy- or bradyarrhythmias, heart failure, and hemodynamic derangement should be started.

Supportive care

In patients with severe left ventricular dysfunction, an implantable cardiac electronic device, left ventricular assist device, or heart transplant may ultimately be needed. However, if possible these should be deferred for several months to determine response to treatment, since the myocardium can possibly recover.16

Diuretics, beta-blockers, angiotensin II receptor blockers, angiotensin-converting enzyme inhibitors, and aldosterone antagonists should be given as part of guideline-directed medical therapy for patients with heart failure and reduced ejection fraction.3,27 However, whether and how the patient should be weaned from these agents after disease recovery are unknown.3

Intravenous immunoglobulin

Intravenous immunoglobulin in high doses has had mixed results. Its efficacy is well documented in children,21 but limited supportive data are available in adults.3 As such, recent ESC guidelines do not provide recommendations regarding its use in adults.3

Interferon beta

Interferon beta has shown promise in improving New York Heart Association class and left ventricular ejection fraction.3 This is attributed to its effects on eliminating adenoviral species and enteroviruses. Treatment of enteroviral organisms in particular has been associated with improved 10-year prognosis.3 Interferon beta also has in vitro data showing efficacy at diminishing apoptosis from parvovirus B19.28

Nucleoside analogues

Empiric treatment with nucleoside analogues (acyclovir, ganciclovir, and valacyclovir) has been tried for patients in whom human herpesvirus is suspected as the causative organism, although with unconfirmed effects.3 Consultation with an infectious disease specialist is recommended before starting these agents, and biopsy is often needed beforehand.3

Immunosuppressive agents

Immunosuppressive agents such as prednisone, azathioprine, and cyclosporine can be used in cases of biopsy-proven disease with manifestations of severe heart failure, especially if biopsy results reveal sarcoidosis, giant cell myocarditis, or necrotizing eosinophilic myocarditis. Although the results were neutral in the Myocarditis Treatment Trial,29 the cause of myocarditis in this trial was unknown. Therapy with such agents should be initiated after active infection is ruled out, which also would require a biopsy.

Colchicine

Mechanisms of chest pain in myocarditis include associated pericarditis and coronary artery vasospasm.3,23 Our patient’s chest pain changed when he changed position, possibly indicating associated pericarditis. In myocarditis with accompanying pericarditis symptoms, colchicine (1–2 mg as an initial dose and then 0.6 mg daily for up to 3 months) can be helpful in alleviating symptoms.21,30 Thus, starting this agent in a patient who presents with myocarditis in absence of heart failure, arrhythmias, or left ventricular dysfunction is prudent.

Colchicine is used mainly to address the pain associated with pericarditis. For patients who present with pericarditis without myocarditis, nonsteroidal anti-inflammatory drugs (NSAIDs) remain the first-line treatment, with the addition of colchicine leading to faster symptom resolution.30 The benefit of colchicine for isolated myocarditis is not well established, with only limited data showing some clinical effects.31

 

 

CASE CONTINUED

The patient was given colchicine 1.2 mg on the first day and then 0.6 mg daily. Within 2 days, his chest pain had resolved. He did not receive any immunosuppressive agents.

DISCHARGE INSTRUCTIONS

4. Before discharge, this patient should be instructed to do which of the following?

  • Take over-the-counter NSAIDs to supplement the effects of colchicine
  • Avoid competitive sports and athletics for at least 6 months
  • Call to schedule repeat cardiac MRI
  • No further instruction is needed

NSAIDs are used by themselves or in combination with colchicine in the treatment of pericarditis, but their use may be associated with worse outcomes in myocarditis.3,21 Thus, their use is not recommended in most cases.3

Excessive physical activity should be avoided for at least 6 months after the clinical syndrome resolves. This recommendation is included in the most recent ESC guidelines but is based mainly on expert opinion and murine models with coxsackievirus B.3 Periodic reassessment is indicated with exercise stress testing before return to strenuous activity.3,16,32 Testing should look for exercise tolerance, and exercise electrocardiography also helps to evaluate for clinically relevant arrythmias.

Cardiac MRI can help clarify the prognosis in myocarditis, but the role of repeat testing in guiding therapy is limited.3 Indications for repeat cardiac MRI include presence of 0 or 1 of the Lake Louise criteria (recall that 2 are necessary to make the diagnosis) with recurrence of symptoms and a high suspicion for myocardial inflammation.3,9 Repeat cardiac MRI was not performed for our patient.

CASE CONCLUDED

The patient was evaluated in the cardiology clinic within 1 week of discharge. At that time, he was in sinus tachycardia with a heart rate of 102 bpm, and he was instructed to avoid any exercise until further notice.

At 6-month follow-up, the sinus tachycardia had resolved. However, because persistent tachycardia had been noted at the first postdischarge visit, and in view of the extent of myocardial involvement, he underwent exercise treadmill testing to evaluate for ventricular arrhythmias. The study did show premature ventricular complexes and 1 ventricular couplet at submaximal exercise levels. As this indicated a higher risk of exercise-induced arrhythmias, he was asked to continue normal activity levels but to abstain from exercise until the next evaluation.

During his 1-year follow-up, a repeat treadmill test showed no ventricular ectopy. Holter monitoring was ordered and showed no premature ventricular complexes, supraventricular arrhythmias, or atrioventricular block within the 48-hour period.

At his 2-year evaluation, he had returned to playing basketball and soccer on weekends and reported no recurrence of his initial symptoms.

KEY POINTS

  • farid_myocarditis_f3.jpg
    %3Cp%3EFigure%203.%20Our%20suggested%20approach%20to%20suspected%20acute%20myocarditis.%3C%2Fp%3E
    Cardiac MRI has emerged as an excellent noninvasive imaging modality for the diagnosis of myocarditis.
  • Treatment of myocarditis depends on the cause and severity of the patient’s presentation, spanning the spectrum from conservative care to immunosuppressive agents and even heart failure therapy.
  • Excessive physical activity should be avoided for the first 6 months after disease diagnosis and treatment.
  • If myocarditis is associated with pericardial involvement, colchicine is the agent of choice, and NSAIDs should be avoided.

Our suggested strategy for approaching myocarditis is shown in Figure 3.

An 18-year-old man without any significant medical history was transferred from another hospital for higher-level care after presenting with unremitting chest pain. He had been in his usual state of good health until 7 days before presentation, when he developed mild rhinorrhea and a sore throat, but not a cough. He went to an outpatient clinic, where a rapid test for group A streptococci was done; the result was negative, and he was sent home on supportive measures.

On the day of admission, he awoke with severe, pressure-like, midsternal, nonradiating pain, which he rated 10 on a scale of 10. The pain intensified in the supine position and improved with sitting. A complete review of systems was otherwise negative. He denied having had similar symptoms in the past, as well as sick contacts, recent travel, toxin exposure, illicit substance abuse, pets at home, or tick bites. His family history was negative for cardiac arrhythmias, premature coronary artery disease, thoracic aneurysms or dissection, and infiltrative disorders. His surgical and social histories were unremarkable. He said he had no drug allergies.

farid_myocarditis_f1.jpg
%3Cp%3EFigure%201.%20The%20patient%E2%80%99s%20electrocardiogram%20on%20presentation%20shows%20ST-segment%20elevation%20(arrows)%20over%20the%20lateral%20and%20inferior%20distribution%20(V%3Csub%3E4%3C%2Fsub%3E%E2%80%93V%3Csub%3E6%3C%2Fsub%3E%2C%20II%2C%20III%2C%20and%20aVF).%3C%2Fp%3E
An electrocardiogram was obtained (Figure 1). His troponin I level was 7.0 ng/mL (reference range < 0.04 ng/mL).

On examination, his temperature was 38.1°C (100.6°F), heart rate 101 beats per minute, blood pressure 142/78 mm Hg, respiratory rate 16 breaths per minute, and oxygen saturation 98% on room air. He appeared anxious but was in no acute distress. Neck examination showed no elevation in jugular venous pulsation, bruits, thyromegaly, or lymphadenopathy. Cardiac examination revealed tachycardia without murmurs, rubs, or gallops. Lungs were clear to auscultation. Examination of all 4 extremities found 2+ pulses (on a scale of 0 to 4+) throughout and no cyanosis, clubbing, or edema. Abdominal, neurologic, and dermatologic examinations were unremarkable.

Further blood testing revealed the following:

  • Troponin I (3 hours after the first level) 15.5 ng/mL
  • B-type natriuretic peptide 200 mg/dL (reference range 0–100 mg/dL)
  • C-reactive protein 0.9 mg/dL (reference range 0.0–0.8 mg/dL)
  • Erythrocyte sedimentation rate 10 mm/h (reference range < 15 mm/h).

Metabolic and hematologic assessments were unremarkable. A toxicology screen for drugs of abuse was negative. Viral serologic testing was not done.

A chest radiograph showed no acute cardiopulmonary processes.

Given his presenting symptoms, persistent tachycardia, rapidly rising troponin I level, and electrocardiogram showing diffuse ST elevation, he was taken for urgent cardiac catheterization. Coronary angiography revealed no evidence of atherosclerotic disease, acute thrombosis, dissection, or aneurysm. Echocardiography 2 hours after the procedure showed a normal ejection fraction and no regional wall-motion abnormalities or valvular heart disease.

 

 

FURTHER TESTING

1. Which test should be done next to further evaluate this patient’s chest pain?

  • Serum viral serologic testing
  • Serum free light chain assay
  • Nuclear myocardial perfusion study
  • Cardiac magnetic resonance imaging (MRI)
  • Endomyocardial biopsy

In this patient without ischemic coronary disease or valvular heart disease, the recent upper respiratory tract prodrome, active positional chest pain, and diffuse electrocardiographic changes raise the possibility of myocarditis with pericardial involvement.

Viral serologic tests

Viral serologic tests are often obtained in the workup of myocarditis as a noninvasive means of detecting an infectious cause.

However, this approach has several problems. First, a positive serologic result is a signal of the peripheral immune response to a pathogen but does not necessarily indicate active myocardial inflammation. Additionally, circulating immunoglobulin G against cardiotropic viruses is commonly found, even in the absence of myocarditis.1 This is often the result of a high prevalence and exposure to these viruses in the general population. Further, trials have shown no correlation between serologic results and organisms identified by endomyocardial biopsy.2

Thus, serologic testing seems to be of limited utility, reserved for testing for infection with Borrelia burgdorferi (Lyme disease) in endemic areas, hepatitis C virus, human immunodeficiency virus in patients at high risk, Rickettsia conorii, and Rickettsia rickettsii.3

Serum free light chain testing for amyloidosis

Serum free light chain testing is replacing serum and urine protein electrophoresis in the workup of cardiac amyloidosis,4 as electrophoresis has poor sensitivity.4,5

Cardiac amyloidosis often affects older persons, although in rare cases it can affect young patients who carry mutations in the transthyretin gene (ATTR amyloidosis).6 This diagnosis is unlikely in our patient, as he has no other affected organ systems (amyloidosis often affects the renal and neurologic systems), normal QRS voltages on electrocardiography (which are often but not always low in amyloidosis), and no left ventricular hypertrophy or diastolic dysfunction on echocardiography (which are often seen in amyloidosis).4

Nuclear perfusion imaging for sarcoidosis

Nuclear imaging has a limited role in evaluating myocarditis,3 but positron-emission tomography with fluorine-18 fluorodeoxyglucose has a diagnostic role in sarcoidosis, an immune-mediated cause of myocarditis.7

Based on the acuity of the patient’s presentation, preceded by upper respiratory tract symptoms, sarcoidosis is less likely. Sarcoidosis is difficult to diagnose, although when it is the cause of myocarditis, some clues exist, as patients usually present with heart failure symptoms, a second- or third-degree atrioventricular block, or a dilated left ventricle on echocardiography.3 All of these were absent in our patient.

Cardiac MRI

Cardiac MRI has undergone many advances, making it an extremely useful noninvasive test. It has excellent utility as a stand-alone test in diagnosing myocarditis and has synergistic value when combined with endomyocardial biopsy.8 It is indicated in hemodynamically stable patients with a clinical suspicion of myocarditis, persistent symptoms, absence of heart failure, and when imaging findings will change management. It is particularly useful to help elucidate a cause and guide tailored therapy.9 Therefore, it is a reasonable next step in the diagnostic pathway for this patient.10

Cardiac MRI also allows for concurrent assessment of scar. In myocardial infarction, the late gadolinium enhancement is subendocardial or transmural. In myocarditis, the pattern differs, being found in the subepicardial lateral free wall (in most patients with parvovirus B19) and mid-myocardial septum (in most patients with herpesvirus 6).9,11 Cardiac MRI also confers prognostic information for patients with suspected myocarditis.12

The Lake Louise criteria9 for the diagnosis of myocarditis require 2 of the following:

  • Evidence of myocardial edema
  • Increased ratio of early gadolinium enhancement between myocardium and skeletal muscle (indicates hyperemia)
  • At least 1 focal lesion with nonischemic late gadolinium enhancement (indicates cardiac myocyte injury or scarring).

The Lake Louise criteria may be replaced by T1 and T2 mapping, which was found to be considerably better for diagnosing myocarditis when the 2 were compared.9,13,14

Endomyocardial biopsy

Endomyocardial biopsy should not be delayed while waiting for cardiac MRI in patients who are hemodynamically unstable or present with life-threatening features (ventricular arrhythmia, left ventricular failure, or resuscitation after sudden cardiac death).3,10

The indications for endomyocardial biopsy have been highly debated. The 2013 guidelines from the European Society of Cardiology (ESC) recommending endomyocardial biopsy  in all clinically suspected cases of myocarditis have only heightened the controversy.3 The American Heart Association (AHA) guidelines reserve biopsy for patients with suspected myocarditis who have acute or subacute heart failure symptoms or who do not respond to standard medical therapy.15 Other reasonable indications may include the following: myocarditis with life-threatening ventricular arrhythmias, suspicion of giant cell myocarditis, necrotizing eosinophilic myocarditis, or cardiac sarcoidosis.16

Endomyocardial biopsy is the only way to make a definitive diagnosis of myocarditis.3 However, given the patchy distribution of myocardial involvement, a negative result does not rule out myocarditis. The diagnostic utility can be improved by increasing the number of samples taken (at least 3 but up to 10), obtaining samples from both ventricles, and using cardiac MRI data to determine which sites to biopsy.3,13,17,18

Noninvasive testing such as cardiac MRI does not distinguish cell type or etiology (viral vs nonviral).3 Further, endomyocardial biopsy must be performed before immunosuppressive therapy can be safely started.3,16 At experienced centers, the complication rate is 0% to 0.8%.3 The addition of immunohistochemical testing and viral genomic detection by polymerase chain reaction testing have increased the sensitivity of this technique.19 Finally, endomyocardial biopsy can help rule out some of the other possibilities in the differential diagnosis for myocarditis, including infiltrative and storage diseases, and possibly cardiac tumors.3

Of additional note, the diffuse ST-segment elevation seen on the patient’s electrocardiogram (Figure 1) is indicative of subepicardial inflammation. Since the distribution involves more than one epicardial coronary territory, this helps to differentiate the changes from those that occur with myocardial infarction.20

 

 

CASE CONTINUED

farid_myocarditis_f2.jpg
%3Cp%3EFigure%202.%20Cardiac%20magnetic%20resonance%20imaging%20shows%20areas%20of%20patchy%20subepicardial%20late%20gadolinium%20enhancement%20(arrows).%3C%2Fp%3E
The patient underwent cardiac MRI, which showed myocardial edema and patchy areas of late gadolinium enhancement, raising suspicion for myocarditis (Figure 2).

Causes of myocarditis are numerous (Table 1),3,21,22 but viral and postinfectious etiologies remain the most common causes of acute myocarditis.23

farid_myocarditis_t1.jpg
2. What is the most likely causative infectious agent?

  • Parvovirus B19
  • Coxsackievirus B
  • Adenovirus species
  • Human herpesvirus 6
  • Staphylococcus aureus
  • Corynebacterium diphtheria
  • Trypanosoma cruzi
  • Influenza H1/N1

INFECTIOUS CAUSES OF MYOCARDITIS

Coxsackievirus B was the agent most often linked to this condition from the 1950s through the 1990s. However, in the last 2 decades, adenovirus species and human herpesvirus 6 have been increasingly encountered, and recently, parvovirus B19 has been credited as the most common culprit,11,23 at least in the Western world. In developing nations, T cruzi and C diphtheria are the most common offenders.21

S aureus is a common cause of endocarditis, but it rarely plays a role in myocarditis. When it does, the myocarditis is often the sequela of profound bacteremia. This was much more common before antibiotics were invented.24,25

Influenza H1/N1 is not among the most common causes of viral myocarditis, but it should be considered during flu season, given its ability to result in fulminant myocarditis.3,26

TREATMENT FOR MYOCARDITIS

3. Which treatment is the most appropriate at this time?

  • Intravenous immunoglobulin
  • Interferon beta
  • Acyclovir
  • Prednisone
  • Colchicine

Treatment for myocarditis depends on the cause but always includes supportive care to address the constellation of presenting symptoms. Standard therapies for tachy- or bradyarrhythmias, heart failure, and hemodynamic derangement should be started.

Supportive care

In patients with severe left ventricular dysfunction, an implantable cardiac electronic device, left ventricular assist device, or heart transplant may ultimately be needed. However, if possible these should be deferred for several months to determine response to treatment, since the myocardium can possibly recover.16

Diuretics, beta-blockers, angiotensin II receptor blockers, angiotensin-converting enzyme inhibitors, and aldosterone antagonists should be given as part of guideline-directed medical therapy for patients with heart failure and reduced ejection fraction.3,27 However, whether and how the patient should be weaned from these agents after disease recovery are unknown.3

Intravenous immunoglobulin

Intravenous immunoglobulin in high doses has had mixed results. Its efficacy is well documented in children,21 but limited supportive data are available in adults.3 As such, recent ESC guidelines do not provide recommendations regarding its use in adults.3

Interferon beta

Interferon beta has shown promise in improving New York Heart Association class and left ventricular ejection fraction.3 This is attributed to its effects on eliminating adenoviral species and enteroviruses. Treatment of enteroviral organisms in particular has been associated with improved 10-year prognosis.3 Interferon beta also has in vitro data showing efficacy at diminishing apoptosis from parvovirus B19.28

Nucleoside analogues

Empiric treatment with nucleoside analogues (acyclovir, ganciclovir, and valacyclovir) has been tried for patients in whom human herpesvirus is suspected as the causative organism, although with unconfirmed effects.3 Consultation with an infectious disease specialist is recommended before starting these agents, and biopsy is often needed beforehand.3

Immunosuppressive agents

Immunosuppressive agents such as prednisone, azathioprine, and cyclosporine can be used in cases of biopsy-proven disease with manifestations of severe heart failure, especially if biopsy results reveal sarcoidosis, giant cell myocarditis, or necrotizing eosinophilic myocarditis. Although the results were neutral in the Myocarditis Treatment Trial,29 the cause of myocarditis in this trial was unknown. Therapy with such agents should be initiated after active infection is ruled out, which also would require a biopsy.

Colchicine

Mechanisms of chest pain in myocarditis include associated pericarditis and coronary artery vasospasm.3,23 Our patient’s chest pain changed when he changed position, possibly indicating associated pericarditis. In myocarditis with accompanying pericarditis symptoms, colchicine (1–2 mg as an initial dose and then 0.6 mg daily for up to 3 months) can be helpful in alleviating symptoms.21,30 Thus, starting this agent in a patient who presents with myocarditis in absence of heart failure, arrhythmias, or left ventricular dysfunction is prudent.

Colchicine is used mainly to address the pain associated with pericarditis. For patients who present with pericarditis without myocarditis, nonsteroidal anti-inflammatory drugs (NSAIDs) remain the first-line treatment, with the addition of colchicine leading to faster symptom resolution.30 The benefit of colchicine for isolated myocarditis is not well established, with only limited data showing some clinical effects.31

 

 

CASE CONTINUED

The patient was given colchicine 1.2 mg on the first day and then 0.6 mg daily. Within 2 days, his chest pain had resolved. He did not receive any immunosuppressive agents.

DISCHARGE INSTRUCTIONS

4. Before discharge, this patient should be instructed to do which of the following?

  • Take over-the-counter NSAIDs to supplement the effects of colchicine
  • Avoid competitive sports and athletics for at least 6 months
  • Call to schedule repeat cardiac MRI
  • No further instruction is needed

NSAIDs are used by themselves or in combination with colchicine in the treatment of pericarditis, but their use may be associated with worse outcomes in myocarditis.3,21 Thus, their use is not recommended in most cases.3

Excessive physical activity should be avoided for at least 6 months after the clinical syndrome resolves. This recommendation is included in the most recent ESC guidelines but is based mainly on expert opinion and murine models with coxsackievirus B.3 Periodic reassessment is indicated with exercise stress testing before return to strenuous activity.3,16,32 Testing should look for exercise tolerance, and exercise electrocardiography also helps to evaluate for clinically relevant arrythmias.

Cardiac MRI can help clarify the prognosis in myocarditis, but the role of repeat testing in guiding therapy is limited.3 Indications for repeat cardiac MRI include presence of 0 or 1 of the Lake Louise criteria (recall that 2 are necessary to make the diagnosis) with recurrence of symptoms and a high suspicion for myocardial inflammation.3,9 Repeat cardiac MRI was not performed for our patient.

CASE CONCLUDED

The patient was evaluated in the cardiology clinic within 1 week of discharge. At that time, he was in sinus tachycardia with a heart rate of 102 bpm, and he was instructed to avoid any exercise until further notice.

At 6-month follow-up, the sinus tachycardia had resolved. However, because persistent tachycardia had been noted at the first postdischarge visit, and in view of the extent of myocardial involvement, he underwent exercise treadmill testing to evaluate for ventricular arrhythmias. The study did show premature ventricular complexes and 1 ventricular couplet at submaximal exercise levels. As this indicated a higher risk of exercise-induced arrhythmias, he was asked to continue normal activity levels but to abstain from exercise until the next evaluation.

During his 1-year follow-up, a repeat treadmill test showed no ventricular ectopy. Holter monitoring was ordered and showed no premature ventricular complexes, supraventricular arrhythmias, or atrioventricular block within the 48-hour period.

At his 2-year evaluation, he had returned to playing basketball and soccer on weekends and reported no recurrence of his initial symptoms.

KEY POINTS

  • farid_myocarditis_f3.jpg
    %3Cp%3EFigure%203.%20Our%20suggested%20approach%20to%20suspected%20acute%20myocarditis.%3C%2Fp%3E
    Cardiac MRI has emerged as an excellent noninvasive imaging modality for the diagnosis of myocarditis.
  • Treatment of myocarditis depends on the cause and severity of the patient’s presentation, spanning the spectrum from conservative care to immunosuppressive agents and even heart failure therapy.
  • Excessive physical activity should be avoided for the first 6 months after disease diagnosis and treatment.
  • If myocarditis is associated with pericardial involvement, colchicine is the agent of choice, and NSAIDs should be avoided.

Our suggested strategy for approaching myocarditis is shown in Figure 3.

References
  1. Dennert R, Crijns HJ, Heymans S. Acute viral myocarditis. Eur Heart J 2008; 29(17):2073–2082. doi:10.1093/eurheartj/ehn296
  2. Mahfoud F, Gärtner B, Kindermann M, et al. Virus serology in patients with suspected myocarditis: utility or futility? Eur Heart J 2011; 32(7):897–903. doi:10.1093/eurheartj/ehq493
  3. Caforio AL, Pankuweit S, Arbustini E, et al; European Society of Cardiology Working Group on Myocardial and Pericardial Diseases. Current state of knowledge on aetiology, diagnosis, management, and therapy of myocarditis: a position statement of the European Society of Cardiology Working Group on Myocardial and Pericardial Diseases. Eur Heart J 2013; 34(33):2636–2648, 2648a–2648d. doi:10.1093/eurheartj/eht210
  4. Donnelly JP, Hanna M. Cardiac amyloidosis: an update on diagnosis and treatment. Cleve Clin J Med 2017; 84(12 suppl 3):12–26. doi:10.3949/ccjm.84.s3.02
  5. Siddiqi OK, Ruberg FL. Cardiac amyloidosis: an update on pathophysiology, diagnosis, and treatment. Trends Cardiovasc Med 2018; 28(1):10–21. doi:10.1016/j.tcm.2017.07.004
  6. Gertz MA, Benson MD, Dyck PJ, et al. Diagnosis, prognosis, and therapy of transthyretin amyloidosis. J Am Coll Cardiol 2015; 66(21):2451–2466. doi:10.1016/j.jacc.2015.09.075
  7. Blankstein R, Osborne M, Naya M, et al. Cardiac positron emission tomography enhances prognostic assessments of patients with suspected cardiac sarcoidosis. J Am Coll Cardiol 2014; 63(4):329–336. doi:10.1016/j.jacc.2013.09.022
  8. Baccouche H, Mahrholtz H, Meinhardt G, et al. Diagnostic synergy of non-invasive cardiovascular magnetic resonance and invasive endomyocardial biopsy in troponin-positive patients without coronary artery disease. Eur Heart J 2009; 30(23):2869–2879. doi:10.1093/eurheartj/ehp328
  9. Friedrich MG, Sechtem U, Schulz-Menger J, et al; International Consensus Group on Cardiovascular Magnetic Resonance in Myocarditis. Cardiovascular magnetic resonance in myocarditis: a JACC white paper. J Am Coll Cardiol 2009; 53(17):1475–1487. doi:10.1016/j.jacc.2009.02.007
  10. Kindermann I, Barth C, Mahfoud F, et al. Update on myocarditis. J Am Coll Cardiol 2012; 59(9):779–792. doi:10.1016/j.jacc.2011.09.074
  11. Mahrholdt H, Wagner A, Deluigi CC, et al. Presentation, patterns of myocardial damage, and clinical course of viral myocarditis. Circulation 2006; 114(15):1581–1590. doi:10.1161/CIRCULATIONAHA.105.606509
  12. Gräni C, Eichhorn C, Bière L, et al. Prognostic value of cardiac magnetic resonance tissue characterization in risk stratifying patients with suspected myocarditis. J Am Coll Cardiol 2017; 70(16):1964–1976. doi:10.1016/j.jacc.2017.08.050
  13. Lurz P, Luecke C, Eitel I, et al. Comprehensive cardiac magnetic resonance imaging in patients with suspected myocarditis: the MyoRacer-Trial. J Am Coll Cardiol 2016; 67(15):1800–1811. doi:10.1016/j.jacc.2016.02.013
  14. Gannon MP, Schaub E, Griens CL, Saba SG. State of the art: evaluation and prognostication of myocarditis using cardiac MRI. J Magn Reson Imaging 2019; 49(7):e122–e131. doi:10.1002/jmri.26611
  15. Cooper LT, Baughman KL, Feldman AM, et al. The role of endomyocardial biopsy in the management of cardiovascular disease: a scientific statement from the American Heart Association, the American College of Cardiology, and the European Society of Cardiology endorsed by the Heart Failure Society of America and the Heart Failure Association of the European Society of Cardiology. Eur Heart J 2007; 28(24):3076–3093. doi:10.1093/eurheartj/ehm456
  16. Sinagra G, Anzini M, Pereira NL, et al. Myocarditis in clinical practice. Mayo Clin Proc 2016; 91(9):1256–1266. doi:10.1016/j.mayocp.2016.05.013
  17. Cooper LT, Baughman KL, Feldman AM, et al; American Heart Association; American College of Cardiology; European Society of Cardiology. The role of endomyocardial biopsy in the management of cardiovascular disease: a scientific statement from the American Heart Association, the American College of Cardiology, and the European Society of Cardiology. Circulation 2007; 116(19):2216–2233. doi:10.1161/CIRCULATIONAHA.107.186093
  18. Leone O, Veinot JP, Angelini A, et al. 2011 consensus statement on endomyocardial biopsy from the Association for European Cardiovascular Pathology and the Society for Cardiovascular Pathology. Cardiovasc Pathol 2012; 21(4):245–274. doi:10.1016/j.carpath.2011.10.001
  19. Baughman KL. Diagnosis of myocarditis: death of Dallas criteria. Circulation 2006; 113(4):593–595. doi:10.1161/CIRCULATIONAHA.105.589663
  20. Alraies MC, Klein AL. Should we still use electrocardiography to diagnose pericardial disease? Cleve Clin J Med 2013; 80(2):97–100. doi:10.3949/ccjm.80a.11144
  21. Sagar S, Liu PP, Cooper LT Jr. Myocarditis. Lancet 2012; 379(9817):738–747. doi:10.1016/S0140-6736(11)60648-X
  22. Caforio AL, Marcolongo R, Basso C, Iliceto S. Clinical presentation and diagnosis of myocarditis. Heart 2015; 101(16):1332–1344. doi:10.1136/heartjnl-2014-306363
  23. Cooper LT Jr. Myocarditis. N Engl J Med 2009; 360(15):1526–1538. doi:10.1056/NEJMra0800028
  24. LeLeiko RM, Bower DJ, Larsen CP. MRSA-associated bacterial myocarditis causing ruptured ventricle and tamponade. Cardiology 2008; 111(3):188–190. doi:10.1159/000121602
  25. Wasi F, Shuter J. Primary bacterial infection of the myocardium. Front Biosci 2003; 8:s228–s231. pmid:12700039
  26. Al-Amoodi M, Rao K, Rao S, Brewer JH, Magalski A, Chhatriwalla AK. Fulminant myocarditis due to H1N1 influenza. Circ Heart Fail 2010; 3(3):e7–e9. doi:10.1161/CIRCHEARTFAILURE.110.938506
  27. Yancy CW, Jessup M, Bozkurt B, et al. 2016 ACC/AHA/HFSA focused update on new pharmacological therapy for heart failure: an update of the 2013 ACCF/AHA guideline for the management of heart failure: a report of the American College of Cardiology/American Heart Association Task Force on Clinical Practice Guidelines and the Heart Failure Society of America. J Am Coll Cardiol 2016; 68(13):1476–1488. doi:10.1016/j.jacc.2016.05.011
  28. Schmidt-Lucke C, Spillmann F, Bock T, et al. Interferon beta modulates endothelial damage in patients with cardiac persistence of human parvovirus b19 infection. J Infect Dis 2010; 201(6):936–945. doi:10.1086/650700
  29. Mason JW, O’Connell JB, Herskowitz A, et al. A clinical trial of immunosuppressive therapy for myocarditis: the Myocarditis Treatment Trial Investigators. N Engl J Med 1995; 333(5):269–275. doi:10.1056/NEJM199508033330501
  30. Imazio M, Bobbio M, Cecchi E, et al. Colchicine in addition to conventional therapy for acute pericarditis: results of the COlchicine for acute PEricarditis (COPE) trial. Circulation 2005; 112(13):2012–2016. doi:10.1161/CIRCULATIONAHA.105.542738
  31. Morgenstern D, Lisko J, Boniface NC, Mikolich BM, Mikolich JR. Myocarditis and colchicine: a new perspective from cardiac MRI. J Cardiovasc Magn Reson 2016; 18(suppl 1):0100.
  32. Maron BJ, Zipes DP, Kovacs RJ. Eligibility and disqualification recommendations for competitive athletes with cardiovascular abnormalities: preamble, principles, and general considerations: a scientific statement from the American Heart Association and American College of Cardiology. J Am Coll Cardiol 2015; 66(21):2343–2349. doi:10.1016/j.jacc.2015.09.032
References
  1. Dennert R, Crijns HJ, Heymans S. Acute viral myocarditis. Eur Heart J 2008; 29(17):2073–2082. doi:10.1093/eurheartj/ehn296
  2. Mahfoud F, Gärtner B, Kindermann M, et al. Virus serology in patients with suspected myocarditis: utility or futility? Eur Heart J 2011; 32(7):897–903. doi:10.1093/eurheartj/ehq493
  3. Caforio AL, Pankuweit S, Arbustini E, et al; European Society of Cardiology Working Group on Myocardial and Pericardial Diseases. Current state of knowledge on aetiology, diagnosis, management, and therapy of myocarditis: a position statement of the European Society of Cardiology Working Group on Myocardial and Pericardial Diseases. Eur Heart J 2013; 34(33):2636–2648, 2648a–2648d. doi:10.1093/eurheartj/eht210
  4. Donnelly JP, Hanna M. Cardiac amyloidosis: an update on diagnosis and treatment. Cleve Clin J Med 2017; 84(12 suppl 3):12–26. doi:10.3949/ccjm.84.s3.02
  5. Siddiqi OK, Ruberg FL. Cardiac amyloidosis: an update on pathophysiology, diagnosis, and treatment. Trends Cardiovasc Med 2018; 28(1):10–21. doi:10.1016/j.tcm.2017.07.004
  6. Gertz MA, Benson MD, Dyck PJ, et al. Diagnosis, prognosis, and therapy of transthyretin amyloidosis. J Am Coll Cardiol 2015; 66(21):2451–2466. doi:10.1016/j.jacc.2015.09.075
  7. Blankstein R, Osborne M, Naya M, et al. Cardiac positron emission tomography enhances prognostic assessments of patients with suspected cardiac sarcoidosis. J Am Coll Cardiol 2014; 63(4):329–336. doi:10.1016/j.jacc.2013.09.022
  8. Baccouche H, Mahrholtz H, Meinhardt G, et al. Diagnostic synergy of non-invasive cardiovascular magnetic resonance and invasive endomyocardial biopsy in troponin-positive patients without coronary artery disease. Eur Heart J 2009; 30(23):2869–2879. doi:10.1093/eurheartj/ehp328
  9. Friedrich MG, Sechtem U, Schulz-Menger J, et al; International Consensus Group on Cardiovascular Magnetic Resonance in Myocarditis. Cardiovascular magnetic resonance in myocarditis: a JACC white paper. J Am Coll Cardiol 2009; 53(17):1475–1487. doi:10.1016/j.jacc.2009.02.007
  10. Kindermann I, Barth C, Mahfoud F, et al. Update on myocarditis. J Am Coll Cardiol 2012; 59(9):779–792. doi:10.1016/j.jacc.2011.09.074
  11. Mahrholdt H, Wagner A, Deluigi CC, et al. Presentation, patterns of myocardial damage, and clinical course of viral myocarditis. Circulation 2006; 114(15):1581–1590. doi:10.1161/CIRCULATIONAHA.105.606509
  12. Gräni C, Eichhorn C, Bière L, et al. Prognostic value of cardiac magnetic resonance tissue characterization in risk stratifying patients with suspected myocarditis. J Am Coll Cardiol 2017; 70(16):1964–1976. doi:10.1016/j.jacc.2017.08.050
  13. Lurz P, Luecke C, Eitel I, et al. Comprehensive cardiac magnetic resonance imaging in patients with suspected myocarditis: the MyoRacer-Trial. J Am Coll Cardiol 2016; 67(15):1800–1811. doi:10.1016/j.jacc.2016.02.013
  14. Gannon MP, Schaub E, Griens CL, Saba SG. State of the art: evaluation and prognostication of myocarditis using cardiac MRI. J Magn Reson Imaging 2019; 49(7):e122–e131. doi:10.1002/jmri.26611
  15. Cooper LT, Baughman KL, Feldman AM, et al. The role of endomyocardial biopsy in the management of cardiovascular disease: a scientific statement from the American Heart Association, the American College of Cardiology, and the European Society of Cardiology endorsed by the Heart Failure Society of America and the Heart Failure Association of the European Society of Cardiology. Eur Heart J 2007; 28(24):3076–3093. doi:10.1093/eurheartj/ehm456
  16. Sinagra G, Anzini M, Pereira NL, et al. Myocarditis in clinical practice. Mayo Clin Proc 2016; 91(9):1256–1266. doi:10.1016/j.mayocp.2016.05.013
  17. Cooper LT, Baughman KL, Feldman AM, et al; American Heart Association; American College of Cardiology; European Society of Cardiology. The role of endomyocardial biopsy in the management of cardiovascular disease: a scientific statement from the American Heart Association, the American College of Cardiology, and the European Society of Cardiology. Circulation 2007; 116(19):2216–2233. doi:10.1161/CIRCULATIONAHA.107.186093
  18. Leone O, Veinot JP, Angelini A, et al. 2011 consensus statement on endomyocardial biopsy from the Association for European Cardiovascular Pathology and the Society for Cardiovascular Pathology. Cardiovasc Pathol 2012; 21(4):245–274. doi:10.1016/j.carpath.2011.10.001
  19. Baughman KL. Diagnosis of myocarditis: death of Dallas criteria. Circulation 2006; 113(4):593–595. doi:10.1161/CIRCULATIONAHA.105.589663
  20. Alraies MC, Klein AL. Should we still use electrocardiography to diagnose pericardial disease? Cleve Clin J Med 2013; 80(2):97–100. doi:10.3949/ccjm.80a.11144
  21. Sagar S, Liu PP, Cooper LT Jr. Myocarditis. Lancet 2012; 379(9817):738–747. doi:10.1016/S0140-6736(11)60648-X
  22. Caforio AL, Marcolongo R, Basso C, Iliceto S. Clinical presentation and diagnosis of myocarditis. Heart 2015; 101(16):1332–1344. doi:10.1136/heartjnl-2014-306363
  23. Cooper LT Jr. Myocarditis. N Engl J Med 2009; 360(15):1526–1538. doi:10.1056/NEJMra0800028
  24. LeLeiko RM, Bower DJ, Larsen CP. MRSA-associated bacterial myocarditis causing ruptured ventricle and tamponade. Cardiology 2008; 111(3):188–190. doi:10.1159/000121602
  25. Wasi F, Shuter J. Primary bacterial infection of the myocardium. Front Biosci 2003; 8:s228–s231. pmid:12700039
  26. Al-Amoodi M, Rao K, Rao S, Brewer JH, Magalski A, Chhatriwalla AK. Fulminant myocarditis due to H1N1 influenza. Circ Heart Fail 2010; 3(3):e7–e9. doi:10.1161/CIRCHEARTFAILURE.110.938506
  27. Yancy CW, Jessup M, Bozkurt B, et al. 2016 ACC/AHA/HFSA focused update on new pharmacological therapy for heart failure: an update of the 2013 ACCF/AHA guideline for the management of heart failure: a report of the American College of Cardiology/American Heart Association Task Force on Clinical Practice Guidelines and the Heart Failure Society of America. J Am Coll Cardiol 2016; 68(13):1476–1488. doi:10.1016/j.jacc.2016.05.011
  28. Schmidt-Lucke C, Spillmann F, Bock T, et al. Interferon beta modulates endothelial damage in patients with cardiac persistence of human parvovirus b19 infection. J Infect Dis 2010; 201(6):936–945. doi:10.1086/650700
  29. Mason JW, O’Connell JB, Herskowitz A, et al. A clinical trial of immunosuppressive therapy for myocarditis: the Myocarditis Treatment Trial Investigators. N Engl J Med 1995; 333(5):269–275. doi:10.1056/NEJM199508033330501
  30. Imazio M, Bobbio M, Cecchi E, et al. Colchicine in addition to conventional therapy for acute pericarditis: results of the COlchicine for acute PEricarditis (COPE) trial. Circulation 2005; 112(13):2012–2016. doi:10.1161/CIRCULATIONAHA.105.542738
  31. Morgenstern D, Lisko J, Boniface NC, Mikolich BM, Mikolich JR. Myocarditis and colchicine: a new perspective from cardiac MRI. J Cardiovasc Magn Reson 2016; 18(suppl 1):0100.
  32. Maron BJ, Zipes DP, Kovacs RJ. Eligibility and disqualification recommendations for competitive athletes with cardiovascular abnormalities: preamble, principles, and general considerations: a scientific statement from the American Heart Association and American College of Cardiology. J Am Coll Cardiol 2015; 66(21):2343–2349. doi:10.1016/j.jacc.2015.09.032
Issue
Cleveland Clinic Journal of Medicine - 86(9)
Issue
Cleveland Clinic Journal of Medicine - 86(9)
Page Number
586-594
Page Number
586-594
Publications
Publications
Topics
Article Type
Display Headline
A young man with acute chest pain
Display Headline
A young man with acute chest pain
Legacy Keywords
chest pain, angina, myocarditis, pericarditis, ST-segment elevation, serologic testing, light chain, myocardial perfusion, magnetic resonance imaging, MRI, biopsy, amyloidosis, sarcoidosis, parvovirus B19, colchicine, Amir Farid, Neil Beri, David Torres-Barba, Charles Whitcomb
Legacy Keywords
chest pain, angina, myocarditis, pericarditis, ST-segment elevation, serologic testing, light chain, myocardial perfusion, magnetic resonance imaging, MRI, biopsy, amyloidosis, sarcoidosis, parvovirus B19, colchicine, Amir Farid, Neil Beri, David Torres-Barba, Charles Whitcomb
Sections
Disallow All Ads
Content Gating
No Gating (article Unlocked/Free)
Alternative CME
Disqus Comments
Default
Gate On Date
Thu, 08/22/2019 - 09:15
Un-Gate On Date
Thu, 08/22/2019 - 09:15
Use ProPublica
CFC Schedule Remove Status
Thu, 08/22/2019 - 09:15
Hide sidebar & use full width
render the right sidebar.
Article PDF Media

An unusual cause of bruising

Article Type
Changed
Thu, 08/01/2019 - 07:55
Display Headline
An unusual cause of bruising

A 61-year-old woman presented to our hematology clinic for evaluation of multiple episodes of bruising. The first episode occurred 8 months earlier, when she developed a large bruise after water skiing. Two months before coming to us, she went to her local emergency room because of new bruising and was found to have a prolonged activated partial thromboplastin time (aPTT) of 60 seconds (reference range 23.3–34.9), but she underwent no further testing at that time.

At presentation to our clinic, she reported having no fevers, night sweats, unintentional weight loss, swollen lymph nodes, joint pain, rashes, mouth sores, nosebleeds, or blood in the urine or stool. Her history was notable only for hypothyroidism, which was diagnosed in the previous year. Her medications included levothyroxine, vitamin D3, and vitamin C. She had been taking a baby aspirin daily for the past 10 years but had stopped 1 month earlier because of the bruising.

atieh_bruising_t1.jpg
On examination, she had a single small hematoma on her right thigh. She had no ecchymoses, petechiae, or adenopathy, and her spleen was nonpalpable.

Ten years earlier she had been evaluated for a possible transient ischemic attack; laboratory results at that time included a normal aPTT of 25.1 seconds and a normal factor VIII level of 153% (reference range 50%–173%).

atieh_bruising_t2.jpg
Laboratory testing at our clinic showed a normal complete blood cell count (Table 1); the coagulation factor assay confirmed that her aPTT was elevated (prolonged), but other values were normal (Table 2).

atieh_bruising_t3.jpg
Causes of an isolated prolonged aPTT include medications (eg, heparin), inherited factor deficiencies, acquired inhibitors of coagulation factors, and inherited or acquired von Willebrand disease. Lupus anticoagulant can prolong the aPTT but is usually associated with thrombosis rather than bleeding.1 The differential diagnoses for this and other patterns of coagulation assay abnormalities are listed in Table 3.

EVALUATION FOR AN ISOLATED PROLONGED aPTT

1. What is the appropriate next test to evaluate this patient’s prolonged aPTT?

  • Lupus anticoagulant panel
  • Coagulation factor levels
  • Mixing studies
  • Bethesda assay

Mixing studies

Once a prolonged aPTT is confirmed, the appropriate next step is a mixing study. This involves mixing the patient’s plasma with pooled normal plasma in a 1-to-1 ratio, then repeating the aPTT test immediately, and again after 1 hour of incubation at 37°C. If the patient does not have enough of one of the coagulation factors, the aPTT immediately returns to the normal range when plasma is mixed with the pooled plasma because the pooled plasma contains the factor that is lacking. If this happens, then factor assays should be performed to identify the deficient factor.1

Various antibodies that inhibit coagulation factors can also affect the aPTT. There are 2 general types: immediate-acting and delayed.

With an immediate-acting inhibitor, the aPTT does not correct into the normal range with initial mixing. Immediate-acting inhibitors are often seen together with lupus anticoagulants, which are nonspecific phospholipid antibodies. If an immediate-acting inhibitor is detected, further testing should focus on evaluation for lupus anticoagulant, including phospholipid-dependency studies.

With a delayed inhibitor, the aPTT initially comes down, but subsequently goes back up after incubation. Acquired factor VIII inhibitor is a classic delayed-type inhibitor and is also the most common factor inhibitor.1 If a delayed-acting inhibitor is found, specific intrinsic factor levels should be measured (factors VIII, IX, XI, and XII),2 and testing should also be done for lupus anticoagulant, as these inhibitors may occur together.

Bethesda assay

atieh_bruising_t4.jpg
If factor levels are decreased, a Bethesda assay should be performed to differentiate a specific factor inhibitor from a lupus anticoagulant. In the case of a factor VIII inhibitor, serial dilutions of patient plasma are incubated at 37°C with pooled normal plasma for 2 hours, then factor VIII activity is measured. The reciprocal dilution of patient plasma that results in 50% of factor VIII activity in the control plasma is 1 Bethesda unit (BU). The stronger the inhibitor in the patient’s sample, the more dilutions are required to measure factor VIII activity, and thus the higher the Bethesda titer.3

Case continued: Results of mixing and Bethesda studies

atieh_bruising_t5.jpg
Results of the mixing study (Table 4) showed an initial correction of the aPTT in a 1-to-1 mix, but after 1 hour of incubation, the aPTT was again prolonged at 42 seconds (reference range < 37.3). Further testing revealed very low levels of factor VIII (< 1%), and the presence of a factor VIII inhibitor, quantified at 5.8 BU (reference range < 0.5) (Table 5). Additional coagulation tests, including von Willebrand factor testing and a lupus anticoagulation panel, were negative.

 

 

FACTOR VIII INHIBITOR EVALUATION

2. What is the most likely underlying condition associated with this patient’s factor VIII inhibitor?

  • Autoimmune disease
  • Malignancy
  • A medication
  • Unknown (idiopathic)

Acquired hemophilia A (AHA) is a rare disorder caused by autoantibodies against factor VIII. Its estimated incidence is about 1 person per million per year.4 It usually presents as unexplained bruising or bleeding and is only rarely diagnosed by an incidentally noted prolonged aPTT. The severity of bleeding is variable and can include subcutaneous, soft-tissue, retroperitoneal, gastrointestinal, and intracranial hemorrhage.5

AHA is considered idiopathic in more than half of cases. A study based on a European registry5 of 501 patients with AHA and a UK study6 of 172 patients found no underlying disease in 52% and 65% of patients, respectively. For patients with an identified cause, the most common causes were malignancy (12%5 and 15%6) and autoimmune disease (12%5 and 17%6).

Drugs have rarely been associated with factor VIII inhibitors. Such occurrences have been reported with interferon, blood thinners, antibiotics, and psychiatric medications, but no study yet has indicated causation. However, patients with congenital hemophilia A treated with factor VIII preparations have about a 15% chance of developing factor VIII inhibitors. In this setting, inhibitors develop in response to recombinant factor VIII exposure, unlike the autoimmune phenomena seen in AHA.

TREATMENT OF ACQUIRED HEMOPHILIA A

3. What is the most appropriate treatment for AHA?

  • Desmopressin and prednisone
  • Recombinant porcine factor VIII and prednisone plus cyclophosphamide
  • Recombinant factor VIIa and rituximab
  • Any of the above

Any of the above regimens can be used. In general, treatment of AHA has two purposes: to stop acute hemorrhage, and to reduce the level of factor VIII inhibitor. No standard treatment guidelines are available; evidence of the effectiveness of different drugs is based largely on data on congenital hemophilia A.3

Acute treatment to stop bleeding

Initial treatment of AHA often focuses on stopping an acute hemorrhage by either raising circulating levels of factor VIII or bypassing it in the coagulation cascade.

Desmopressin can temporarily raise factor VIII levels, but it is often ineffective in AHA unless the patient has very low inhibitor titers.3

Factor VIII concentrate (human or recombinant porcine factor VIII) may be effective in patients with low inhibitor titers (< 5 BU). Higher doses are often required than those used in congenital hemophilia A. Factor VIII concentrate is usually combined with immunosuppressive treatment to lower the factor VIII inhibitor level (described below).3

If these methods are ineffective or the patient has high inhibitor titers (> 5 BU), activated prothrombin complex concentrates, known as FEIBA (factor eight inhibitor bypassing activity), or recombinant factor VIIa is available. These agents bypass factor VIII in the clotting cascade.

Immunosuppression to reduce factor VIII inhibitor

Immunosuppressive agents are the mainstay of AHA treatment to lower the inhibitor level.

Regimens vary. A 2003 meta-analysis4 including 249 patients found that prednisone alone resulted in complete response in about 30% of patients, and the addition of cyclophosphamide increased the response rate to 60% to 100%. High-dose intravenous immunoglobulin led to conflicting results. Conclusions were limited by the variability of dosing and duration in treatment regimens among the 20 different studies included.

An analysis of 331 patients in the European Acquired Hemophilia Registry (EACH2)7 found that steroids alone produced remission in 48% of patients, while steroids combined with cyclophosphamide raised the rate to 70%. Rituximab-based regimens were successful in 59% but required twice as long to achieve remission as steroid or cyclophosphamide-based regimens. No benefit was noted from intravenous immunoglobulin.

Risks of disease and treatment

AHA is associated with significant risk of morbidity and death related to bleeding, complications of treatment, and underlying disease.

In EACH2, 16 of the 331 patients died of bleeding, 16 died of causes related to immunosuppression, and 45 died of causes related to the underlying condition.5 In the UK registry of 172 patients, 13 patients died of bleeding, and 12 died of sepsis related to immunosuppression.6

The factor VIII level and inhibitor titer are not necessarily useful in stratifying bleeding risk, as severe and fatal bleeding can occur at variable levels and patients remain at risk of bleeding as long as the inhibitor persists.6,7

 

 

CASE CONTINUED: TREATMENT, LYMPHOCYTOSIS

The patient was started on 60 mg daily of prednisone, resulting in a decrease in her aPTT, increase in factor VIII level, and lower Bethesda titer. On a return visit, her absolute lymphocyte count was 7.04 × 109/L (reference range 1.0–4.0). She reported no fevers, chills, or recent infections.  

EVALUATING LYMPHOCYTOSIS

Lymphocytosis is defined in most laboratories as an absolute lymphocyte count greater than 4.0 × 109/L for adults. Normally, T cells (CD3+) make up 60% to 80% of lymphocytes, B cells (CD20+) 10% to 20%, and natural killer (NK) cells (CD3–, CD56+) 5% to 10%. Lymphocytosis is usually caused by infection, but it can have other causes, including malignancy.

Peripheral blood smear. If there is no clear cause of lymphocytosis, a peripheral blood smear can be used to assess lymphocyte morphology, providing clues to the underlying etiology. For example, atypical lymphocytes are often seen in infectious mononucleosis, while “smudge” lymphocytes are characteristic of chronic lymphocytic leukemia. If a peripheral smear shows abnormal morphology, further workup should include establishing whether the lymphocytes are polyclonal or clonal.8

CASE CONTINUED: LARGE GRANULAR LYMPHOCYTES

atieh_bruising_f1.jpg
%3Cp%3EFigure%201.%20A%20large%20granular%20lymphocyte.%3C%2Fp%3E
On the patient’s peripheral smear, 54% of lymphocytes were large lymphocytes with moderate amounts of pale cytoplasm filled with azurophilic granules, consistent with large granular lymphocytes (LGLs) (Figure 1).

4. What is the next step to evaluate the patient’s lymphocytosis?

  • Bone marrow biopsy
  • Karyotype analysis
  • Flow cytometry
  • Fluorescence in situ hybridization

Flow cytometry with V-beta analysis is the best first test to determine the cause of lymphocytosis after review of the peripheral smear. For persistent lymphocytosis, flow cytometry should be done even if a peripheral smear shows normal lymphocyte morphology.

Most T cells possess receptors composed of alpha and beta chains, each encoded by variable (V), diversity (D), joining (J), and constant (C) gene segments. The V, D, and J segments undergo rearrangement during T-cell development in the thymus based on antigen exposure, producing a diverse T-cell receptor population.

In a polyclonal population of lymphocytes, the T-cell receptors have a variety of gene segment arrangements, indicating normal T-cell development. But in a clonal population of lymphocytes, the T-cell receptors have a single identical gene segment arrangement, indicating they all originated from a single clone.9 Lymphocytosis in response to an infection is typically polyclonal, while malignant lymphocytosis is clonal. 

Monoclonal antibodies against many of the variable regions of the beta chain (V-beta) of T-cell receptors have been developed, enabling flow cytometry to establish clonality.

T-cell receptor gene rearrangement studies can also be performed using polymerase chain reaction and Southern blot techniques.9

Karyotype analysis is usually not performed for the finding of LGLs, because most leukemias (eg, T-cell and NK-cell leukemias) have cells with a normal karyotype. 

Bone marrow biopsy is invasive and usually not required to evaluate LGLs. It can be especially risky for a patient with a bleeding disorder such as a factor VIII inhibitor.10

Case continued: Flow cytometry confirms clonality

Subsequent flow cytometry found that more than 50% of the patient’s lymphocytes were LGLs that co-expressed CD3+, CD8+, CD56+, and CD57+, with aberrantly decreased CD7 expression. T-cell V-beta analysis demonstrated an expansion of the V-beta 17 family, and T-cell receptor gene analysis with polymerase chain reaction confirmed the presence of a clonal rearrangement.

LGL LEUKEMIA: CLASSIFICATION AND MANAGEMENT

LGLs normally account for 10% to 15% of peripheral mononuclear cells.11 LGL leukemia is caused by a clonal population of cytotoxic T cells or NK cells and involves an increased number of LGLs (usually > 2 × 109/L).10

LGL leukemia is divided into 3 categories according to the most recent World Health Organization classification10,12:

T-cell LGL leukemia (about 85% of cases) is considered indolent but can cause significant cytopenias and is often associated with autoimmune disease.13 Cells usually express a CD3+, CD8+, CD16+, and CD57+ phenotype. Survival is about 70% at 10 years.

Chronic NK-cell lymphocytosis (about 10%) also tends to have an indolent course with cytopenia and an autoimmune association, and with a similar prognosis to T-cell LGL leukemia. Cells express a CD3–, CD16+, and CD56+ phenotype.

Aggressive NK-cell LGL leukemia (about 5%) is associated with Epstein-Barr virus infection and occurs in younger patients. It is characterized by severe cytopenias, “B symptoms” (ie, fever, night sweats, weight loss), and has a very poor prognosis. Like chronic NK-cell lymphocytosis, cells express a CD3–, CD16+, and CD56+ phenotype. Fas (CD95) and Fas-ligand (CD178) are strongly expressed.10,13

Most cases of LGL leukemia can be diagnosed on the basis of classic morphology on peripheral blood smear and evidence of clonality on flow cytometry or gene rearrangement studies. T-cell receptor gene studies cannot be used to establish clonality in the NK subtypes, as NK cells do not express T-cell receptors.11

Case continued: Diagnosis, continued course 

In our patient, T-cell LGL leukemia was diagnosed on the basis of the peripheral smear, flow cytometry results, and positive T-cell receptor gene studies for clonal rearrangement in the T-cell receptor beta region.

While her corticosteroid therapy was being tapered, her factor III inhibitor level increased, and she had a small episode of bleeding, prompting the start of cyclophosphamide 50 mg daily with lower doses of prednisone.

atieh_bruising_f2.jpg
%3Cp%3EFigure%202.%20The%20patient%E2%80%99s%20clinical%20course%3A%20factor%20VIII%20inhibitor%20response%20to%20treatment.%3C%2Fp%3E
She then developed elevated liver enzymes, prompting discontinuation of cyclophosphamide. Rituximab was started and continued for 4 weekly doses, resulting in normalization of aPTT and factor VIII level with undetectable Bethesda titers (Figure 2).

 

 

LGL LEUKEMIA AND AUTOIMMUNE DISEASE

Patients with LGL leukemia commonly have or develop autoimmune conditions. Immune-mediated cytopenias including pure red cell aplasia, aplastic anemia, and autoimmune hemolytic anemias can occur. Neutropenia, the most common cytopenia in LGL leukemia, is thought to be at least partly autoimmune, as the degree of neutropenia is often worse than would be expected solely from bone-marrow infiltration of LGL cells.10,14,15

Rheumatoid arthritis is the most common autoimmune condition associated with LGL leukemia, with a reported incidence between 11% and 36%.13–15

Felty syndrome (rheumatoid arthritis, splenomegaly, and neutropenia) is often associated with LGL leukemia and is thought by some to be part of the same disease process.15

Treat with immunosuppressives if needed

Indications for treating LGL leukemia include the development of cytopenias and associated autoimmune diseases. Immunosuppressive agents, such as methotrexate, cyclophosphamide, and cyclosporine, are commonly used.10,11,14 Most evidence of treatment efficacy is from retrospective studies and case reports, with widely variable response rates that overall are around 50%.10

ACQUIRED HEMOPHILIA A AND HEMATOLOGIC MALIGNANCY

A systematic review found 30 cases of AHA associated with hematologic malignancies.16 The largest case series17 in this analysis had 8 patients, and included diagnoses of chronic lymphocytic leukemia, erythroleukemia, myelofibrosis, multiple myeloma, and myelodysplastic syndrome. In 3 of these patients, the appearance of the inhibitor preceded the diagnosis of the underlying malignancy by an average of 3.5 months. In 1 patient with erythroleukemia and another with multiple myeloma, the activity of the inhibitor could be clearly correlated with the underlying malignancy. In the other 6 patients, no association between the two could be made.

In the same series, complete resolution of the inhibitor was related only to the level of Bethesda titer present at diagnosis, with those who achieved resolution having lower mean Bethesda titers.17 Similarly, in EACH2, lower inhibitor Bethesda titers and higher factor VIII levels at presentation were associated with faster inhibitor eradication and normalization of factor VIII levels.7

Murphy et al18 described a 62-year-old woman with Felty syndrome who developed a factor VIII inhibitor and was subsequently given a diagnosis of LGL leukemia. Treatment with immunosuppressive agents, including cyclophosphamide, azathioprine, and rituximab, successfully eradicated her factor VIII inhibitor, although the LGL leukemia persisted.

Case conclusion: Eradication of factor VIII inhibitor

Our patient, similar to the patient described by Murphy et al18 above, had eradication of the factor VIII inhibitor despite persistence of LGL leukemia. Between the time of diagnosis at our clinic, when she had 54% LGLs, and eradication of the inhibitor 3 months later, the LGL percentage ranged from 45% to 89%. No clear direct correlation between LGL and factor VIII inhibitor levels could be detected.

Given the strong association of LGL leukemia with autoimmune disease, it is tempting to believe that her factor VIII inhibitor was somehow related to her malignancy, although the exact mechanism remained unclear. The average age at diagnosis is 60 for LGL leukemia11 and over 70 for AHA,5,6 so advanced age may be the common denominator. Whether or not our patient will have recurrence of her factor VIII inhibitor or the development of other autoimmune diseases with the persistence of her LGL leukemia remains to be seen.

At last follow-up, our patient was off all therapy and continued to have normal aPTT and factor VIII levels. Repeat flow cytometry after treatment of her factor VIII inhibitor showed persistence of a clonal T-cell population, although reduced from 72% to 60%. It may be that the 2 entities were unrelated, and the clonal T-cell population was simply fluctuating over time. This can be determined only with further observation. As the patient had no symptoms from her LGL leukemia, she continued to be observed without treatment.

TAKE-HOME POINTS

  • The coagulation assay is key to initially assessing a bleeding abnormality; whether the prothrombin time and aPTT are normal or prolonged narrows the differential diagnosis and determines next steps in evaluation.
  • Mixing studies can help pinpoint the responsible deficient factor.
  • Acquired factor VIII deficiency, also known as AHA, may be caused by autoimmune disease, malignancy, or medications, but it is usually idiopathic.
  • AHA treatment is focused on achieving hemostasis and reducing factor VIII inhibitor.
  • Lymphocytosis should be evaluated with a peripheral blood smear and flow cytometry to determine if the population is polyclonal (associated with infection) or clonal (associated with malignancy).
  • LGL leukemia is usually a chronic, indolent disease, although an uncommon subtype has an aggressive course.
  • The association between AHA and LGL leukemia is unclear, and both conditions must be monitored and managed.
References
  1. Kamal AH, Tefferi A, Pruthi RK. How to interpret and pursue an abnormal prothrombin time, activated partial thromboplastin time, and bleeding time in adults. Mayo Clin Proc 2007; 82(7):864–873. doi:10.4065/82.7.864
  2. Tcherniantchouk O, Laposata M, Marques MB. The isolated prolonged PTT. Am J Hematol 2013; 88(1):82–85. doi:10.1002/ajh.23285
  3. Ma AD, Carrizosa D. Acquired factor VIII inhibitors: pathophysiology and treatment. Hematology Am Soc Hematol Educ Program 2006:432–437. doi:10.1182/asheducation-2006.1.432
  4. Delgado J, Jimenez-Yuste V, Hernandez-Navarro F, Villar A. Acquired haemophilia: review and meta-analysis focused on therapy and prognostic factors. Br J Haematol 2003; 121(1):21–35. pmid:12670328
  5. Knoebl P, Marco P, Baudo F, et al; EACH2 Registry Contributors. Demographic and clinical data in acquired hemophilia A: results from the European Acquired Haemophilia Registry (EACH2). J Thromb Haemost 2012; 10(4):622–631. doi:10.1111/j.1538-7836.2012.04654.x
  6. Collins PW, Hirsch S, Baglin TP, et al; UK Haemophilia Centre Doctors’ Organisation. Acquired hemophilia A in the United Kingdom: a 2-year national surveillance study by the United Kingdom Haemophilia Centre Doctors’ Organisation. Blood 2007; 109(5):1870–1877. doi:10.1182/blood-2006-06-029850
  7. Collins P, Baudo F, Knoebl P, et al; EACH2 Registry Collaborators. Immunosuppression for acquired hemophilia A: results from the European Acquired Haemophilia Registry (EACH2). Blood 2012; 120(1):47–55. doi:10.1182/blood-2012-02-409185
  8. George TI. Malignant or benign leukocytosis. Hematology Am Soc Hematol Educ Program 2012; 2012:475–484. doi:10.1182/asheducation-2012.1.475
  9. Watters RJ, Liu X, Loughran TP Jr. T-cell and natural killer-cell large granular lymphocyte leukemia neoplasias. Leuk Lymphoma 2011; 52(12):2217–2225. doi:10.3109/10428194.2011.593276
  10. Lamy T, Moignet A, Loughran TP Jr. LGL leukemia: from pathogenesis to treatment. Blood 2017; 129(9):1082–1094. doi:10.1182/blood-2016-08-692590
  11. Zhang D, Loughran TP Jr. Large granular lymphocytic leukemia: molecular pathogenesis, clinical manifestations, and treatment. Hematology Am Soc Hematol Educ Program 2012; 2012:652–659. doi:10.1182/asheducation-2012.1.652
  12. Swerdlow SH, Campo E, Pileri SA, et al. The 2016 revision of the World Health Organization classification of lymphoid neoplasms. Blood 2016; 127(20):2375–2390. doi:10.1182/blood-2016-01-643569
  13. Rose MG, Berliner N. T-cell large granular lymphocyte leukemia and related disorders. Oncologist 2004; 9(3):247–258. pmid:15169980
  14. Bockorny B, Dasanu CA. Autoimmune manifestations in large granular lymphocyte leukemia. Clin Lymphoma Myeloma Leuk 2012; 12(6):400–405. doi:10.1016/j.clml.2012.06.006
  15. Liu X, Loughran TP Jr. The spectrum of large granular lymphocyte leukemia and Felty’s syndrome. Curr Opin Hematol 2011; 18(4):254–259. doi:10.1097/MOH.0b013e32834760fb
  16. Franchini M, Lippi G. Acquired factor V inhibitors: a systematic review. J Thromb Thrombolysis 2011; 31(4):449–457. doi:10.1007/s11239-010-0529-6
  17. Sallah S, Nguyen NP, Abdallah JM, Hanrahan LR. Acquired hemophilia in patients with hematologic malignancies. Arch Pathol Lab Med 2000; 124(5):730–734.
  18. Murphy PW, Brett LK, Verla-Tebit E, Macik BG, Loughran TP Jr. Acquired inhibitors to factor VIII and fibrinogen in the setting of T-cell large granular lymphocyte leukemia: a case report and review of the literature. Blood Coagul Fibrinolysis 2015; 26(2):211–213. doi:10.1097/MBC.0000000000000209
Article PDF
Author and Disclosure Information

Tahani Atieh, DO
Department of Internal Medicine, Cleveland Clinic; Clinical Instructor, Cleveland Clinic Lerner College of Medicine of Case Western Reserve University, Cleveland, OH

Alan Lichtin, MD
Department of Hematologic Oncology and Blood Disorders, Cleveland Clinic; Associate Professor, Cleveland Clinic Lerner College of Medicine of Case Western Reserve University, Cleveland, OH

Address: Alan Lichtin, MD, Department of Hematology and Medical Oncology, CA-60, Cleveland Clinic, 9500 Euclid Avenue, Cleveland, OH 44195; lichtia@ccf.org

Issue
Cleveland Clinic Journal of Medicine - 86(8)
Publications
Topics
Page Number
535-542
Legacy Keywords
Bruising, coagulation disorder, bleeding disorder, activated partial thromboplastin time, aPTT, prothrombin time, PT, factor VIII, factor VIII inhibitor, antibody, acquired hemophilia A, AHA, coagulation assay, mixing study, lymphocytosis, large granular lymphocytes, LGLs, LGL leukemia, Tahani Atieh, Alan Lichtin
Sections
Author and Disclosure Information

Tahani Atieh, DO
Department of Internal Medicine, Cleveland Clinic; Clinical Instructor, Cleveland Clinic Lerner College of Medicine of Case Western Reserve University, Cleveland, OH

Alan Lichtin, MD
Department of Hematologic Oncology and Blood Disorders, Cleveland Clinic; Associate Professor, Cleveland Clinic Lerner College of Medicine of Case Western Reserve University, Cleveland, OH

Address: Alan Lichtin, MD, Department of Hematology and Medical Oncology, CA-60, Cleveland Clinic, 9500 Euclid Avenue, Cleveland, OH 44195; lichtia@ccf.org

Author and Disclosure Information

Tahani Atieh, DO
Department of Internal Medicine, Cleveland Clinic; Clinical Instructor, Cleveland Clinic Lerner College of Medicine of Case Western Reserve University, Cleveland, OH

Alan Lichtin, MD
Department of Hematologic Oncology and Blood Disorders, Cleveland Clinic; Associate Professor, Cleveland Clinic Lerner College of Medicine of Case Western Reserve University, Cleveland, OH

Address: Alan Lichtin, MD, Department of Hematology and Medical Oncology, CA-60, Cleveland Clinic, 9500 Euclid Avenue, Cleveland, OH 44195; lichtia@ccf.org

Article PDF
Article PDF
Related Articles

A 61-year-old woman presented to our hematology clinic for evaluation of multiple episodes of bruising. The first episode occurred 8 months earlier, when she developed a large bruise after water skiing. Two months before coming to us, she went to her local emergency room because of new bruising and was found to have a prolonged activated partial thromboplastin time (aPTT) of 60 seconds (reference range 23.3–34.9), but she underwent no further testing at that time.

At presentation to our clinic, she reported having no fevers, night sweats, unintentional weight loss, swollen lymph nodes, joint pain, rashes, mouth sores, nosebleeds, or blood in the urine or stool. Her history was notable only for hypothyroidism, which was diagnosed in the previous year. Her medications included levothyroxine, vitamin D3, and vitamin C. She had been taking a baby aspirin daily for the past 10 years but had stopped 1 month earlier because of the bruising.

atieh_bruising_t1.jpg
On examination, she had a single small hematoma on her right thigh. She had no ecchymoses, petechiae, or adenopathy, and her spleen was nonpalpable.

Ten years earlier she had been evaluated for a possible transient ischemic attack; laboratory results at that time included a normal aPTT of 25.1 seconds and a normal factor VIII level of 153% (reference range 50%–173%).

atieh_bruising_t2.jpg
Laboratory testing at our clinic showed a normal complete blood cell count (Table 1); the coagulation factor assay confirmed that her aPTT was elevated (prolonged), but other values were normal (Table 2).

atieh_bruising_t3.jpg
Causes of an isolated prolonged aPTT include medications (eg, heparin), inherited factor deficiencies, acquired inhibitors of coagulation factors, and inherited or acquired von Willebrand disease. Lupus anticoagulant can prolong the aPTT but is usually associated with thrombosis rather than bleeding.1 The differential diagnoses for this and other patterns of coagulation assay abnormalities are listed in Table 3.

EVALUATION FOR AN ISOLATED PROLONGED aPTT

1. What is the appropriate next test to evaluate this patient’s prolonged aPTT?

  • Lupus anticoagulant panel
  • Coagulation factor levels
  • Mixing studies
  • Bethesda assay

Mixing studies

Once a prolonged aPTT is confirmed, the appropriate next step is a mixing study. This involves mixing the patient’s plasma with pooled normal plasma in a 1-to-1 ratio, then repeating the aPTT test immediately, and again after 1 hour of incubation at 37°C. If the patient does not have enough of one of the coagulation factors, the aPTT immediately returns to the normal range when plasma is mixed with the pooled plasma because the pooled plasma contains the factor that is lacking. If this happens, then factor assays should be performed to identify the deficient factor.1

Various antibodies that inhibit coagulation factors can also affect the aPTT. There are 2 general types: immediate-acting and delayed.

With an immediate-acting inhibitor, the aPTT does not correct into the normal range with initial mixing. Immediate-acting inhibitors are often seen together with lupus anticoagulants, which are nonspecific phospholipid antibodies. If an immediate-acting inhibitor is detected, further testing should focus on evaluation for lupus anticoagulant, including phospholipid-dependency studies.

With a delayed inhibitor, the aPTT initially comes down, but subsequently goes back up after incubation. Acquired factor VIII inhibitor is a classic delayed-type inhibitor and is also the most common factor inhibitor.1 If a delayed-acting inhibitor is found, specific intrinsic factor levels should be measured (factors VIII, IX, XI, and XII),2 and testing should also be done for lupus anticoagulant, as these inhibitors may occur together.

Bethesda assay

atieh_bruising_t4.jpg
If factor levels are decreased, a Bethesda assay should be performed to differentiate a specific factor inhibitor from a lupus anticoagulant. In the case of a factor VIII inhibitor, serial dilutions of patient plasma are incubated at 37°C with pooled normal plasma for 2 hours, then factor VIII activity is measured. The reciprocal dilution of patient plasma that results in 50% of factor VIII activity in the control plasma is 1 Bethesda unit (BU). The stronger the inhibitor in the patient’s sample, the more dilutions are required to measure factor VIII activity, and thus the higher the Bethesda titer.3

Case continued: Results of mixing and Bethesda studies

atieh_bruising_t5.jpg
Results of the mixing study (Table 4) showed an initial correction of the aPTT in a 1-to-1 mix, but after 1 hour of incubation, the aPTT was again prolonged at 42 seconds (reference range < 37.3). Further testing revealed very low levels of factor VIII (< 1%), and the presence of a factor VIII inhibitor, quantified at 5.8 BU (reference range < 0.5) (Table 5). Additional coagulation tests, including von Willebrand factor testing and a lupus anticoagulation panel, were negative.

 

 

FACTOR VIII INHIBITOR EVALUATION

2. What is the most likely underlying condition associated with this patient’s factor VIII inhibitor?

  • Autoimmune disease
  • Malignancy
  • A medication
  • Unknown (idiopathic)

Acquired hemophilia A (AHA) is a rare disorder caused by autoantibodies against factor VIII. Its estimated incidence is about 1 person per million per year.4 It usually presents as unexplained bruising or bleeding and is only rarely diagnosed by an incidentally noted prolonged aPTT. The severity of bleeding is variable and can include subcutaneous, soft-tissue, retroperitoneal, gastrointestinal, and intracranial hemorrhage.5

AHA is considered idiopathic in more than half of cases. A study based on a European registry5 of 501 patients with AHA and a UK study6 of 172 patients found no underlying disease in 52% and 65% of patients, respectively. For patients with an identified cause, the most common causes were malignancy (12%5 and 15%6) and autoimmune disease (12%5 and 17%6).

Drugs have rarely been associated with factor VIII inhibitors. Such occurrences have been reported with interferon, blood thinners, antibiotics, and psychiatric medications, but no study yet has indicated causation. However, patients with congenital hemophilia A treated with factor VIII preparations have about a 15% chance of developing factor VIII inhibitors. In this setting, inhibitors develop in response to recombinant factor VIII exposure, unlike the autoimmune phenomena seen in AHA.

TREATMENT OF ACQUIRED HEMOPHILIA A

3. What is the most appropriate treatment for AHA?

  • Desmopressin and prednisone
  • Recombinant porcine factor VIII and prednisone plus cyclophosphamide
  • Recombinant factor VIIa and rituximab
  • Any of the above

Any of the above regimens can be used. In general, treatment of AHA has two purposes: to stop acute hemorrhage, and to reduce the level of factor VIII inhibitor. No standard treatment guidelines are available; evidence of the effectiveness of different drugs is based largely on data on congenital hemophilia A.3

Acute treatment to stop bleeding

Initial treatment of AHA often focuses on stopping an acute hemorrhage by either raising circulating levels of factor VIII or bypassing it in the coagulation cascade.

Desmopressin can temporarily raise factor VIII levels, but it is often ineffective in AHA unless the patient has very low inhibitor titers.3

Factor VIII concentrate (human or recombinant porcine factor VIII) may be effective in patients with low inhibitor titers (< 5 BU). Higher doses are often required than those used in congenital hemophilia A. Factor VIII concentrate is usually combined with immunosuppressive treatment to lower the factor VIII inhibitor level (described below).3

If these methods are ineffective or the patient has high inhibitor titers (> 5 BU), activated prothrombin complex concentrates, known as FEIBA (factor eight inhibitor bypassing activity), or recombinant factor VIIa is available. These agents bypass factor VIII in the clotting cascade.

Immunosuppression to reduce factor VIII inhibitor

Immunosuppressive agents are the mainstay of AHA treatment to lower the inhibitor level.

Regimens vary. A 2003 meta-analysis4 including 249 patients found that prednisone alone resulted in complete response in about 30% of patients, and the addition of cyclophosphamide increased the response rate to 60% to 100%. High-dose intravenous immunoglobulin led to conflicting results. Conclusions were limited by the variability of dosing and duration in treatment regimens among the 20 different studies included.

An analysis of 331 patients in the European Acquired Hemophilia Registry (EACH2)7 found that steroids alone produced remission in 48% of patients, while steroids combined with cyclophosphamide raised the rate to 70%. Rituximab-based regimens were successful in 59% but required twice as long to achieve remission as steroid or cyclophosphamide-based regimens. No benefit was noted from intravenous immunoglobulin.

Risks of disease and treatment

AHA is associated with significant risk of morbidity and death related to bleeding, complications of treatment, and underlying disease.

In EACH2, 16 of the 331 patients died of bleeding, 16 died of causes related to immunosuppression, and 45 died of causes related to the underlying condition.5 In the UK registry of 172 patients, 13 patients died of bleeding, and 12 died of sepsis related to immunosuppression.6

The factor VIII level and inhibitor titer are not necessarily useful in stratifying bleeding risk, as severe and fatal bleeding can occur at variable levels and patients remain at risk of bleeding as long as the inhibitor persists.6,7

 

 

CASE CONTINUED: TREATMENT, LYMPHOCYTOSIS

The patient was started on 60 mg daily of prednisone, resulting in a decrease in her aPTT, increase in factor VIII level, and lower Bethesda titer. On a return visit, her absolute lymphocyte count was 7.04 × 109/L (reference range 1.0–4.0). She reported no fevers, chills, or recent infections.  

EVALUATING LYMPHOCYTOSIS

Lymphocytosis is defined in most laboratories as an absolute lymphocyte count greater than 4.0 × 109/L for adults. Normally, T cells (CD3+) make up 60% to 80% of lymphocytes, B cells (CD20+) 10% to 20%, and natural killer (NK) cells (CD3–, CD56+) 5% to 10%. Lymphocytosis is usually caused by infection, but it can have other causes, including malignancy.

Peripheral blood smear. If there is no clear cause of lymphocytosis, a peripheral blood smear can be used to assess lymphocyte morphology, providing clues to the underlying etiology. For example, atypical lymphocytes are often seen in infectious mononucleosis, while “smudge” lymphocytes are characteristic of chronic lymphocytic leukemia. If a peripheral smear shows abnormal morphology, further workup should include establishing whether the lymphocytes are polyclonal or clonal.8

CASE CONTINUED: LARGE GRANULAR LYMPHOCYTES

atieh_bruising_f1.jpg
%3Cp%3EFigure%201.%20A%20large%20granular%20lymphocyte.%3C%2Fp%3E
On the patient’s peripheral smear, 54% of lymphocytes were large lymphocytes with moderate amounts of pale cytoplasm filled with azurophilic granules, consistent with large granular lymphocytes (LGLs) (Figure 1).

4. What is the next step to evaluate the patient’s lymphocytosis?

  • Bone marrow biopsy
  • Karyotype analysis
  • Flow cytometry
  • Fluorescence in situ hybridization

Flow cytometry with V-beta analysis is the best first test to determine the cause of lymphocytosis after review of the peripheral smear. For persistent lymphocytosis, flow cytometry should be done even if a peripheral smear shows normal lymphocyte morphology.

Most T cells possess receptors composed of alpha and beta chains, each encoded by variable (V), diversity (D), joining (J), and constant (C) gene segments. The V, D, and J segments undergo rearrangement during T-cell development in the thymus based on antigen exposure, producing a diverse T-cell receptor population.

In a polyclonal population of lymphocytes, the T-cell receptors have a variety of gene segment arrangements, indicating normal T-cell development. But in a clonal population of lymphocytes, the T-cell receptors have a single identical gene segment arrangement, indicating they all originated from a single clone.9 Lymphocytosis in response to an infection is typically polyclonal, while malignant lymphocytosis is clonal. 

Monoclonal antibodies against many of the variable regions of the beta chain (V-beta) of T-cell receptors have been developed, enabling flow cytometry to establish clonality.

T-cell receptor gene rearrangement studies can also be performed using polymerase chain reaction and Southern blot techniques.9

Karyotype analysis is usually not performed for the finding of LGLs, because most leukemias (eg, T-cell and NK-cell leukemias) have cells with a normal karyotype. 

Bone marrow biopsy is invasive and usually not required to evaluate LGLs. It can be especially risky for a patient with a bleeding disorder such as a factor VIII inhibitor.10

Case continued: Flow cytometry confirms clonality

Subsequent flow cytometry found that more than 50% of the patient’s lymphocytes were LGLs that co-expressed CD3+, CD8+, CD56+, and CD57+, with aberrantly decreased CD7 expression. T-cell V-beta analysis demonstrated an expansion of the V-beta 17 family, and T-cell receptor gene analysis with polymerase chain reaction confirmed the presence of a clonal rearrangement.

LGL LEUKEMIA: CLASSIFICATION AND MANAGEMENT

LGLs normally account for 10% to 15% of peripheral mononuclear cells.11 LGL leukemia is caused by a clonal population of cytotoxic T cells or NK cells and involves an increased number of LGLs (usually > 2 × 109/L).10

LGL leukemia is divided into 3 categories according to the most recent World Health Organization classification10,12:

T-cell LGL leukemia (about 85% of cases) is considered indolent but can cause significant cytopenias and is often associated with autoimmune disease.13 Cells usually express a CD3+, CD8+, CD16+, and CD57+ phenotype. Survival is about 70% at 10 years.

Chronic NK-cell lymphocytosis (about 10%) also tends to have an indolent course with cytopenia and an autoimmune association, and with a similar prognosis to T-cell LGL leukemia. Cells express a CD3–, CD16+, and CD56+ phenotype.

Aggressive NK-cell LGL leukemia (about 5%) is associated with Epstein-Barr virus infection and occurs in younger patients. It is characterized by severe cytopenias, “B symptoms” (ie, fever, night sweats, weight loss), and has a very poor prognosis. Like chronic NK-cell lymphocytosis, cells express a CD3–, CD16+, and CD56+ phenotype. Fas (CD95) and Fas-ligand (CD178) are strongly expressed.10,13

Most cases of LGL leukemia can be diagnosed on the basis of classic morphology on peripheral blood smear and evidence of clonality on flow cytometry or gene rearrangement studies. T-cell receptor gene studies cannot be used to establish clonality in the NK subtypes, as NK cells do not express T-cell receptors.11

Case continued: Diagnosis, continued course 

In our patient, T-cell LGL leukemia was diagnosed on the basis of the peripheral smear, flow cytometry results, and positive T-cell receptor gene studies for clonal rearrangement in the T-cell receptor beta region.

While her corticosteroid therapy was being tapered, her factor III inhibitor level increased, and she had a small episode of bleeding, prompting the start of cyclophosphamide 50 mg daily with lower doses of prednisone.

atieh_bruising_f2.jpg
%3Cp%3EFigure%202.%20The%20patient%E2%80%99s%20clinical%20course%3A%20factor%20VIII%20inhibitor%20response%20to%20treatment.%3C%2Fp%3E
She then developed elevated liver enzymes, prompting discontinuation of cyclophosphamide. Rituximab was started and continued for 4 weekly doses, resulting in normalization of aPTT and factor VIII level with undetectable Bethesda titers (Figure 2).

 

 

LGL LEUKEMIA AND AUTOIMMUNE DISEASE

Patients with LGL leukemia commonly have or develop autoimmune conditions. Immune-mediated cytopenias including pure red cell aplasia, aplastic anemia, and autoimmune hemolytic anemias can occur. Neutropenia, the most common cytopenia in LGL leukemia, is thought to be at least partly autoimmune, as the degree of neutropenia is often worse than would be expected solely from bone-marrow infiltration of LGL cells.10,14,15

Rheumatoid arthritis is the most common autoimmune condition associated with LGL leukemia, with a reported incidence between 11% and 36%.13–15

Felty syndrome (rheumatoid arthritis, splenomegaly, and neutropenia) is often associated with LGL leukemia and is thought by some to be part of the same disease process.15

Treat with immunosuppressives if needed

Indications for treating LGL leukemia include the development of cytopenias and associated autoimmune diseases. Immunosuppressive agents, such as methotrexate, cyclophosphamide, and cyclosporine, are commonly used.10,11,14 Most evidence of treatment efficacy is from retrospective studies and case reports, with widely variable response rates that overall are around 50%.10

ACQUIRED HEMOPHILIA A AND HEMATOLOGIC MALIGNANCY

A systematic review found 30 cases of AHA associated with hematologic malignancies.16 The largest case series17 in this analysis had 8 patients, and included diagnoses of chronic lymphocytic leukemia, erythroleukemia, myelofibrosis, multiple myeloma, and myelodysplastic syndrome. In 3 of these patients, the appearance of the inhibitor preceded the diagnosis of the underlying malignancy by an average of 3.5 months. In 1 patient with erythroleukemia and another with multiple myeloma, the activity of the inhibitor could be clearly correlated with the underlying malignancy. In the other 6 patients, no association between the two could be made.

In the same series, complete resolution of the inhibitor was related only to the level of Bethesda titer present at diagnosis, with those who achieved resolution having lower mean Bethesda titers.17 Similarly, in EACH2, lower inhibitor Bethesda titers and higher factor VIII levels at presentation were associated with faster inhibitor eradication and normalization of factor VIII levels.7

Murphy et al18 described a 62-year-old woman with Felty syndrome who developed a factor VIII inhibitor and was subsequently given a diagnosis of LGL leukemia. Treatment with immunosuppressive agents, including cyclophosphamide, azathioprine, and rituximab, successfully eradicated her factor VIII inhibitor, although the LGL leukemia persisted.

Case conclusion: Eradication of factor VIII inhibitor

Our patient, similar to the patient described by Murphy et al18 above, had eradication of the factor VIII inhibitor despite persistence of LGL leukemia. Between the time of diagnosis at our clinic, when she had 54% LGLs, and eradication of the inhibitor 3 months later, the LGL percentage ranged from 45% to 89%. No clear direct correlation between LGL and factor VIII inhibitor levels could be detected.

Given the strong association of LGL leukemia with autoimmune disease, it is tempting to believe that her factor VIII inhibitor was somehow related to her malignancy, although the exact mechanism remained unclear. The average age at diagnosis is 60 for LGL leukemia11 and over 70 for AHA,5,6 so advanced age may be the common denominator. Whether or not our patient will have recurrence of her factor VIII inhibitor or the development of other autoimmune diseases with the persistence of her LGL leukemia remains to be seen.

At last follow-up, our patient was off all therapy and continued to have normal aPTT and factor VIII levels. Repeat flow cytometry after treatment of her factor VIII inhibitor showed persistence of a clonal T-cell population, although reduced from 72% to 60%. It may be that the 2 entities were unrelated, and the clonal T-cell population was simply fluctuating over time. This can be determined only with further observation. As the patient had no symptoms from her LGL leukemia, she continued to be observed without treatment.

TAKE-HOME POINTS

  • The coagulation assay is key to initially assessing a bleeding abnormality; whether the prothrombin time and aPTT are normal or prolonged narrows the differential diagnosis and determines next steps in evaluation.
  • Mixing studies can help pinpoint the responsible deficient factor.
  • Acquired factor VIII deficiency, also known as AHA, may be caused by autoimmune disease, malignancy, or medications, but it is usually idiopathic.
  • AHA treatment is focused on achieving hemostasis and reducing factor VIII inhibitor.
  • Lymphocytosis should be evaluated with a peripheral blood smear and flow cytometry to determine if the population is polyclonal (associated with infection) or clonal (associated with malignancy).
  • LGL leukemia is usually a chronic, indolent disease, although an uncommon subtype has an aggressive course.
  • The association between AHA and LGL leukemia is unclear, and both conditions must be monitored and managed.

A 61-year-old woman presented to our hematology clinic for evaluation of multiple episodes of bruising. The first episode occurred 8 months earlier, when she developed a large bruise after water skiing. Two months before coming to us, she went to her local emergency room because of new bruising and was found to have a prolonged activated partial thromboplastin time (aPTT) of 60 seconds (reference range 23.3–34.9), but she underwent no further testing at that time.

At presentation to our clinic, she reported having no fevers, night sweats, unintentional weight loss, swollen lymph nodes, joint pain, rashes, mouth sores, nosebleeds, or blood in the urine or stool. Her history was notable only for hypothyroidism, which was diagnosed in the previous year. Her medications included levothyroxine, vitamin D3, and vitamin C. She had been taking a baby aspirin daily for the past 10 years but had stopped 1 month earlier because of the bruising.

atieh_bruising_t1.jpg
On examination, she had a single small hematoma on her right thigh. She had no ecchymoses, petechiae, or adenopathy, and her spleen was nonpalpable.

Ten years earlier she had been evaluated for a possible transient ischemic attack; laboratory results at that time included a normal aPTT of 25.1 seconds and a normal factor VIII level of 153% (reference range 50%–173%).

atieh_bruising_t2.jpg
Laboratory testing at our clinic showed a normal complete blood cell count (Table 1); the coagulation factor assay confirmed that her aPTT was elevated (prolonged), but other values were normal (Table 2).

atieh_bruising_t3.jpg
Causes of an isolated prolonged aPTT include medications (eg, heparin), inherited factor deficiencies, acquired inhibitors of coagulation factors, and inherited or acquired von Willebrand disease. Lupus anticoagulant can prolong the aPTT but is usually associated with thrombosis rather than bleeding.1 The differential diagnoses for this and other patterns of coagulation assay abnormalities are listed in Table 3.

EVALUATION FOR AN ISOLATED PROLONGED aPTT

1. What is the appropriate next test to evaluate this patient’s prolonged aPTT?

  • Lupus anticoagulant panel
  • Coagulation factor levels
  • Mixing studies
  • Bethesda assay

Mixing studies

Once a prolonged aPTT is confirmed, the appropriate next step is a mixing study. This involves mixing the patient’s plasma with pooled normal plasma in a 1-to-1 ratio, then repeating the aPTT test immediately, and again after 1 hour of incubation at 37°C. If the patient does not have enough of one of the coagulation factors, the aPTT immediately returns to the normal range when plasma is mixed with the pooled plasma because the pooled plasma contains the factor that is lacking. If this happens, then factor assays should be performed to identify the deficient factor.1

Various antibodies that inhibit coagulation factors can also affect the aPTT. There are 2 general types: immediate-acting and delayed.

With an immediate-acting inhibitor, the aPTT does not correct into the normal range with initial mixing. Immediate-acting inhibitors are often seen together with lupus anticoagulants, which are nonspecific phospholipid antibodies. If an immediate-acting inhibitor is detected, further testing should focus on evaluation for lupus anticoagulant, including phospholipid-dependency studies.

With a delayed inhibitor, the aPTT initially comes down, but subsequently goes back up after incubation. Acquired factor VIII inhibitor is a classic delayed-type inhibitor and is also the most common factor inhibitor.1 If a delayed-acting inhibitor is found, specific intrinsic factor levels should be measured (factors VIII, IX, XI, and XII),2 and testing should also be done for lupus anticoagulant, as these inhibitors may occur together.

Bethesda assay

atieh_bruising_t4.jpg
If factor levels are decreased, a Bethesda assay should be performed to differentiate a specific factor inhibitor from a lupus anticoagulant. In the case of a factor VIII inhibitor, serial dilutions of patient plasma are incubated at 37°C with pooled normal plasma for 2 hours, then factor VIII activity is measured. The reciprocal dilution of patient plasma that results in 50% of factor VIII activity in the control plasma is 1 Bethesda unit (BU). The stronger the inhibitor in the patient’s sample, the more dilutions are required to measure factor VIII activity, and thus the higher the Bethesda titer.3

Case continued: Results of mixing and Bethesda studies

atieh_bruising_t5.jpg
Results of the mixing study (Table 4) showed an initial correction of the aPTT in a 1-to-1 mix, but after 1 hour of incubation, the aPTT was again prolonged at 42 seconds (reference range < 37.3). Further testing revealed very low levels of factor VIII (< 1%), and the presence of a factor VIII inhibitor, quantified at 5.8 BU (reference range < 0.5) (Table 5). Additional coagulation tests, including von Willebrand factor testing and a lupus anticoagulation panel, were negative.

 

 

FACTOR VIII INHIBITOR EVALUATION

2. What is the most likely underlying condition associated with this patient’s factor VIII inhibitor?

  • Autoimmune disease
  • Malignancy
  • A medication
  • Unknown (idiopathic)

Acquired hemophilia A (AHA) is a rare disorder caused by autoantibodies against factor VIII. Its estimated incidence is about 1 person per million per year.4 It usually presents as unexplained bruising or bleeding and is only rarely diagnosed by an incidentally noted prolonged aPTT. The severity of bleeding is variable and can include subcutaneous, soft-tissue, retroperitoneal, gastrointestinal, and intracranial hemorrhage.5

AHA is considered idiopathic in more than half of cases. A study based on a European registry5 of 501 patients with AHA and a UK study6 of 172 patients found no underlying disease in 52% and 65% of patients, respectively. For patients with an identified cause, the most common causes were malignancy (12%5 and 15%6) and autoimmune disease (12%5 and 17%6).

Drugs have rarely been associated with factor VIII inhibitors. Such occurrences have been reported with interferon, blood thinners, antibiotics, and psychiatric medications, but no study yet has indicated causation. However, patients with congenital hemophilia A treated with factor VIII preparations have about a 15% chance of developing factor VIII inhibitors. In this setting, inhibitors develop in response to recombinant factor VIII exposure, unlike the autoimmune phenomena seen in AHA.

TREATMENT OF ACQUIRED HEMOPHILIA A

3. What is the most appropriate treatment for AHA?

  • Desmopressin and prednisone
  • Recombinant porcine factor VIII and prednisone plus cyclophosphamide
  • Recombinant factor VIIa and rituximab
  • Any of the above

Any of the above regimens can be used. In general, treatment of AHA has two purposes: to stop acute hemorrhage, and to reduce the level of factor VIII inhibitor. No standard treatment guidelines are available; evidence of the effectiveness of different drugs is based largely on data on congenital hemophilia A.3

Acute treatment to stop bleeding

Initial treatment of AHA often focuses on stopping an acute hemorrhage by either raising circulating levels of factor VIII or bypassing it in the coagulation cascade.

Desmopressin can temporarily raise factor VIII levels, but it is often ineffective in AHA unless the patient has very low inhibitor titers.3

Factor VIII concentrate (human or recombinant porcine factor VIII) may be effective in patients with low inhibitor titers (< 5 BU). Higher doses are often required than those used in congenital hemophilia A. Factor VIII concentrate is usually combined with immunosuppressive treatment to lower the factor VIII inhibitor level (described below).3

If these methods are ineffective or the patient has high inhibitor titers (> 5 BU), activated prothrombin complex concentrates, known as FEIBA (factor eight inhibitor bypassing activity), or recombinant factor VIIa is available. These agents bypass factor VIII in the clotting cascade.

Immunosuppression to reduce factor VIII inhibitor

Immunosuppressive agents are the mainstay of AHA treatment to lower the inhibitor level.

Regimens vary. A 2003 meta-analysis4 including 249 patients found that prednisone alone resulted in complete response in about 30% of patients, and the addition of cyclophosphamide increased the response rate to 60% to 100%. High-dose intravenous immunoglobulin led to conflicting results. Conclusions were limited by the variability of dosing and duration in treatment regimens among the 20 different studies included.

An analysis of 331 patients in the European Acquired Hemophilia Registry (EACH2)7 found that steroids alone produced remission in 48% of patients, while steroids combined with cyclophosphamide raised the rate to 70%. Rituximab-based regimens were successful in 59% but required twice as long to achieve remission as steroid or cyclophosphamide-based regimens. No benefit was noted from intravenous immunoglobulin.

Risks of disease and treatment

AHA is associated with significant risk of morbidity and death related to bleeding, complications of treatment, and underlying disease.

In EACH2, 16 of the 331 patients died of bleeding, 16 died of causes related to immunosuppression, and 45 died of causes related to the underlying condition.5 In the UK registry of 172 patients, 13 patients died of bleeding, and 12 died of sepsis related to immunosuppression.6

The factor VIII level and inhibitor titer are not necessarily useful in stratifying bleeding risk, as severe and fatal bleeding can occur at variable levels and patients remain at risk of bleeding as long as the inhibitor persists.6,7

 

 

CASE CONTINUED: TREATMENT, LYMPHOCYTOSIS

The patient was started on 60 mg daily of prednisone, resulting in a decrease in her aPTT, increase in factor VIII level, and lower Bethesda titer. On a return visit, her absolute lymphocyte count was 7.04 × 109/L (reference range 1.0–4.0). She reported no fevers, chills, or recent infections.  

EVALUATING LYMPHOCYTOSIS

Lymphocytosis is defined in most laboratories as an absolute lymphocyte count greater than 4.0 × 109/L for adults. Normally, T cells (CD3+) make up 60% to 80% of lymphocytes, B cells (CD20+) 10% to 20%, and natural killer (NK) cells (CD3–, CD56+) 5% to 10%. Lymphocytosis is usually caused by infection, but it can have other causes, including malignancy.

Peripheral blood smear. If there is no clear cause of lymphocytosis, a peripheral blood smear can be used to assess lymphocyte morphology, providing clues to the underlying etiology. For example, atypical lymphocytes are often seen in infectious mononucleosis, while “smudge” lymphocytes are characteristic of chronic lymphocytic leukemia. If a peripheral smear shows abnormal morphology, further workup should include establishing whether the lymphocytes are polyclonal or clonal.8

CASE CONTINUED: LARGE GRANULAR LYMPHOCYTES

atieh_bruising_f1.jpg
%3Cp%3EFigure%201.%20A%20large%20granular%20lymphocyte.%3C%2Fp%3E
On the patient’s peripheral smear, 54% of lymphocytes were large lymphocytes with moderate amounts of pale cytoplasm filled with azurophilic granules, consistent with large granular lymphocytes (LGLs) (Figure 1).

4. What is the next step to evaluate the patient’s lymphocytosis?

  • Bone marrow biopsy
  • Karyotype analysis
  • Flow cytometry
  • Fluorescence in situ hybridization

Flow cytometry with V-beta analysis is the best first test to determine the cause of lymphocytosis after review of the peripheral smear. For persistent lymphocytosis, flow cytometry should be done even if a peripheral smear shows normal lymphocyte morphology.

Most T cells possess receptors composed of alpha and beta chains, each encoded by variable (V), diversity (D), joining (J), and constant (C) gene segments. The V, D, and J segments undergo rearrangement during T-cell development in the thymus based on antigen exposure, producing a diverse T-cell receptor population.

In a polyclonal population of lymphocytes, the T-cell receptors have a variety of gene segment arrangements, indicating normal T-cell development. But in a clonal population of lymphocytes, the T-cell receptors have a single identical gene segment arrangement, indicating they all originated from a single clone.9 Lymphocytosis in response to an infection is typically polyclonal, while malignant lymphocytosis is clonal. 

Monoclonal antibodies against many of the variable regions of the beta chain (V-beta) of T-cell receptors have been developed, enabling flow cytometry to establish clonality.

T-cell receptor gene rearrangement studies can also be performed using polymerase chain reaction and Southern blot techniques.9

Karyotype analysis is usually not performed for the finding of LGLs, because most leukemias (eg, T-cell and NK-cell leukemias) have cells with a normal karyotype. 

Bone marrow biopsy is invasive and usually not required to evaluate LGLs. It can be especially risky for a patient with a bleeding disorder such as a factor VIII inhibitor.10

Case continued: Flow cytometry confirms clonality

Subsequent flow cytometry found that more than 50% of the patient’s lymphocytes were LGLs that co-expressed CD3+, CD8+, CD56+, and CD57+, with aberrantly decreased CD7 expression. T-cell V-beta analysis demonstrated an expansion of the V-beta 17 family, and T-cell receptor gene analysis with polymerase chain reaction confirmed the presence of a clonal rearrangement.

LGL LEUKEMIA: CLASSIFICATION AND MANAGEMENT

LGLs normally account for 10% to 15% of peripheral mononuclear cells.11 LGL leukemia is caused by a clonal population of cytotoxic T cells or NK cells and involves an increased number of LGLs (usually > 2 × 109/L).10

LGL leukemia is divided into 3 categories according to the most recent World Health Organization classification10,12:

T-cell LGL leukemia (about 85% of cases) is considered indolent but can cause significant cytopenias and is often associated with autoimmune disease.13 Cells usually express a CD3+, CD8+, CD16+, and CD57+ phenotype. Survival is about 70% at 10 years.

Chronic NK-cell lymphocytosis (about 10%) also tends to have an indolent course with cytopenia and an autoimmune association, and with a similar prognosis to T-cell LGL leukemia. Cells express a CD3–, CD16+, and CD56+ phenotype.

Aggressive NK-cell LGL leukemia (about 5%) is associated with Epstein-Barr virus infection and occurs in younger patients. It is characterized by severe cytopenias, “B symptoms” (ie, fever, night sweats, weight loss), and has a very poor prognosis. Like chronic NK-cell lymphocytosis, cells express a CD3–, CD16+, and CD56+ phenotype. Fas (CD95) and Fas-ligand (CD178) are strongly expressed.10,13

Most cases of LGL leukemia can be diagnosed on the basis of classic morphology on peripheral blood smear and evidence of clonality on flow cytometry or gene rearrangement studies. T-cell receptor gene studies cannot be used to establish clonality in the NK subtypes, as NK cells do not express T-cell receptors.11

Case continued: Diagnosis, continued course 

In our patient, T-cell LGL leukemia was diagnosed on the basis of the peripheral smear, flow cytometry results, and positive T-cell receptor gene studies for clonal rearrangement in the T-cell receptor beta region.

While her corticosteroid therapy was being tapered, her factor III inhibitor level increased, and she had a small episode of bleeding, prompting the start of cyclophosphamide 50 mg daily with lower doses of prednisone.

atieh_bruising_f2.jpg
%3Cp%3EFigure%202.%20The%20patient%E2%80%99s%20clinical%20course%3A%20factor%20VIII%20inhibitor%20response%20to%20treatment.%3C%2Fp%3E
She then developed elevated liver enzymes, prompting discontinuation of cyclophosphamide. Rituximab was started and continued for 4 weekly doses, resulting in normalization of aPTT and factor VIII level with undetectable Bethesda titers (Figure 2).

 

 

LGL LEUKEMIA AND AUTOIMMUNE DISEASE

Patients with LGL leukemia commonly have or develop autoimmune conditions. Immune-mediated cytopenias including pure red cell aplasia, aplastic anemia, and autoimmune hemolytic anemias can occur. Neutropenia, the most common cytopenia in LGL leukemia, is thought to be at least partly autoimmune, as the degree of neutropenia is often worse than would be expected solely from bone-marrow infiltration of LGL cells.10,14,15

Rheumatoid arthritis is the most common autoimmune condition associated with LGL leukemia, with a reported incidence between 11% and 36%.13–15

Felty syndrome (rheumatoid arthritis, splenomegaly, and neutropenia) is often associated with LGL leukemia and is thought by some to be part of the same disease process.15

Treat with immunosuppressives if needed

Indications for treating LGL leukemia include the development of cytopenias and associated autoimmune diseases. Immunosuppressive agents, such as methotrexate, cyclophosphamide, and cyclosporine, are commonly used.10,11,14 Most evidence of treatment efficacy is from retrospective studies and case reports, with widely variable response rates that overall are around 50%.10

ACQUIRED HEMOPHILIA A AND HEMATOLOGIC MALIGNANCY

A systematic review found 30 cases of AHA associated with hematologic malignancies.16 The largest case series17 in this analysis had 8 patients, and included diagnoses of chronic lymphocytic leukemia, erythroleukemia, myelofibrosis, multiple myeloma, and myelodysplastic syndrome. In 3 of these patients, the appearance of the inhibitor preceded the diagnosis of the underlying malignancy by an average of 3.5 months. In 1 patient with erythroleukemia and another with multiple myeloma, the activity of the inhibitor could be clearly correlated with the underlying malignancy. In the other 6 patients, no association between the two could be made.

In the same series, complete resolution of the inhibitor was related only to the level of Bethesda titer present at diagnosis, with those who achieved resolution having lower mean Bethesda titers.17 Similarly, in EACH2, lower inhibitor Bethesda titers and higher factor VIII levels at presentation were associated with faster inhibitor eradication and normalization of factor VIII levels.7

Murphy et al18 described a 62-year-old woman with Felty syndrome who developed a factor VIII inhibitor and was subsequently given a diagnosis of LGL leukemia. Treatment with immunosuppressive agents, including cyclophosphamide, azathioprine, and rituximab, successfully eradicated her factor VIII inhibitor, although the LGL leukemia persisted.

Case conclusion: Eradication of factor VIII inhibitor

Our patient, similar to the patient described by Murphy et al18 above, had eradication of the factor VIII inhibitor despite persistence of LGL leukemia. Between the time of diagnosis at our clinic, when she had 54% LGLs, and eradication of the inhibitor 3 months later, the LGL percentage ranged from 45% to 89%. No clear direct correlation between LGL and factor VIII inhibitor levels could be detected.

Given the strong association of LGL leukemia with autoimmune disease, it is tempting to believe that her factor VIII inhibitor was somehow related to her malignancy, although the exact mechanism remained unclear. The average age at diagnosis is 60 for LGL leukemia11 and over 70 for AHA,5,6 so advanced age may be the common denominator. Whether or not our patient will have recurrence of her factor VIII inhibitor or the development of other autoimmune diseases with the persistence of her LGL leukemia remains to be seen.

At last follow-up, our patient was off all therapy and continued to have normal aPTT and factor VIII levels. Repeat flow cytometry after treatment of her factor VIII inhibitor showed persistence of a clonal T-cell population, although reduced from 72% to 60%. It may be that the 2 entities were unrelated, and the clonal T-cell population was simply fluctuating over time. This can be determined only with further observation. As the patient had no symptoms from her LGL leukemia, she continued to be observed without treatment.

TAKE-HOME POINTS

  • The coagulation assay is key to initially assessing a bleeding abnormality; whether the prothrombin time and aPTT are normal or prolonged narrows the differential diagnosis and determines next steps in evaluation.
  • Mixing studies can help pinpoint the responsible deficient factor.
  • Acquired factor VIII deficiency, also known as AHA, may be caused by autoimmune disease, malignancy, or medications, but it is usually idiopathic.
  • AHA treatment is focused on achieving hemostasis and reducing factor VIII inhibitor.
  • Lymphocytosis should be evaluated with a peripheral blood smear and flow cytometry to determine if the population is polyclonal (associated with infection) or clonal (associated with malignancy).
  • LGL leukemia is usually a chronic, indolent disease, although an uncommon subtype has an aggressive course.
  • The association between AHA and LGL leukemia is unclear, and both conditions must be monitored and managed.
References
  1. Kamal AH, Tefferi A, Pruthi RK. How to interpret and pursue an abnormal prothrombin time, activated partial thromboplastin time, and bleeding time in adults. Mayo Clin Proc 2007; 82(7):864–873. doi:10.4065/82.7.864
  2. Tcherniantchouk O, Laposata M, Marques MB. The isolated prolonged PTT. Am J Hematol 2013; 88(1):82–85. doi:10.1002/ajh.23285
  3. Ma AD, Carrizosa D. Acquired factor VIII inhibitors: pathophysiology and treatment. Hematology Am Soc Hematol Educ Program 2006:432–437. doi:10.1182/asheducation-2006.1.432
  4. Delgado J, Jimenez-Yuste V, Hernandez-Navarro F, Villar A. Acquired haemophilia: review and meta-analysis focused on therapy and prognostic factors. Br J Haematol 2003; 121(1):21–35. pmid:12670328
  5. Knoebl P, Marco P, Baudo F, et al; EACH2 Registry Contributors. Demographic and clinical data in acquired hemophilia A: results from the European Acquired Haemophilia Registry (EACH2). J Thromb Haemost 2012; 10(4):622–631. doi:10.1111/j.1538-7836.2012.04654.x
  6. Collins PW, Hirsch S, Baglin TP, et al; UK Haemophilia Centre Doctors’ Organisation. Acquired hemophilia A in the United Kingdom: a 2-year national surveillance study by the United Kingdom Haemophilia Centre Doctors’ Organisation. Blood 2007; 109(5):1870–1877. doi:10.1182/blood-2006-06-029850
  7. Collins P, Baudo F, Knoebl P, et al; EACH2 Registry Collaborators. Immunosuppression for acquired hemophilia A: results from the European Acquired Haemophilia Registry (EACH2). Blood 2012; 120(1):47–55. doi:10.1182/blood-2012-02-409185
  8. George TI. Malignant or benign leukocytosis. Hematology Am Soc Hematol Educ Program 2012; 2012:475–484. doi:10.1182/asheducation-2012.1.475
  9. Watters RJ, Liu X, Loughran TP Jr. T-cell and natural killer-cell large granular lymphocyte leukemia neoplasias. Leuk Lymphoma 2011; 52(12):2217–2225. doi:10.3109/10428194.2011.593276
  10. Lamy T, Moignet A, Loughran TP Jr. LGL leukemia: from pathogenesis to treatment. Blood 2017; 129(9):1082–1094. doi:10.1182/blood-2016-08-692590
  11. Zhang D, Loughran TP Jr. Large granular lymphocytic leukemia: molecular pathogenesis, clinical manifestations, and treatment. Hematology Am Soc Hematol Educ Program 2012; 2012:652–659. doi:10.1182/asheducation-2012.1.652
  12. Swerdlow SH, Campo E, Pileri SA, et al. The 2016 revision of the World Health Organization classification of lymphoid neoplasms. Blood 2016; 127(20):2375–2390. doi:10.1182/blood-2016-01-643569
  13. Rose MG, Berliner N. T-cell large granular lymphocyte leukemia and related disorders. Oncologist 2004; 9(3):247–258. pmid:15169980
  14. Bockorny B, Dasanu CA. Autoimmune manifestations in large granular lymphocyte leukemia. Clin Lymphoma Myeloma Leuk 2012; 12(6):400–405. doi:10.1016/j.clml.2012.06.006
  15. Liu X, Loughran TP Jr. The spectrum of large granular lymphocyte leukemia and Felty’s syndrome. Curr Opin Hematol 2011; 18(4):254–259. doi:10.1097/MOH.0b013e32834760fb
  16. Franchini M, Lippi G. Acquired factor V inhibitors: a systematic review. J Thromb Thrombolysis 2011; 31(4):449–457. doi:10.1007/s11239-010-0529-6
  17. Sallah S, Nguyen NP, Abdallah JM, Hanrahan LR. Acquired hemophilia in patients with hematologic malignancies. Arch Pathol Lab Med 2000; 124(5):730–734.
  18. Murphy PW, Brett LK, Verla-Tebit E, Macik BG, Loughran TP Jr. Acquired inhibitors to factor VIII and fibrinogen in the setting of T-cell large granular lymphocyte leukemia: a case report and review of the literature. Blood Coagul Fibrinolysis 2015; 26(2):211–213. doi:10.1097/MBC.0000000000000209
References
  1. Kamal AH, Tefferi A, Pruthi RK. How to interpret and pursue an abnormal prothrombin time, activated partial thromboplastin time, and bleeding time in adults. Mayo Clin Proc 2007; 82(7):864–873. doi:10.4065/82.7.864
  2. Tcherniantchouk O, Laposata M, Marques MB. The isolated prolonged PTT. Am J Hematol 2013; 88(1):82–85. doi:10.1002/ajh.23285
  3. Ma AD, Carrizosa D. Acquired factor VIII inhibitors: pathophysiology and treatment. Hematology Am Soc Hematol Educ Program 2006:432–437. doi:10.1182/asheducation-2006.1.432
  4. Delgado J, Jimenez-Yuste V, Hernandez-Navarro F, Villar A. Acquired haemophilia: review and meta-analysis focused on therapy and prognostic factors. Br J Haematol 2003; 121(1):21–35. pmid:12670328
  5. Knoebl P, Marco P, Baudo F, et al; EACH2 Registry Contributors. Demographic and clinical data in acquired hemophilia A: results from the European Acquired Haemophilia Registry (EACH2). J Thromb Haemost 2012; 10(4):622–631. doi:10.1111/j.1538-7836.2012.04654.x
  6. Collins PW, Hirsch S, Baglin TP, et al; UK Haemophilia Centre Doctors’ Organisation. Acquired hemophilia A in the United Kingdom: a 2-year national surveillance study by the United Kingdom Haemophilia Centre Doctors’ Organisation. Blood 2007; 109(5):1870–1877. doi:10.1182/blood-2006-06-029850
  7. Collins P, Baudo F, Knoebl P, et al; EACH2 Registry Collaborators. Immunosuppression for acquired hemophilia A: results from the European Acquired Haemophilia Registry (EACH2). Blood 2012; 120(1):47–55. doi:10.1182/blood-2012-02-409185
  8. George TI. Malignant or benign leukocytosis. Hematology Am Soc Hematol Educ Program 2012; 2012:475–484. doi:10.1182/asheducation-2012.1.475
  9. Watters RJ, Liu X, Loughran TP Jr. T-cell and natural killer-cell large granular lymphocyte leukemia neoplasias. Leuk Lymphoma 2011; 52(12):2217–2225. doi:10.3109/10428194.2011.593276
  10. Lamy T, Moignet A, Loughran TP Jr. LGL leukemia: from pathogenesis to treatment. Blood 2017; 129(9):1082–1094. doi:10.1182/blood-2016-08-692590
  11. Zhang D, Loughran TP Jr. Large granular lymphocytic leukemia: molecular pathogenesis, clinical manifestations, and treatment. Hematology Am Soc Hematol Educ Program 2012; 2012:652–659. doi:10.1182/asheducation-2012.1.652
  12. Swerdlow SH, Campo E, Pileri SA, et al. The 2016 revision of the World Health Organization classification of lymphoid neoplasms. Blood 2016; 127(20):2375–2390. doi:10.1182/blood-2016-01-643569
  13. Rose MG, Berliner N. T-cell large granular lymphocyte leukemia and related disorders. Oncologist 2004; 9(3):247–258. pmid:15169980
  14. Bockorny B, Dasanu CA. Autoimmune manifestations in large granular lymphocyte leukemia. Clin Lymphoma Myeloma Leuk 2012; 12(6):400–405. doi:10.1016/j.clml.2012.06.006
  15. Liu X, Loughran TP Jr. The spectrum of large granular lymphocyte leukemia and Felty’s syndrome. Curr Opin Hematol 2011; 18(4):254–259. doi:10.1097/MOH.0b013e32834760fb
  16. Franchini M, Lippi G. Acquired factor V inhibitors: a systematic review. J Thromb Thrombolysis 2011; 31(4):449–457. doi:10.1007/s11239-010-0529-6
  17. Sallah S, Nguyen NP, Abdallah JM, Hanrahan LR. Acquired hemophilia in patients with hematologic malignancies. Arch Pathol Lab Med 2000; 124(5):730–734.
  18. Murphy PW, Brett LK, Verla-Tebit E, Macik BG, Loughran TP Jr. Acquired inhibitors to factor VIII and fibrinogen in the setting of T-cell large granular lymphocyte leukemia: a case report and review of the literature. Blood Coagul Fibrinolysis 2015; 26(2):211–213. doi:10.1097/MBC.0000000000000209
Issue
Cleveland Clinic Journal of Medicine - 86(8)
Issue
Cleveland Clinic Journal of Medicine - 86(8)
Page Number
535-542
Page Number
535-542
Publications
Publications
Topics
Article Type
Display Headline
An unusual cause of bruising
Display Headline
An unusual cause of bruising
Legacy Keywords
Bruising, coagulation disorder, bleeding disorder, activated partial thromboplastin time, aPTT, prothrombin time, PT, factor VIII, factor VIII inhibitor, antibody, acquired hemophilia A, AHA, coagulation assay, mixing study, lymphocytosis, large granular lymphocytes, LGLs, LGL leukemia, Tahani Atieh, Alan Lichtin
Legacy Keywords
Bruising, coagulation disorder, bleeding disorder, activated partial thromboplastin time, aPTT, prothrombin time, PT, factor VIII, factor VIII inhibitor, antibody, acquired hemophilia A, AHA, coagulation assay, mixing study, lymphocytosis, large granular lymphocytes, LGLs, LGL leukemia, Tahani Atieh, Alan Lichtin
Sections
Disallow All Ads
Content Gating
No Gating (article Unlocked/Free)
Alternative CME
Disqus Comments
Default
Gate On Date
Tue, 07/30/2019 - 07:30
Un-Gate On Date
Tue, 07/30/2019 - 07:30
Use ProPublica
CFC Schedule Remove Status
Tue, 07/30/2019 - 07:30
Hide sidebar & use full width
render the right sidebar.
Article PDF Media

A 69-year-old woman with double vision and lower-extremity weakness

Article Type
Changed
Mon, 06/03/2019 - 08:10
Display Headline
A 69-year-old woman with double vision and lower-extremity weakness

A 69-year-old woman was admitted to the hospital with double vision, weakness in the lower extremities, sensory loss, pain, and falls. Her symptoms started with sudden onset of horizontal diplopia 6 weeks before, followed by gradually worsening lower-extremity weakness, as well as ataxia and patchy and bilateral radicular burning leg pain more pronounced on the right. Her medical history included narcolepsy, obstructive sleep apnea, hypertension, hyperlipidemia, and bilateral knee replacements for osteoarthritis.

Neurologic examination showed inability to abduct the right eye, bilateral hip flexion weakness, decreased pinprick response, decreased proprioception, and diminished muscle stretch reflexes in the lower extremities. Magnetic resonance imaging (MRI) of the brain without contrast and magnetic resonance angiography of the brain and carotid arteries showed no evidence of acute stroke. No abnormalities were noted on electrocardiography and echocardiography.

A diagnosis of idiopathic peripheral neuropathy was made, and outpatient physical therapy was recommended. Over the subsequent 2 weeks, her condition declined to the point where she needed a walker. She continued to have worsening leg weakness with falls, prompting hospital readmission.

INITIAL EVALUATION

In addition to her diplopia and weakness, she said she had lost 15 pounds since the onset of symptoms and had experienced symptoms suggesting urinary retention.

Physical examination

Her temperature was 37°C (98.6°F), heart rate 79 beats per minute, blood pressure 117/86 mm Hg, respiratory rate 14 breaths per minute, and oxygen saturation 98% on room air. Examination of the head, neck, heart, lung, abdomen, lymph nodes, and extremities yielded nothing remarkable except for chronic venous changes in the lower extremities.

The neurologic examination showed incomplete lateral gaze bilaterally (cranial nerve VI dysfunction). Strength in the upper extremities was normal. In the legs, the Medical Research Council scale score for proximal muscle strength was 2 to 3 out of 5, and for distal muscles 3 to 4 out of 5, with the right side worse than the left and flexors and extensors affected equally. Muscle stretch reflexes were absent in both lower extremities and the left upper extremity, but intact in the right upper extremity. No abnormal corticospinal tract reflexes were elicited.

Sensory testing revealed diminished pin-prick perception in a length-dependent fashion in the lower extremities, reduced 50% compared with the hands. Gait could not be assessed due to weakness.

Initial laboratory testing

Results of initial laboratory tests—complete blood cell count, complete metabolic panel, erythrocyte sedimentation rate, C-reactive protein, thyroid-stimulating hormone, and hemoglobin A1c—were unremarkable.

 

 

FURTHER EVALUATION AND DIFFERENTIAL DIAGNOSIS

1. Which of the following is the most likely diagnosis at this point?

  • Cerebral infarction
  • Guillain-Barré syndrome
  • Progressive polyneuropathy
  • Transverse myelitis
  • Polyradiculopathy

In the absence of definitive diagnostic tests, all of the above options were considered in the differential diagnosis for this patient.

Cerebral infarction

Although acute-onset diplopia can be explained by brainstem stroke involving cranial nerve nuclei or their projections, the onset of diplopia with progressive bilateral lower-extremity weakness makes stroke unlikely. Flaccid paralysis, areflexia of the lower extremities, and sensory involvement can also be caused by acute anterior spinal artery occlusion leading to spinal cord infarction; however, the deficits are usually maximal at onset.

Guillain-Barré syndrome

The combination of acute-subacute progressive ascending weakness, sensory involvement, and diminished or absent reflexes is typical of Guillain-Barré syndrome. Cranial nerve involvement can overlap with the more typical features of the syndrome. However, most patients reach the nadir of their disease by 4 weeks after initial symptom onset, even without treatment.1 This patient’s condition continued to worsen over 8 weeks. In addition, the asymmetric lower-extremity weakness and sparing of the arms are atypical for Guillain-Barré syndrome.

Given the progression of symptoms, chronic inflammatory demyelinating polyneuropathy is also a consideration, typically presenting as a relapsing or progressive neuropathy in proximal and distal muscles and worsening over at least an 8-week period.2

The initial workup for Guillain-Barré syndrome or chronic inflammatory demyelinating polyneuropathy includes lumbar puncture to assess for albuminocytologic dissociation (elevated protein with normal white blood cell count) in cerebrospinal fluid (CSF), and electromyography (EMG) to assess for neuro­physiologic evidence of peripheral nerve demyelination. In Miller-Fisher syndrome, a rare variant of Guillain-Barré syndrome characterized by ataxia, ophthalmoparesis, and areflexia, serum ganglioside antibodies to GQ1b are found in over 90% of patients.3,4 Although MRI of the spine is not necessary to diagnose Guillain-Barré syndrome, it is often done to exclude other causes of lower-extremity weakness such as spinal cord or cauda equina compression that would require urgent neurosurgical consultation. MRI can support the diagnosis of Guillain-Barré syndrome when it reveals enhancement of the spinal nerve roots or cauda equina.

Other polyneuropathies

Polyneuropathy is caused by a variety of diseases that affect the function of peripheral motor, sensory, or autonomic nerves. The differential diagnosis is broad and involves inflammatory diseases (including autoimmune and paraneoplastic causes), hereditary disorders, infection, toxicity, and ischemic and nutritional deficiencies.5 Polyneuropathy can present in a distal-predominant, generalized, or asymmetric pattern involving individual nerve trunks termed “mononeuropathy multiplex,” as in our patient’s presentation. The initial workup includes EMG and a battery of serologic tests. In cases of severe and progressive polyneuropathy, nerve biopsy can assess for the presence of vasculitis, amyloidosis, and paraprotein deposition.

Transverse myelitis

Transverse myelitis is an inflammatory myelopathy that usually presents with acute or subacute weakness of the upper extremities or lower extremities, or both, corresponding to the level of the lesion, hyperreflexia, bladder and bowel dysfunction, spinal level of sensory loss, and autonomic involvement.6 The differential diagnosis of acute myelopathy includes:

  • Infection (eg, herpes simplex virus, West Nile virus, Lyme disease, Mycoplasma pneumoniae, human immunodeficiency virus)
  • Systemic inflammatory disease (systemic lupus erythematosus, sarcoidosis, Sjögren syndrome, scleroderma, paraneoplastic syndrome)
  • Central nervous system demyelinating disease (acute disseminated encephalomyelitis, multiple sclerosis, neuromyelitis optica)
  • Vascular malformation (dural arteriovenous fistula)
  • Compression due to tumor, bleeding, disc herniation, infection, or abscess.

The workup involves laboratory tests to exclude systemic inflammatory and infectious causes, as well as MRI of the spine with and without contrast to identify a causative lesion. Lumbar puncture and CSF analysis may show pleocytosis, elevated protein concentration, and increased intrathecal immunoglobulin G (IgG) index.7

Although our patient’s presentation with subacute lower-extremity weakness, sensory changes, and bladder dysfunction were consistent with transverse myelitis, her cranial nerve abnormalities would be atypical for it.

Polyradiculopathy


Polyradiculopathy has many possible causes. In the United States, the most common causes are lumbar spondylosis, lumbar canal stenosis, and diabetic polyradiculoneuropathy.

When multiple spinal segments are affected, leptomeningeal disease involving the arachnoid and pia mater should be considered. Causes include malignant invasion, inflammatory cell accumulation, and protein deposition, leading to patchy but widespread dysfunction of spinal nerve roots and cranial nerves. Specific causes are myriad and include carcinomatous meningitis,8 syphilis, tuberculosis, sarcoidosis, and paraproteinemias. CSF and MRI changes are often nonspecific, leading to the need for meningeal biopsy for diagnosis.

 

 

CASE CONTINUED

During her hospitalization, our patient developed acute right upper and lower facial weakness consistent with peripheral facial mononeuropathy. Bilateral lower-extremity weakness progressed to disabling paraparesis.

migdady_diplopia_t1.jpg

She underwent lumbar puncture and CSF analysis (Table 1). The most notable findings were significant pleocytosis (72% lymphocytic predominance), protein elevation, and elevated IgG index (indicative of elevated intrathecal immunoglobulin synthesis in the central nervous system). Viral, bacterial, and fungal studies were negative. Guillain-Barré syndrome, other polyneuropathies, and spinal cord infarction would not be expected with these CSF features.

Surface EMG demonstrated normal sensory responses, and needle EMG showed chronic and active motor axon loss in the L3 and S1 root distributions, suggesting polyradiculopathy without polyneuropathy. These findings would not be expected in typical acute transverse myelitis but could be seen with spinal cord infarction.

migdady_diplopia_f1.jpg
%3Cp%3EFigure%201.%20Magnetic%20resonance%20imaging%20of%20the%20lumbar%20spine%20with%20contrast%20showed%20cauda%20equina%20enhancement%20at%20level%20L5%20to%20S1%20(arrows)%20in%20axial%20T1%20sequence%20(top)%20and%20sagittal%20T1%20sequence%20(bottom).%3C%2Fp%3E

MRI of the entire spine with and without contrast showed cauda equina nerve root thickening and enhancement, especially involving the L5 and S1 roots (Figure 1). The spinal cord appeared normal. These findings further supported polyradiculopathy and a leptomeningeal process.

Further evaluation included chest radiography, erythrocyte sedimentation rate, C-reactive protein, hemoglobin A1c, human immunodeficiency virus testing, antinuclear antibody, antineutrophil cytoplasmic antibody, extractable nuclear antibody, GQ1b antibody, serum and CSF paraneoplastic panels, levels of vitamin B1, B12, and B6, copper, and ceruloplasmin, and a screen for heavy metals. All results were within normal ranges.

ESTABLISHING THE DIAGNOSIS

Serum monoclonal protein analysis with immunofixation revealed IgM kappa monoclonal gammopathy with an IgM level of 1,570 (reference range 53–334 mg/dL) and M-spike 0.75 (0.00 mg/dL), serum free kappa light chains 61.1 (3.30–19.40 mg/L), lambda 9.3 (5.7–26.3 mg/L), and kappa-lambda ratio 6.57 (0.26–1.65).

2. Which is the best next step in this patient’s neurologic evaluation?

  • Test CSF angiotensin-converting enzyme level
  • CSF cytology
  • Meningeal biopsy
  • Peripheral nerve biopsy

Given the high suspicion for malignancy, CSF cytology was performed and showed increased numbers of mononuclear chronic inflammatory cells, including a mixture of lymphocytes and monocytes, favoring a reactive lymphoid pleocytosis. Flow cytometry indicated the presence of a monoclonal, CD5- and CD10- negative, B-cell lymphoproliferative disorder. The immunophenotypic findings were not specific for a single diagnosis. The differential diagnosis included marginal zone lymphoma and lymphoplasmacytic lymphoma.

3. Given the presence of serum IgM monoclonal gammopathy in this patient, which is the most likely diagnosis?

  • Neurosarcoidosis
  • Multiple myeloma
  • Waldenström macroglobulinemia
  • Carcinomatous meningitis

Study of bone marrow biopsy demonstrated limited bone marrow involvement (1%) by a lymphoproliferative disorder with plasmacytoid features, and DNA testing detected an MYD88 L265P mutation, reported to be present in 90% of patients with Waldenström macroglobulinemia.9 This finding confirmed the diagnosis of Waldenström macroglobulinemia with central nervous system involvement. Our patient began therapy with rituximab and methotrexate, which resulted in some improvement in strength, gait, and vision.

 

 

WALDENSTRÖM MACROGLOBULINEMIA AND BING-NEEL SYNDROME

Waldenström macroglobulinemia is a lympho­plasmacytic lymphoma associated with a monoclonal IgM protein.10 It is considered a paraproteinemic disorder, similar to multiple myeloma. The presenting symptoms and complications are related to direct tumor infiltration, hyperviscosity syndrome, and deposition of IgM in various tissues.11,12

Waldenström macroglobulinemia is usually indolent, and treatment is reserved for patients with symptoms.13,14 It includes rituximab, usually in combination with chemotherapy or other targeted agents.15,16

Paraneoplastic antibody-mediated polyneuropathy may occur in these patients. However, the pattern is usually symmetrical clinically, with demyelination on EMG, and is not associated with cranial nerve or meningeal involvement. Management with plasmapheresis, corticosteroids, and intravenous immunoglobulin has not been shown to be effective.17

Involvement of the central nervous system as a complication of Waldenström macroglobulinemia has been described as Bing-Neel syndrome. It can present as diffuse malignant cell infiltration of the leptomeningeal space, white matter, or spinal cord, or in a tumoral form presenting as intraparenchymal masses or nodular lesions. The distinction between the tumoral and diffuse forms is based primarily on imaging findings.18

In a report of 44 patients with Bing-Neel syndrome, 36% presented with the disorder as the initial manifestation of Waldenström macroglobulinemia.18 The primary presenting symptoms were imbalance and gait difficulty (48%) and cranial nerve involvement (36%), which presented as predominantly facial or oculomotor nerve palsy. Cauda equina syndrome with motor involvement (seen in our patient) occurred in 14% of patients. Other presenting symptoms included cognitive impairment, sensory deficits, headache, dysarthria, aphasia, and seizures.

LEARNING POINTS

The differential diagnosis for patients presenting with multifocal neurologic symptoms can be broad, and a systematic approach to the diagnosis is necessary. Localizing the lesion is important in determining the diagnosis for patients presenting with neurologic symptoms. The process of localization begins with taking the history, is further refined during the examination, and is confirmed with diagnostic studies. Atypical presentations of relatively common neurologic diseases such as Guillain-Barré syndrome, transverse myelitis, and peripheral polyneuropathy do occur, but uncommon diagnoses need to be considered when support for the initial diagnosis is lacking.

References
  1. Fokke C, van den Berg B, Drenthen J, Walgaard C, van Doorn PA, Jacobs BC. Diagnosis of Guillain-Barre syndrome and validation of Brighton criteria. Brain 2014; 137(Pt 1):33–43. doi:10.1093/brain/awt285
  2. Mathey EK, Park SB, Hughes RA, et al. Chronic inflammatory demyelinating polyradiculoneuropathy: from pathology to phenotype. J Neurol Neurosurg Psychiatry 2015; 86(9):973–985. doi:10.1136/jnnp-2014-309697
  3. Chiba A, Kusunoki S, Obata H, Machinami R, Kanazawa I. Serum anti-GQ1b IgG antibody is associated with ophthalmoplegia in Miller Fisher syndrome and Guillain-Barré syndrome: clinical and immunohistochemical studies. Neurology 1993; 43(10):1911–1917. pmid:8413947
  4. Teener J. Miller Fisher’s syndrome. Semin Neurol 2012; 32(5):512–516. doi:10.1055/s-0033-1334470
  5. Watson JC, Dyck PJ. Peripheral neuropathy: a practical approach to diagnosis and symptom management. Mayo Clin Proc 2015; 90(7):940–951. doi:10.1016/j.mayocp.2015.05.004
  6. Greenberg BM. Treatment of acute transverse myelitis and its early complications. Continuum (Minneap Minn) 2011; 17(4):733–743. doi:10.1212/01.CON.0000403792.36161.f5
  7. West TW. Transverse myelitis—a review of the presentation, diagnosis, and initial management. Discov Med 2013; 16(88):167–177. pmid:24099672
  8. Le Rhun E, Taillibert S, Chamberlain MC. Carcinomatous meningitis: leptomeningeal metastases in solid tumors. Surg Neurol Int 2013; 4(suppl 4):S265–S288. doi:10.4103/2152-7806.111304
  9. Treon SP, Xu L, Yang G, et al. MYD88 L265P somatic mutation in Waldenström’s macroglobulinemia. N Engl J Med 2012; 367(9):826–833. doi:10.1056/NEJMoa1200710
  10. Owen RG, Treon SP, Al-Katib A, et al. Clinicopathological definition of Waldenstrom’s macroglobulinemia: consensus panel recommendations from the Second International Workshop on Waldenstrom’s Macroglobulinemia. Semin Oncol 2003; 30(2):110–115. doi:10.1053/sonc.2003.50082
  11. Björkholm M, Johansson E, Papamichael D, et al. Patterns of clinical presentation, treatment, and outcome in patients with Waldenstrom’s macroglobulinemia: a two-institution study. Semin Oncol 2003; 30(2):226–230. doi:10.1053/sonc.2003.50054
  12. Rison RA, Beydoun SR. Paraproteinemic neuropathy: a practical review. BMC Neurol 2016; 16:13. doi:10.1186/s12883-016-0532-4
  13. Kyle RA, Benson J, Larson D, et al. IgM monoclonal gammopathy of undetermined significance and smoldering Waldenström’s macroglobulinemia. Clin Lymphoma Myeloma 2009; 9(1):17–18. doi:10.3816/CLM.2009.n.002
  14. Kyle RA, Benson JT, Larson DR, et al. Progression in smoldering Waldenstrom macroglobulinemia: long-term results. Blood 2012; 119(19):4462–4466. doi:10.1182/blood-2011-10-384768
  15. Leblond V, Kastritis E, Advani R, et al. Treatment recommendations from the Eighth International Workshop on Waldenström’s macroglobulinemia. Blood 2016; 128(10):1321–1328. doi:10.1182/blood-2016-04-711234
  16. Kapoor P, Ansell SM, Fonseca R, et al. Diagnosis and management of Waldenström macroglobulinemia: Mayo stratification of macroglobulinemia and risk-adapted therapy (mSMART) guidelines 2016. JAMA Oncol 2017; 3(9):1257–1265. doi:10.1001/jamaoncol.2016.5763
  17. D’Sa S, Kersten MJ, Castillo JJ, et al. Investigation and management of IgM and Waldenström-associated peripheral neuropathies: recommendations from the IWWM-8 consensus panel. Br J Haematol 2017; 176(5):728–742. doi:10.1111/bjh.14492
  18. Simon L, Fitsiori A, Lemal R, et al. Bing-Neel syndrome, a rare complication of Waldenström macroglobulinemia: analysis of 44 cases and review of the literature. A study on behalf of the French Innovative Leukemia Organization (FILO). Haematologica 2015; 100(12):1587–1594. doi:10.3324/haematol.2015.133744
Article PDF
Author and Disclosure Information

Ibrahim Migdady, MD
Department of Neurology, Neurological Institute, Cleveland Clinic

MaryAnn Mays, MD
Department of Neurology, Neurological Institute, Cleveland Clinic

Kerry H. Levin, MD
Chair, Department of Neurology, and Director, Neuromuscular Center, Neurological Institute, Cleveland Clinic

Address: Kerry H. Levin, MD, Department of Neurology, Neurological Institute, S90, Cleveland Clinic, 9500 Euclid Avenue, Cleveland, OH 44195; levink@ccf.org

Dr. Mays has disclosed teaching and speaking for Allergan, Amgen, and Teva.

Issue
Cleveland Clinic Journal of Medicine - 86(6)
Publications
Topics
Page Number
374-379
Legacy Keywords
double vision, diplopia, weakness, cerebral infarction, stroke, Guillain-Baré syndrome, GBS, neuropathy, polyneuropathy, transverse myelitis, radiculopathy, monoclonal gammopathy, neurosarcoidosis, multiplemyeloma, Waldenström macroglobulinemia, Bing-Neel syndrome, Ibrahim Migdady, Maryann Mays, Kerry Levin
Sections
Author and Disclosure Information

Ibrahim Migdady, MD
Department of Neurology, Neurological Institute, Cleveland Clinic

MaryAnn Mays, MD
Department of Neurology, Neurological Institute, Cleveland Clinic

Kerry H. Levin, MD
Chair, Department of Neurology, and Director, Neuromuscular Center, Neurological Institute, Cleveland Clinic

Address: Kerry H. Levin, MD, Department of Neurology, Neurological Institute, S90, Cleveland Clinic, 9500 Euclid Avenue, Cleveland, OH 44195; levink@ccf.org

Dr. Mays has disclosed teaching and speaking for Allergan, Amgen, and Teva.

Author and Disclosure Information

Ibrahim Migdady, MD
Department of Neurology, Neurological Institute, Cleveland Clinic

MaryAnn Mays, MD
Department of Neurology, Neurological Institute, Cleveland Clinic

Kerry H. Levin, MD
Chair, Department of Neurology, and Director, Neuromuscular Center, Neurological Institute, Cleveland Clinic

Address: Kerry H. Levin, MD, Department of Neurology, Neurological Institute, S90, Cleveland Clinic, 9500 Euclid Avenue, Cleveland, OH 44195; levink@ccf.org

Dr. Mays has disclosed teaching and speaking for Allergan, Amgen, and Teva.

Article PDF
Article PDF
Related Articles

A 69-year-old woman was admitted to the hospital with double vision, weakness in the lower extremities, sensory loss, pain, and falls. Her symptoms started with sudden onset of horizontal diplopia 6 weeks before, followed by gradually worsening lower-extremity weakness, as well as ataxia and patchy and bilateral radicular burning leg pain more pronounced on the right. Her medical history included narcolepsy, obstructive sleep apnea, hypertension, hyperlipidemia, and bilateral knee replacements for osteoarthritis.

Neurologic examination showed inability to abduct the right eye, bilateral hip flexion weakness, decreased pinprick response, decreased proprioception, and diminished muscle stretch reflexes in the lower extremities. Magnetic resonance imaging (MRI) of the brain without contrast and magnetic resonance angiography of the brain and carotid arteries showed no evidence of acute stroke. No abnormalities were noted on electrocardiography and echocardiography.

A diagnosis of idiopathic peripheral neuropathy was made, and outpatient physical therapy was recommended. Over the subsequent 2 weeks, her condition declined to the point where she needed a walker. She continued to have worsening leg weakness with falls, prompting hospital readmission.

INITIAL EVALUATION

In addition to her diplopia and weakness, she said she had lost 15 pounds since the onset of symptoms and had experienced symptoms suggesting urinary retention.

Physical examination

Her temperature was 37°C (98.6°F), heart rate 79 beats per minute, blood pressure 117/86 mm Hg, respiratory rate 14 breaths per minute, and oxygen saturation 98% on room air. Examination of the head, neck, heart, lung, abdomen, lymph nodes, and extremities yielded nothing remarkable except for chronic venous changes in the lower extremities.

The neurologic examination showed incomplete lateral gaze bilaterally (cranial nerve VI dysfunction). Strength in the upper extremities was normal. In the legs, the Medical Research Council scale score for proximal muscle strength was 2 to 3 out of 5, and for distal muscles 3 to 4 out of 5, with the right side worse than the left and flexors and extensors affected equally. Muscle stretch reflexes were absent in both lower extremities and the left upper extremity, but intact in the right upper extremity. No abnormal corticospinal tract reflexes were elicited.

Sensory testing revealed diminished pin-prick perception in a length-dependent fashion in the lower extremities, reduced 50% compared with the hands. Gait could not be assessed due to weakness.

Initial laboratory testing

Results of initial laboratory tests—complete blood cell count, complete metabolic panel, erythrocyte sedimentation rate, C-reactive protein, thyroid-stimulating hormone, and hemoglobin A1c—were unremarkable.

 

 

FURTHER EVALUATION AND DIFFERENTIAL DIAGNOSIS

1. Which of the following is the most likely diagnosis at this point?

  • Cerebral infarction
  • Guillain-Barré syndrome
  • Progressive polyneuropathy
  • Transverse myelitis
  • Polyradiculopathy

In the absence of definitive diagnostic tests, all of the above options were considered in the differential diagnosis for this patient.

Cerebral infarction

Although acute-onset diplopia can be explained by brainstem stroke involving cranial nerve nuclei or their projections, the onset of diplopia with progressive bilateral lower-extremity weakness makes stroke unlikely. Flaccid paralysis, areflexia of the lower extremities, and sensory involvement can also be caused by acute anterior spinal artery occlusion leading to spinal cord infarction; however, the deficits are usually maximal at onset.

Guillain-Barré syndrome

The combination of acute-subacute progressive ascending weakness, sensory involvement, and diminished or absent reflexes is typical of Guillain-Barré syndrome. Cranial nerve involvement can overlap with the more typical features of the syndrome. However, most patients reach the nadir of their disease by 4 weeks after initial symptom onset, even without treatment.1 This patient’s condition continued to worsen over 8 weeks. In addition, the asymmetric lower-extremity weakness and sparing of the arms are atypical for Guillain-Barré syndrome.

Given the progression of symptoms, chronic inflammatory demyelinating polyneuropathy is also a consideration, typically presenting as a relapsing or progressive neuropathy in proximal and distal muscles and worsening over at least an 8-week period.2

The initial workup for Guillain-Barré syndrome or chronic inflammatory demyelinating polyneuropathy includes lumbar puncture to assess for albuminocytologic dissociation (elevated protein with normal white blood cell count) in cerebrospinal fluid (CSF), and electromyography (EMG) to assess for neuro­physiologic evidence of peripheral nerve demyelination. In Miller-Fisher syndrome, a rare variant of Guillain-Barré syndrome characterized by ataxia, ophthalmoparesis, and areflexia, serum ganglioside antibodies to GQ1b are found in over 90% of patients.3,4 Although MRI of the spine is not necessary to diagnose Guillain-Barré syndrome, it is often done to exclude other causes of lower-extremity weakness such as spinal cord or cauda equina compression that would require urgent neurosurgical consultation. MRI can support the diagnosis of Guillain-Barré syndrome when it reveals enhancement of the spinal nerve roots or cauda equina.

Other polyneuropathies

Polyneuropathy is caused by a variety of diseases that affect the function of peripheral motor, sensory, or autonomic nerves. The differential diagnosis is broad and involves inflammatory diseases (including autoimmune and paraneoplastic causes), hereditary disorders, infection, toxicity, and ischemic and nutritional deficiencies.5 Polyneuropathy can present in a distal-predominant, generalized, or asymmetric pattern involving individual nerve trunks termed “mononeuropathy multiplex,” as in our patient’s presentation. The initial workup includes EMG and a battery of serologic tests. In cases of severe and progressive polyneuropathy, nerve biopsy can assess for the presence of vasculitis, amyloidosis, and paraprotein deposition.

Transverse myelitis

Transverse myelitis is an inflammatory myelopathy that usually presents with acute or subacute weakness of the upper extremities or lower extremities, or both, corresponding to the level of the lesion, hyperreflexia, bladder and bowel dysfunction, spinal level of sensory loss, and autonomic involvement.6 The differential diagnosis of acute myelopathy includes:

  • Infection (eg, herpes simplex virus, West Nile virus, Lyme disease, Mycoplasma pneumoniae, human immunodeficiency virus)
  • Systemic inflammatory disease (systemic lupus erythematosus, sarcoidosis, Sjögren syndrome, scleroderma, paraneoplastic syndrome)
  • Central nervous system demyelinating disease (acute disseminated encephalomyelitis, multiple sclerosis, neuromyelitis optica)
  • Vascular malformation (dural arteriovenous fistula)
  • Compression due to tumor, bleeding, disc herniation, infection, or abscess.

The workup involves laboratory tests to exclude systemic inflammatory and infectious causes, as well as MRI of the spine with and without contrast to identify a causative lesion. Lumbar puncture and CSF analysis may show pleocytosis, elevated protein concentration, and increased intrathecal immunoglobulin G (IgG) index.7

Although our patient’s presentation with subacute lower-extremity weakness, sensory changes, and bladder dysfunction were consistent with transverse myelitis, her cranial nerve abnormalities would be atypical for it.

Polyradiculopathy


Polyradiculopathy has many possible causes. In the United States, the most common causes are lumbar spondylosis, lumbar canal stenosis, and diabetic polyradiculoneuropathy.

When multiple spinal segments are affected, leptomeningeal disease involving the arachnoid and pia mater should be considered. Causes include malignant invasion, inflammatory cell accumulation, and protein deposition, leading to patchy but widespread dysfunction of spinal nerve roots and cranial nerves. Specific causes are myriad and include carcinomatous meningitis,8 syphilis, tuberculosis, sarcoidosis, and paraproteinemias. CSF and MRI changes are often nonspecific, leading to the need for meningeal biopsy for diagnosis.

 

 

CASE CONTINUED

During her hospitalization, our patient developed acute right upper and lower facial weakness consistent with peripheral facial mononeuropathy. Bilateral lower-extremity weakness progressed to disabling paraparesis.

migdady_diplopia_t1.jpg

She underwent lumbar puncture and CSF analysis (Table 1). The most notable findings were significant pleocytosis (72% lymphocytic predominance), protein elevation, and elevated IgG index (indicative of elevated intrathecal immunoglobulin synthesis in the central nervous system). Viral, bacterial, and fungal studies were negative. Guillain-Barré syndrome, other polyneuropathies, and spinal cord infarction would not be expected with these CSF features.

Surface EMG demonstrated normal sensory responses, and needle EMG showed chronic and active motor axon loss in the L3 and S1 root distributions, suggesting polyradiculopathy without polyneuropathy. These findings would not be expected in typical acute transverse myelitis but could be seen with spinal cord infarction.

migdady_diplopia_f1.jpg
%3Cp%3EFigure%201.%20Magnetic%20resonance%20imaging%20of%20the%20lumbar%20spine%20with%20contrast%20showed%20cauda%20equina%20enhancement%20at%20level%20L5%20to%20S1%20(arrows)%20in%20axial%20T1%20sequence%20(top)%20and%20sagittal%20T1%20sequence%20(bottom).%3C%2Fp%3E

MRI of the entire spine with and without contrast showed cauda equina nerve root thickening and enhancement, especially involving the L5 and S1 roots (Figure 1). The spinal cord appeared normal. These findings further supported polyradiculopathy and a leptomeningeal process.

Further evaluation included chest radiography, erythrocyte sedimentation rate, C-reactive protein, hemoglobin A1c, human immunodeficiency virus testing, antinuclear antibody, antineutrophil cytoplasmic antibody, extractable nuclear antibody, GQ1b antibody, serum and CSF paraneoplastic panels, levels of vitamin B1, B12, and B6, copper, and ceruloplasmin, and a screen for heavy metals. All results were within normal ranges.

ESTABLISHING THE DIAGNOSIS

Serum monoclonal protein analysis with immunofixation revealed IgM kappa monoclonal gammopathy with an IgM level of 1,570 (reference range 53–334 mg/dL) and M-spike 0.75 (0.00 mg/dL), serum free kappa light chains 61.1 (3.30–19.40 mg/L), lambda 9.3 (5.7–26.3 mg/L), and kappa-lambda ratio 6.57 (0.26–1.65).

2. Which is the best next step in this patient’s neurologic evaluation?

  • Test CSF angiotensin-converting enzyme level
  • CSF cytology
  • Meningeal biopsy
  • Peripheral nerve biopsy

Given the high suspicion for malignancy, CSF cytology was performed and showed increased numbers of mononuclear chronic inflammatory cells, including a mixture of lymphocytes and monocytes, favoring a reactive lymphoid pleocytosis. Flow cytometry indicated the presence of a monoclonal, CD5- and CD10- negative, B-cell lymphoproliferative disorder. The immunophenotypic findings were not specific for a single diagnosis. The differential diagnosis included marginal zone lymphoma and lymphoplasmacytic lymphoma.

3. Given the presence of serum IgM monoclonal gammopathy in this patient, which is the most likely diagnosis?

  • Neurosarcoidosis
  • Multiple myeloma
  • Waldenström macroglobulinemia
  • Carcinomatous meningitis

Study of bone marrow biopsy demonstrated limited bone marrow involvement (1%) by a lymphoproliferative disorder with plasmacytoid features, and DNA testing detected an MYD88 L265P mutation, reported to be present in 90% of patients with Waldenström macroglobulinemia.9 This finding confirmed the diagnosis of Waldenström macroglobulinemia with central nervous system involvement. Our patient began therapy with rituximab and methotrexate, which resulted in some improvement in strength, gait, and vision.

 

 

WALDENSTRÖM MACROGLOBULINEMIA AND BING-NEEL SYNDROME

Waldenström macroglobulinemia is a lympho­plasmacytic lymphoma associated with a monoclonal IgM protein.10 It is considered a paraproteinemic disorder, similar to multiple myeloma. The presenting symptoms and complications are related to direct tumor infiltration, hyperviscosity syndrome, and deposition of IgM in various tissues.11,12

Waldenström macroglobulinemia is usually indolent, and treatment is reserved for patients with symptoms.13,14 It includes rituximab, usually in combination with chemotherapy or other targeted agents.15,16

Paraneoplastic antibody-mediated polyneuropathy may occur in these patients. However, the pattern is usually symmetrical clinically, with demyelination on EMG, and is not associated with cranial nerve or meningeal involvement. Management with plasmapheresis, corticosteroids, and intravenous immunoglobulin has not been shown to be effective.17

Involvement of the central nervous system as a complication of Waldenström macroglobulinemia has been described as Bing-Neel syndrome. It can present as diffuse malignant cell infiltration of the leptomeningeal space, white matter, or spinal cord, or in a tumoral form presenting as intraparenchymal masses or nodular lesions. The distinction between the tumoral and diffuse forms is based primarily on imaging findings.18

In a report of 44 patients with Bing-Neel syndrome, 36% presented with the disorder as the initial manifestation of Waldenström macroglobulinemia.18 The primary presenting symptoms were imbalance and gait difficulty (48%) and cranial nerve involvement (36%), which presented as predominantly facial or oculomotor nerve palsy. Cauda equina syndrome with motor involvement (seen in our patient) occurred in 14% of patients. Other presenting symptoms included cognitive impairment, sensory deficits, headache, dysarthria, aphasia, and seizures.

LEARNING POINTS

The differential diagnosis for patients presenting with multifocal neurologic symptoms can be broad, and a systematic approach to the diagnosis is necessary. Localizing the lesion is important in determining the diagnosis for patients presenting with neurologic symptoms. The process of localization begins with taking the history, is further refined during the examination, and is confirmed with diagnostic studies. Atypical presentations of relatively common neurologic diseases such as Guillain-Barré syndrome, transverse myelitis, and peripheral polyneuropathy do occur, but uncommon diagnoses need to be considered when support for the initial diagnosis is lacking.

A 69-year-old woman was admitted to the hospital with double vision, weakness in the lower extremities, sensory loss, pain, and falls. Her symptoms started with sudden onset of horizontal diplopia 6 weeks before, followed by gradually worsening lower-extremity weakness, as well as ataxia and patchy and bilateral radicular burning leg pain more pronounced on the right. Her medical history included narcolepsy, obstructive sleep apnea, hypertension, hyperlipidemia, and bilateral knee replacements for osteoarthritis.

Neurologic examination showed inability to abduct the right eye, bilateral hip flexion weakness, decreased pinprick response, decreased proprioception, and diminished muscle stretch reflexes in the lower extremities. Magnetic resonance imaging (MRI) of the brain without contrast and magnetic resonance angiography of the brain and carotid arteries showed no evidence of acute stroke. No abnormalities were noted on electrocardiography and echocardiography.

A diagnosis of idiopathic peripheral neuropathy was made, and outpatient physical therapy was recommended. Over the subsequent 2 weeks, her condition declined to the point where she needed a walker. She continued to have worsening leg weakness with falls, prompting hospital readmission.

INITIAL EVALUATION

In addition to her diplopia and weakness, she said she had lost 15 pounds since the onset of symptoms and had experienced symptoms suggesting urinary retention.

Physical examination

Her temperature was 37°C (98.6°F), heart rate 79 beats per minute, blood pressure 117/86 mm Hg, respiratory rate 14 breaths per minute, and oxygen saturation 98% on room air. Examination of the head, neck, heart, lung, abdomen, lymph nodes, and extremities yielded nothing remarkable except for chronic venous changes in the lower extremities.

The neurologic examination showed incomplete lateral gaze bilaterally (cranial nerve VI dysfunction). Strength in the upper extremities was normal. In the legs, the Medical Research Council scale score for proximal muscle strength was 2 to 3 out of 5, and for distal muscles 3 to 4 out of 5, with the right side worse than the left and flexors and extensors affected equally. Muscle stretch reflexes were absent in both lower extremities and the left upper extremity, but intact in the right upper extremity. No abnormal corticospinal tract reflexes were elicited.

Sensory testing revealed diminished pin-prick perception in a length-dependent fashion in the lower extremities, reduced 50% compared with the hands. Gait could not be assessed due to weakness.

Initial laboratory testing

Results of initial laboratory tests—complete blood cell count, complete metabolic panel, erythrocyte sedimentation rate, C-reactive protein, thyroid-stimulating hormone, and hemoglobin A1c—were unremarkable.

 

 

FURTHER EVALUATION AND DIFFERENTIAL DIAGNOSIS

1. Which of the following is the most likely diagnosis at this point?

  • Cerebral infarction
  • Guillain-Barré syndrome
  • Progressive polyneuropathy
  • Transverse myelitis
  • Polyradiculopathy

In the absence of definitive diagnostic tests, all of the above options were considered in the differential diagnosis for this patient.

Cerebral infarction

Although acute-onset diplopia can be explained by brainstem stroke involving cranial nerve nuclei or their projections, the onset of diplopia with progressive bilateral lower-extremity weakness makes stroke unlikely. Flaccid paralysis, areflexia of the lower extremities, and sensory involvement can also be caused by acute anterior spinal artery occlusion leading to spinal cord infarction; however, the deficits are usually maximal at onset.

Guillain-Barré syndrome

The combination of acute-subacute progressive ascending weakness, sensory involvement, and diminished or absent reflexes is typical of Guillain-Barré syndrome. Cranial nerve involvement can overlap with the more typical features of the syndrome. However, most patients reach the nadir of their disease by 4 weeks after initial symptom onset, even without treatment.1 This patient’s condition continued to worsen over 8 weeks. In addition, the asymmetric lower-extremity weakness and sparing of the arms are atypical for Guillain-Barré syndrome.

Given the progression of symptoms, chronic inflammatory demyelinating polyneuropathy is also a consideration, typically presenting as a relapsing or progressive neuropathy in proximal and distal muscles and worsening over at least an 8-week period.2

The initial workup for Guillain-Barré syndrome or chronic inflammatory demyelinating polyneuropathy includes lumbar puncture to assess for albuminocytologic dissociation (elevated protein with normal white blood cell count) in cerebrospinal fluid (CSF), and electromyography (EMG) to assess for neuro­physiologic evidence of peripheral nerve demyelination. In Miller-Fisher syndrome, a rare variant of Guillain-Barré syndrome characterized by ataxia, ophthalmoparesis, and areflexia, serum ganglioside antibodies to GQ1b are found in over 90% of patients.3,4 Although MRI of the spine is not necessary to diagnose Guillain-Barré syndrome, it is often done to exclude other causes of lower-extremity weakness such as spinal cord or cauda equina compression that would require urgent neurosurgical consultation. MRI can support the diagnosis of Guillain-Barré syndrome when it reveals enhancement of the spinal nerve roots or cauda equina.

Other polyneuropathies

Polyneuropathy is caused by a variety of diseases that affect the function of peripheral motor, sensory, or autonomic nerves. The differential diagnosis is broad and involves inflammatory diseases (including autoimmune and paraneoplastic causes), hereditary disorders, infection, toxicity, and ischemic and nutritional deficiencies.5 Polyneuropathy can present in a distal-predominant, generalized, or asymmetric pattern involving individual nerve trunks termed “mononeuropathy multiplex,” as in our patient’s presentation. The initial workup includes EMG and a battery of serologic tests. In cases of severe and progressive polyneuropathy, nerve biopsy can assess for the presence of vasculitis, amyloidosis, and paraprotein deposition.

Transverse myelitis

Transverse myelitis is an inflammatory myelopathy that usually presents with acute or subacute weakness of the upper extremities or lower extremities, or both, corresponding to the level of the lesion, hyperreflexia, bladder and bowel dysfunction, spinal level of sensory loss, and autonomic involvement.6 The differential diagnosis of acute myelopathy includes:

  • Infection (eg, herpes simplex virus, West Nile virus, Lyme disease, Mycoplasma pneumoniae, human immunodeficiency virus)
  • Systemic inflammatory disease (systemic lupus erythematosus, sarcoidosis, Sjögren syndrome, scleroderma, paraneoplastic syndrome)
  • Central nervous system demyelinating disease (acute disseminated encephalomyelitis, multiple sclerosis, neuromyelitis optica)
  • Vascular malformation (dural arteriovenous fistula)
  • Compression due to tumor, bleeding, disc herniation, infection, or abscess.

The workup involves laboratory tests to exclude systemic inflammatory and infectious causes, as well as MRI of the spine with and without contrast to identify a causative lesion. Lumbar puncture and CSF analysis may show pleocytosis, elevated protein concentration, and increased intrathecal immunoglobulin G (IgG) index.7

Although our patient’s presentation with subacute lower-extremity weakness, sensory changes, and bladder dysfunction were consistent with transverse myelitis, her cranial nerve abnormalities would be atypical for it.

Polyradiculopathy


Polyradiculopathy has many possible causes. In the United States, the most common causes are lumbar spondylosis, lumbar canal stenosis, and diabetic polyradiculoneuropathy.

When multiple spinal segments are affected, leptomeningeal disease involving the arachnoid and pia mater should be considered. Causes include malignant invasion, inflammatory cell accumulation, and protein deposition, leading to patchy but widespread dysfunction of spinal nerve roots and cranial nerves. Specific causes are myriad and include carcinomatous meningitis,8 syphilis, tuberculosis, sarcoidosis, and paraproteinemias. CSF and MRI changes are often nonspecific, leading to the need for meningeal biopsy for diagnosis.

 

 

CASE CONTINUED

During her hospitalization, our patient developed acute right upper and lower facial weakness consistent with peripheral facial mononeuropathy. Bilateral lower-extremity weakness progressed to disabling paraparesis.

migdady_diplopia_t1.jpg

She underwent lumbar puncture and CSF analysis (Table 1). The most notable findings were significant pleocytosis (72% lymphocytic predominance), protein elevation, and elevated IgG index (indicative of elevated intrathecal immunoglobulin synthesis in the central nervous system). Viral, bacterial, and fungal studies were negative. Guillain-Barré syndrome, other polyneuropathies, and spinal cord infarction would not be expected with these CSF features.

Surface EMG demonstrated normal sensory responses, and needle EMG showed chronic and active motor axon loss in the L3 and S1 root distributions, suggesting polyradiculopathy without polyneuropathy. These findings would not be expected in typical acute transverse myelitis but could be seen with spinal cord infarction.

migdady_diplopia_f1.jpg
%3Cp%3EFigure%201.%20Magnetic%20resonance%20imaging%20of%20the%20lumbar%20spine%20with%20contrast%20showed%20cauda%20equina%20enhancement%20at%20level%20L5%20to%20S1%20(arrows)%20in%20axial%20T1%20sequence%20(top)%20and%20sagittal%20T1%20sequence%20(bottom).%3C%2Fp%3E

MRI of the entire spine with and without contrast showed cauda equina nerve root thickening and enhancement, especially involving the L5 and S1 roots (Figure 1). The spinal cord appeared normal. These findings further supported polyradiculopathy and a leptomeningeal process.

Further evaluation included chest radiography, erythrocyte sedimentation rate, C-reactive protein, hemoglobin A1c, human immunodeficiency virus testing, antinuclear antibody, antineutrophil cytoplasmic antibody, extractable nuclear antibody, GQ1b antibody, serum and CSF paraneoplastic panels, levels of vitamin B1, B12, and B6, copper, and ceruloplasmin, and a screen for heavy metals. All results were within normal ranges.

ESTABLISHING THE DIAGNOSIS

Serum monoclonal protein analysis with immunofixation revealed IgM kappa monoclonal gammopathy with an IgM level of 1,570 (reference range 53–334 mg/dL) and M-spike 0.75 (0.00 mg/dL), serum free kappa light chains 61.1 (3.30–19.40 mg/L), lambda 9.3 (5.7–26.3 mg/L), and kappa-lambda ratio 6.57 (0.26–1.65).

2. Which is the best next step in this patient’s neurologic evaluation?

  • Test CSF angiotensin-converting enzyme level
  • CSF cytology
  • Meningeal biopsy
  • Peripheral nerve biopsy

Given the high suspicion for malignancy, CSF cytology was performed and showed increased numbers of mononuclear chronic inflammatory cells, including a mixture of lymphocytes and monocytes, favoring a reactive lymphoid pleocytosis. Flow cytometry indicated the presence of a monoclonal, CD5- and CD10- negative, B-cell lymphoproliferative disorder. The immunophenotypic findings were not specific for a single diagnosis. The differential diagnosis included marginal zone lymphoma and lymphoplasmacytic lymphoma.

3. Given the presence of serum IgM monoclonal gammopathy in this patient, which is the most likely diagnosis?

  • Neurosarcoidosis
  • Multiple myeloma
  • Waldenström macroglobulinemia
  • Carcinomatous meningitis

Study of bone marrow biopsy demonstrated limited bone marrow involvement (1%) by a lymphoproliferative disorder with plasmacytoid features, and DNA testing detected an MYD88 L265P mutation, reported to be present in 90% of patients with Waldenström macroglobulinemia.9 This finding confirmed the diagnosis of Waldenström macroglobulinemia with central nervous system involvement. Our patient began therapy with rituximab and methotrexate, which resulted in some improvement in strength, gait, and vision.

 

 

WALDENSTRÖM MACROGLOBULINEMIA AND BING-NEEL SYNDROME

Waldenström macroglobulinemia is a lympho­plasmacytic lymphoma associated with a monoclonal IgM protein.10 It is considered a paraproteinemic disorder, similar to multiple myeloma. The presenting symptoms and complications are related to direct tumor infiltration, hyperviscosity syndrome, and deposition of IgM in various tissues.11,12

Waldenström macroglobulinemia is usually indolent, and treatment is reserved for patients with symptoms.13,14 It includes rituximab, usually in combination with chemotherapy or other targeted agents.15,16

Paraneoplastic antibody-mediated polyneuropathy may occur in these patients. However, the pattern is usually symmetrical clinically, with demyelination on EMG, and is not associated with cranial nerve or meningeal involvement. Management with plasmapheresis, corticosteroids, and intravenous immunoglobulin has not been shown to be effective.17

Involvement of the central nervous system as a complication of Waldenström macroglobulinemia has been described as Bing-Neel syndrome. It can present as diffuse malignant cell infiltration of the leptomeningeal space, white matter, or spinal cord, or in a tumoral form presenting as intraparenchymal masses or nodular lesions. The distinction between the tumoral and diffuse forms is based primarily on imaging findings.18

In a report of 44 patients with Bing-Neel syndrome, 36% presented with the disorder as the initial manifestation of Waldenström macroglobulinemia.18 The primary presenting symptoms were imbalance and gait difficulty (48%) and cranial nerve involvement (36%), which presented as predominantly facial or oculomotor nerve palsy. Cauda equina syndrome with motor involvement (seen in our patient) occurred in 14% of patients. Other presenting symptoms included cognitive impairment, sensory deficits, headache, dysarthria, aphasia, and seizures.

LEARNING POINTS

The differential diagnosis for patients presenting with multifocal neurologic symptoms can be broad, and a systematic approach to the diagnosis is necessary. Localizing the lesion is important in determining the diagnosis for patients presenting with neurologic symptoms. The process of localization begins with taking the history, is further refined during the examination, and is confirmed with diagnostic studies. Atypical presentations of relatively common neurologic diseases such as Guillain-Barré syndrome, transverse myelitis, and peripheral polyneuropathy do occur, but uncommon diagnoses need to be considered when support for the initial diagnosis is lacking.

References
  1. Fokke C, van den Berg B, Drenthen J, Walgaard C, van Doorn PA, Jacobs BC. Diagnosis of Guillain-Barre syndrome and validation of Brighton criteria. Brain 2014; 137(Pt 1):33–43. doi:10.1093/brain/awt285
  2. Mathey EK, Park SB, Hughes RA, et al. Chronic inflammatory demyelinating polyradiculoneuropathy: from pathology to phenotype. J Neurol Neurosurg Psychiatry 2015; 86(9):973–985. doi:10.1136/jnnp-2014-309697
  3. Chiba A, Kusunoki S, Obata H, Machinami R, Kanazawa I. Serum anti-GQ1b IgG antibody is associated with ophthalmoplegia in Miller Fisher syndrome and Guillain-Barré syndrome: clinical and immunohistochemical studies. Neurology 1993; 43(10):1911–1917. pmid:8413947
  4. Teener J. Miller Fisher’s syndrome. Semin Neurol 2012; 32(5):512–516. doi:10.1055/s-0033-1334470
  5. Watson JC, Dyck PJ. Peripheral neuropathy: a practical approach to diagnosis and symptom management. Mayo Clin Proc 2015; 90(7):940–951. doi:10.1016/j.mayocp.2015.05.004
  6. Greenberg BM. Treatment of acute transverse myelitis and its early complications. Continuum (Minneap Minn) 2011; 17(4):733–743. doi:10.1212/01.CON.0000403792.36161.f5
  7. West TW. Transverse myelitis—a review of the presentation, diagnosis, and initial management. Discov Med 2013; 16(88):167–177. pmid:24099672
  8. Le Rhun E, Taillibert S, Chamberlain MC. Carcinomatous meningitis: leptomeningeal metastases in solid tumors. Surg Neurol Int 2013; 4(suppl 4):S265–S288. doi:10.4103/2152-7806.111304
  9. Treon SP, Xu L, Yang G, et al. MYD88 L265P somatic mutation in Waldenström’s macroglobulinemia. N Engl J Med 2012; 367(9):826–833. doi:10.1056/NEJMoa1200710
  10. Owen RG, Treon SP, Al-Katib A, et al. Clinicopathological definition of Waldenstrom’s macroglobulinemia: consensus panel recommendations from the Second International Workshop on Waldenstrom’s Macroglobulinemia. Semin Oncol 2003; 30(2):110–115. doi:10.1053/sonc.2003.50082
  11. Björkholm M, Johansson E, Papamichael D, et al. Patterns of clinical presentation, treatment, and outcome in patients with Waldenstrom’s macroglobulinemia: a two-institution study. Semin Oncol 2003; 30(2):226–230. doi:10.1053/sonc.2003.50054
  12. Rison RA, Beydoun SR. Paraproteinemic neuropathy: a practical review. BMC Neurol 2016; 16:13. doi:10.1186/s12883-016-0532-4
  13. Kyle RA, Benson J, Larson D, et al. IgM monoclonal gammopathy of undetermined significance and smoldering Waldenström’s macroglobulinemia. Clin Lymphoma Myeloma 2009; 9(1):17–18. doi:10.3816/CLM.2009.n.002
  14. Kyle RA, Benson JT, Larson DR, et al. Progression in smoldering Waldenstrom macroglobulinemia: long-term results. Blood 2012; 119(19):4462–4466. doi:10.1182/blood-2011-10-384768
  15. Leblond V, Kastritis E, Advani R, et al. Treatment recommendations from the Eighth International Workshop on Waldenström’s macroglobulinemia. Blood 2016; 128(10):1321–1328. doi:10.1182/blood-2016-04-711234
  16. Kapoor P, Ansell SM, Fonseca R, et al. Diagnosis and management of Waldenström macroglobulinemia: Mayo stratification of macroglobulinemia and risk-adapted therapy (mSMART) guidelines 2016. JAMA Oncol 2017; 3(9):1257–1265. doi:10.1001/jamaoncol.2016.5763
  17. D’Sa S, Kersten MJ, Castillo JJ, et al. Investigation and management of IgM and Waldenström-associated peripheral neuropathies: recommendations from the IWWM-8 consensus panel. Br J Haematol 2017; 176(5):728–742. doi:10.1111/bjh.14492
  18. Simon L, Fitsiori A, Lemal R, et al. Bing-Neel syndrome, a rare complication of Waldenström macroglobulinemia: analysis of 44 cases and review of the literature. A study on behalf of the French Innovative Leukemia Organization (FILO). Haematologica 2015; 100(12):1587–1594. doi:10.3324/haematol.2015.133744
References
  1. Fokke C, van den Berg B, Drenthen J, Walgaard C, van Doorn PA, Jacobs BC. Diagnosis of Guillain-Barre syndrome and validation of Brighton criteria. Brain 2014; 137(Pt 1):33–43. doi:10.1093/brain/awt285
  2. Mathey EK, Park SB, Hughes RA, et al. Chronic inflammatory demyelinating polyradiculoneuropathy: from pathology to phenotype. J Neurol Neurosurg Psychiatry 2015; 86(9):973–985. doi:10.1136/jnnp-2014-309697
  3. Chiba A, Kusunoki S, Obata H, Machinami R, Kanazawa I. Serum anti-GQ1b IgG antibody is associated with ophthalmoplegia in Miller Fisher syndrome and Guillain-Barré syndrome: clinical and immunohistochemical studies. Neurology 1993; 43(10):1911–1917. pmid:8413947
  4. Teener J. Miller Fisher’s syndrome. Semin Neurol 2012; 32(5):512–516. doi:10.1055/s-0033-1334470
  5. Watson JC, Dyck PJ. Peripheral neuropathy: a practical approach to diagnosis and symptom management. Mayo Clin Proc 2015; 90(7):940–951. doi:10.1016/j.mayocp.2015.05.004
  6. Greenberg BM. Treatment of acute transverse myelitis and its early complications. Continuum (Minneap Minn) 2011; 17(4):733–743. doi:10.1212/01.CON.0000403792.36161.f5
  7. West TW. Transverse myelitis—a review of the presentation, diagnosis, and initial management. Discov Med 2013; 16(88):167–177. pmid:24099672
  8. Le Rhun E, Taillibert S, Chamberlain MC. Carcinomatous meningitis: leptomeningeal metastases in solid tumors. Surg Neurol Int 2013; 4(suppl 4):S265–S288. doi:10.4103/2152-7806.111304
  9. Treon SP, Xu L, Yang G, et al. MYD88 L265P somatic mutation in Waldenström’s macroglobulinemia. N Engl J Med 2012; 367(9):826–833. doi:10.1056/NEJMoa1200710
  10. Owen RG, Treon SP, Al-Katib A, et al. Clinicopathological definition of Waldenstrom’s macroglobulinemia: consensus panel recommendations from the Second International Workshop on Waldenstrom’s Macroglobulinemia. Semin Oncol 2003; 30(2):110–115. doi:10.1053/sonc.2003.50082
  11. Björkholm M, Johansson E, Papamichael D, et al. Patterns of clinical presentation, treatment, and outcome in patients with Waldenstrom’s macroglobulinemia: a two-institution study. Semin Oncol 2003; 30(2):226–230. doi:10.1053/sonc.2003.50054
  12. Rison RA, Beydoun SR. Paraproteinemic neuropathy: a practical review. BMC Neurol 2016; 16:13. doi:10.1186/s12883-016-0532-4
  13. Kyle RA, Benson J, Larson D, et al. IgM monoclonal gammopathy of undetermined significance and smoldering Waldenström’s macroglobulinemia. Clin Lymphoma Myeloma 2009; 9(1):17–18. doi:10.3816/CLM.2009.n.002
  14. Kyle RA, Benson JT, Larson DR, et al. Progression in smoldering Waldenstrom macroglobulinemia: long-term results. Blood 2012; 119(19):4462–4466. doi:10.1182/blood-2011-10-384768
  15. Leblond V, Kastritis E, Advani R, et al. Treatment recommendations from the Eighth International Workshop on Waldenström’s macroglobulinemia. Blood 2016; 128(10):1321–1328. doi:10.1182/blood-2016-04-711234
  16. Kapoor P, Ansell SM, Fonseca R, et al. Diagnosis and management of Waldenström macroglobulinemia: Mayo stratification of macroglobulinemia and risk-adapted therapy (mSMART) guidelines 2016. JAMA Oncol 2017; 3(9):1257–1265. doi:10.1001/jamaoncol.2016.5763
  17. D’Sa S, Kersten MJ, Castillo JJ, et al. Investigation and management of IgM and Waldenström-associated peripheral neuropathies: recommendations from the IWWM-8 consensus panel. Br J Haematol 2017; 176(5):728–742. doi:10.1111/bjh.14492
  18. Simon L, Fitsiori A, Lemal R, et al. Bing-Neel syndrome, a rare complication of Waldenström macroglobulinemia: analysis of 44 cases and review of the literature. A study on behalf of the French Innovative Leukemia Organization (FILO). Haematologica 2015; 100(12):1587–1594. doi:10.3324/haematol.2015.133744
Issue
Cleveland Clinic Journal of Medicine - 86(6)
Issue
Cleveland Clinic Journal of Medicine - 86(6)
Page Number
374-379
Page Number
374-379
Publications
Publications
Topics
Article Type
Display Headline
A 69-year-old woman with double vision and lower-extremity weakness
Display Headline
A 69-year-old woman with double vision and lower-extremity weakness
Legacy Keywords
double vision, diplopia, weakness, cerebral infarction, stroke, Guillain-Baré syndrome, GBS, neuropathy, polyneuropathy, transverse myelitis, radiculopathy, monoclonal gammopathy, neurosarcoidosis, multiplemyeloma, Waldenström macroglobulinemia, Bing-Neel syndrome, Ibrahim Migdady, Maryann Mays, Kerry Levin
Legacy Keywords
double vision, diplopia, weakness, cerebral infarction, stroke, Guillain-Baré syndrome, GBS, neuropathy, polyneuropathy, transverse myelitis, radiculopathy, monoclonal gammopathy, neurosarcoidosis, multiplemyeloma, Waldenström macroglobulinemia, Bing-Neel syndrome, Ibrahim Migdady, Maryann Mays, Kerry Levin
Sections
Disallow All Ads
Content Gating
No Gating (article Unlocked/Free)
Alternative CME
Disqus Comments
Default
Gate On Date
Tue, 05/28/2019 - 15:15
Un-Gate On Date
Tue, 05/28/2019 - 15:15
Use ProPublica
CFC Schedule Remove Status
Tue, 05/28/2019 - 15:15
Hide sidebar & use full width
render the right sidebar.
Article PDF Media